You are on page 1of 82

A1 PASSERS TRAINING, RESEARCH, REVIEW & DEVELOPMENT COMPANY

2nd Floor Sommerset Bldg., Lopez Jaena St. Jaro, Iloilo City
Tel No.: (033) 320-2728; 09106547262
Email Address: a1nursingreviewic@yahoo.com.ph

MEDICAL TECHNOLOGY DRILLS AND PRACTICE TESTS


CLINICAL CHEMISTRY
1.a volumetric pipette has an accuracy of:
a. 1:10
c. 1:500
b. 1:100
d. 1:1000
2.A pipette with a bulb to the delivery lip is used for viscous fluids and is called a:
a. Mohr pipette
c. Lewis-dahn pipette
b. Volumetric pipette
d. Ostwald- Follin pipette
3.The etched rings on the top of a pipette means:
a. The pipette should be allowed to drain and the last drop should remain in the pipette
b. The last drop is to be blown out after the pipettes drains
c. The pipette is color coded
d. The pipette is a volumetric pipette
4.Standard solution from which 99.95% of the chemical can be retrieved are referred to as:
a. Secondary standards
c. Lycphilized standards
b. Primary standards
d. Preset standards
5.When preparing solutions:
a. A percent solution can be prepared from a volumetric solution
b. A volumetric solution can be prepared from a percent solution
c. A triple beam balance and cylinder are necessary in the preparation of any type of solution
d. Only an analytical balance can be used in solution preparation
6.A solution in which the molecules of solute in solution are in equilibrium with excess undissolved molecules
referred to as:
a. c. Saturated
b. Concentrated
d. Supersaturated
7.Pure water has pH of:
a. 5.0
c. 7.4
b. 7.0
d. 9.0
8.A 1x10-6 N solution of hydrochloric acid (HCI) has a pH of:
a. 4
c. 8
b. 6
d. 12
9.The statistical term that most specifically describes annlytical precision is the:
a. Mode
c. Median
b. Mean
d. Coefficient of varation
10. Quality control should be used with ali procedures to determine:
a. Accuracy
c. Precision
b. Reliability
d. Coefficient of varation
11. 12. A procedure has a 2 SD variation from a mean value. How many values from a normal population will the
2.30 include:
a. 68.27%
c. 95.45%
b. 84.76%
d. 99.73%
13. Material with physical and chemical properties closely resembling the test specimen and containing
concentrations of the substances being measured in known as:
a. Control
c. Calibrator
b. Standard
d. Reference
14. Optical density according to Beers law, is:
a. Inversely proportional to the concentration
b. Directly proportional to the concentration
c. Proportional to the square of the concentration
d. Proportional to the square root of the concentration
15. The difference between serum and plasma is the serum does not contain:
a. Thrombin
c. Fibrinogen
b. Fibrin
d. Calcium
16.

A1 PASSERS REVIEW CENTER 320-2728 //

17. Two control sera are desirable for each general chemistry procedure. Which of these combinations would be
appropriate?
a. Mean of normal range and abnormal
c. Low normal range and abnormal (low)
(high)
d. Abnormal (low) and abnormal (high)
b. High normal range and abnormal (high)
18. Sodium fluoride is used in specimen collection to:
a. Prevent glycolysis
c. Chelate calcium
b. Prevent conversion of prothrombin to
d. Bind calcium
thrombin
19. The concentration of oxalate recommended to be used as an anticoagulant, in mg/mL blood, is:
a. 1-2
c. 4-5
b. 2-4
d. 6-8
20. Ethylenediamine tetraacelic acid (EDTA) works as anticoagnant by:
a. Chelating calcium
b. Preventing conversion of prothrombin to thrombin
c. Forming an insoluble complex with calcium
d. Binding magnesium
21. The venipuncture site for a rouline puncture is commonly cleaned with:
a. 90% alcohol
c. 70% alcohol
b. BETADINE (Purdue-Frederick)
d. Quatemary ammonlum compounds
22. The one precaution to be followed with a specimen for billirubin determination is to:
a. Refrigerate
c. Project from light
b. Cap and refrigerate
d. Freeze immediately
23. Which of these substances cannot be preserved by freezing?
a. Blood urea nitrogen
c. Lactic dehydrogenase
b. Creatinine kinase isoenzyme
d. Prostatic acid phosphatase
24. Serum iron should be drawn at the same time on successive days to avoid
a. Diumal variation
c. Effect of medication
b. Chance of consumption of dietary iron
d. Gastrointestinal absorption of iron
25. Which of the proteins is soluble in water?
a. Albumin
c. Gamma globulins
b. Beta globulins
d. Histones
26. The alpha amino acids constitute the basic building blocks of proteins. The amino acids are joined together
to form a chainlike structure through:
a. Disulfide bonding
c. Covalent bonding
b. Hydrogen bonding
d. Peptide bonding
27. Which of the four elements in proteins differentiate this class of substances from carbohydrates and lipids?
a. Carbon
c. Nitrogen
b. Hydrogen
d. Oxygen
28. Which amino acids cannot rotate polarized light?
a. Cystine
c. Histidine
b. Glycine
d. Thionine
29. Proteins may become denatured when subjected to mechanical agitation , heat or extreme chemical
treatment. Denaturation of proteins refers to an
a. Alteration in primary structure
c. Alteration in tertiary structure
b. Alteration in secondary structure
d. Increase in solubility
30. The haemoglobin measurement technique employed for a blood donors is
a. Refractive index
c. Flotation technique of Philips
b. Hemoglobin electrophoresis
d. Falling drop technique
31. The ration of velocities of light in two different media refers to the:
a. Density gradient
b. Refractive index
c. Rotation of polarized light
d. Ultraviolet absorption

A1 PASSERS REVIEW CENTER 320-2728 //

32. The most anodic protein on electrophoresis at pH 8.6 is

35.
36. The macroglobulins belong to what sedimentation class?
a. 4.5 S
c. 12.0S
b. 7.0S
d. 19.0S
37. As urine in heated, a precipitate appears at 60 C and disappears at 100C. The substance present is a
a. Macroglobulin
b. Cryglobulin
c. Bence Jones protein
d. Protease
38. The process of separating albumin from globulins using sodium sulfate is known as
a. Flatation
c. Salting out
b. Protein precipitation
d. Ultracentrifugation
39. The conversion of protein nitrogen content depends on the factor
a. 6.25
c. 6.63
b. 6.54
d. 6.78
40. Which of the following is not true of the biuret reaction?
a. Follows Beers law for a reasonable range of protein
b. Depends on the presence of at least two peptide linkages
c. Is relatively free from int reference by lipids and haemoglobin
41. In acute renal failure which non protein nitrogen rises the fastest?
a. Uric acid
c. Creatinine
b. Blood urea nitrogen
d. Creatine
42. Why is bromcresol purple the preferred indicator of albumin dye binding technics?
a. It lends itself to amnual and automated procedures best
b. It never binds with alpha and bea globulins
c. It can be employed with heparinised plasma
d. There is less interference from pigment
43. Which reagent is employed in the serum protein determination?
a. Molyddenum blue
c. Resorcinol HCI
b. Ferrifarrocyanide
d. Biuret
44. Hemolyzed serum should not be used for total protein because
a. Haemoglobin is absorbed at the same wavelength as protein biuret reagent
b. Of elevated protein amounts in red blood cells as compared to serum
c. Haemoglobin reacts with biuret reagent
d. The value will be falsely lowered
45. Turbidity in serum is associated with the presence of
a. Cholesterol
c. Free fatty acids
b. Chylomicrons
d. Total lipids
46. Glycoproteins and mucoproteins are usually bound to
a. Lactose and sucrose
c. Galactose and mannose
b. Lactose and maltose
d. Glucose and sucrose
47. All of the following are glycoproteins, except
a. Ceruloplasmin
c. Fibrinogen
b. Follicle stimulating hormone
d. Cryoglobulin
48. Which of these statements is not application to mucoproteins?
a. They are proteins complexed with carbohydrates
b. They are related to glycoproteins
c. They are contain more than 4% hexosamine
d. Hormones and coagulation proteins are included in this class
49. How many immunoglobulin classes are currently recognized?
a. One
c. Four
b. Three
d. Five
50. When performing an immunoglobulin electrophoresis, there must be present:

A1 PASSERS REVIEW CENTER 320-2728 //

a. Excess antigen
b. Excess antibody and constant trough distance between antibody and antigen
c. Acelate matrix
d. Direct current
51. A normal serum protein electrophoresis has approximately 60% albumin and 5-10% each of other 4
fractions. If an electrophoretic pattern shows 30% albumin 4-10% of other fractions except gamma which is
45%, you would expect what condition to exist in the patient?
a. Cirrhosis
c. Inflammation
b. Monoctonal gammapathy
d. Chronic lymphocytic leukemia
52.
53.
54. Which of the following applies to cryoglobulins?
a. They are temperature sensitive proteins
b. The blood specimen should clot at room temperature
c. The sample unit must remain at room temperature so that precipitation can be observed
d. Only qualitative tests are available
55. Which of the following statements is true of albumin?
a. Compared to globulin , it makes up the lesser portion of total protein
b. Its size prevents its passage through even a damaged glomerular banier
c. It is produced in the liver
d. Clinical problems are usually related to high serum values
56. To measure urinary protein you can use
a. Biuret reagent
b. Biuret reagent with trichloroacetic acid pretreatment of urine
c. Bromcresol purple
d. Bromcresol green
57. An elevated creatinine value is most likely to be accompanied by which of these values?
a. Normal uric acid
b. Elevated uric acid , 10x creatinine value
c. Normal blood area nitrogen
d. Elevated blood area nitrogen, 10x creatinine value
58. The conversation factor for blood urea nitrogen to urea is:
a. 1.10
c. 3.14
b. 2.14
d. 6.25
59. By the urease method , urea is enzymatically converted to:
a. Ammonia
c. Nitrogen
b. Carbon dioxide
d. Amino groups
60. Untreated urine can be used for the determination to urea by the diacetyl monoxide method because
a. Dialysis in an autoanalyzer removes interfering substances
b. The method is not measuring ammonia
c. The increased temperature destroys the ammonia
d. Zinc does not interfere
61. Urinary creatine may be elevated in which of the following?
a. Kidney failure
c. Muscle destruction
b. Cirrhosis of the liver
d. Intestinal blockage
62. The measurement of creatinine is based on the formation of yellow red color with:
a. Alkaline picrate
c. Phosphomolybdate
b. Ehrlich reagent
d. Titan yellow
63. The classic creatinine reaction is that of
a. Jaffe
c. Kjeldahl
b. Lloyd
d. Nessler
64. The specificity of the Jaffe reaction can be enhanced by:
a. Adsorption with Lloyds reagent
c. Prior treatment with urease
b. Boiling
d. Prior treatment with uricase
65. A disease state associated with an elevation of serum uric acid is
a. Atherosderosis
c. Diabetes
b. Arthritis
d. Gout
66. The most common reagent employed in uric acid methodology by alkaline oxidation is
a. Potassium permanganate
c. Potassium persulfate
b. Phosphotungstic acid
d. Hydrochloric acid (HCI)
67. The uricase method for uric acid assay depends on
a. Ultraviolet absorption peak at 290 nm before and after treatment with uricase
b. Collection of evolved gas after uricase treatment

A1 PASSERS REVIEW CENTER 320-2728 //

c. Increase in absorption after uricase treatment


d. Measurement of blue violet color
68. The most precise method for amino acid determination is
a. Isothermal distillation
d. Color development with Berthelot
b. Gasometric ninhydrin reaction
reagent
c. Alkaline oxidation
69. The urea clearance test is an indication of
a. Overall kidney function
c. Tubular function
b. Tubular reabsorption
d. Glomerular filtration
70.
71.
72. The creatinine clearance is based on the assumption that creatinine
a. Passes into the ultrafiltrate
d. Is passively reabsorbed by the kidney
b. Is retained in the blood
tubule
c. Is converted to creatine
73. Which of the clearance tests offers the most accurate measure of glomerular filtration?
a. Inulin
c. Urea
b. Creatinine
d. P-aminohippurate (PAH)
74. Which of the clearance tests offers the best measure of tubular function?
a. Creatinine
c. P-aminohlppurate (PAH)
b. Inullin
d. Urea
75. A creathine clearance tests is performed. The 24 hour volume of urine is 770ml: serum creatinine is 2.0
mg/dL and urine creatinine is 240mg/dL. What is the clearance expressed in mL plasma cleared per minute,
assuming average body surface?
a. 6.0
c. 60.0
b. 38.4
d. 924.0
76. The usual determination of osmolality involves measurement of
a. pCO2 and pO2
c. Electrolytes
b. Na+ and K+
d. Freezing point depression
77. Which of the following is true of blood glucose?
a. The renal threshold is 200-240 mg/dL
b. Once excreted in the glomerular filtrate, glucose must be excreted in the urine
c. A high blood glucose must be associated with a positive urine glucose
d. In the glucose tolerance test (GTT), one may be faced with a normal blood glucose, accompanied by
a positive urine glucose
78. Patients with borderline blood glucose levels are further investigated by performing
a. Postprandial sugar (PPS)
c. Inulin tolerance test
b. Fasting blood sugar
d. 3 hour glucose tolerance test (GTT)
79. The test that gives a two or three (2-3) month picture of a diabetes glucose levels it
a. Haemoglobin A1
c. Haemoglobin A1 C
b. 2 hour postprandial sugar
d. 3 hour glucose tolerance test
80. Which of the following precautions is necessary to insure validity of the glucose tolerance tests?
a. A 150 gram/day carbohydrate diet for 3 days prior to the test
b. A fasting state before glucose is given
c. No undue stress before or during the test
d. All of the above
81. Contamination of the glucose reagent with catalase is a problem with which glucose method?
a. Alkaline cupric ion reduction
c. Glucose oxidase
(neocuproine)
d. Hexokinase
b. Alkaline ferric ion reduction
(ferricyanide)
82. The hexokinase methodology for glucose determination is base on
a. Reduced coenzyme read at 340 nm
b. Rose pink color of oxygen o diansidine
c. Peroxidise acting upon H2O2 released from glucose
d. Protein free filtrate
83. Proteins are precipitated by which of the following reagents in the Folin-Wu method?
a. Tungstic acid
c. Sulfuric acid
b. Trionloroacetic acid
d. Hydrochloric acid
84. The interfering reducing substances in the Folin-Wu method?
a. Dialysis
c. Zinc barium precipitation
b. Tungstate precipitation
d. Heating
85. In the copper reduction method for glucose, measurable color is developed through the formation of

A1 PASSERS REVIEW CENTER 320-2728 //

a. Cooper sulfate
c. Copper cobalt complex
b. Molybdenum blue
d. Phosphomolybdenum
86. The most frequently employed automated method for glucose uses
a. Coupled enzyme system consisting of glucose oxidase and catalase
b. Reduction of alkaline cupric ion
c. Addition of molybdenum blue
d. Reduction of alkaline ferricyanide to ferrocyanide
87. When sugars are separated by chromatography , the application of anisaldehyde/sulfuric acid/ethyl alcohol
spray will identify galactose as what color spot?
a. Blue
c. Green
b. Grey
d. Yellow
88. Saliwanoffs test detects
a. Dextrose
c. Lactose
b. Glucose
d. Fructose
89. Xylose excretion is helpful in the determination of
a. Intestinal malabsorption
c. Pancreatic secretion
b. Renal clearance
d. Liver failure
90. Prehepatic bilirubin has which of the following for a protein carrier?
a. Albumin
c. Beta globulin
b. Alpha2 globulin
d. Gamma globulin
91. An increase in indirect reacting bilirubin is suggestive of
a. Bile acid build up
c. Hemoglobin breakdown
b. Lipid accumulation
d. Gamma bound to red blood cells
92. Direct reacting bilirubin is
a. Free bilirubin
c. Bilirubin diglucuronide
b. Bilirubin bound to albumin
d. Bilirubin bound to red blood cells
93. Bilirubin is converted in the intestines to which of the following
a. Porphobilinogen
c. Urobilinogen
b. Haemoglobin
d. Phospholipid
94. Which of the following interfere with the Evelyn and Malloy method for bilirubin?
a. Lipenia
c. Anticonvulsants
b. Hemolysis
d. Alcohol
95. The port wine color of some urines can be attributes to
a. Porphyrins
c. Red blood cells
b. Melanin
d. Billirubin
96. Para dimethylaminobenzaldehyde is an constituent of which reagent?
a. Ehrlich
c. Drabkin
b. Salkowski
d. Bloor
97. Both porphobilinogen and urobilinogen form a red-colored compound with Ehrlich reagent. Differentiation
can be obtained by
a. Solubility of porphobilinogen in
c. Difference in UV absorption
chloroform
d. Solubility of urobilinogen in chloroform
b. Solubility of urobillnogen in water
98. Bilirubin will develop a violet color after which of the following is added:
a. Hydrochloric acid
c. Methyl alcohol
b. Ehrlich reagent
d. Van den Beighs reagent
99. A urine specimen that exhibits yellow foam on being shaken should be suspected of having an increased
concentration of
a. Ketones
c. Bilirubin
b. Glucose
d. Nitrite
100. An enzyme may be defined as a/an
a. Biological catalyst
c. Metal catalyst
b. Inorganic catalyst
d. Large molecule
101. Hemolysis will interfere with all of the following enzyme measurements except
a. Alkaline phosphate
c. Aldolase
b. Lactate dehydrogenese
d. Aspartate aminotransferase
102. A high alkaline phosphatase level in the presence of other negative liver function tests is indicative of
a. Obstructive jaundice
c. Carcinoma of the liver
b. Hepatitis
d. Hemolytic disorder
103. A metal ion is sometimes necessary in an enzymatic reaction to
a. Act as an activator of the enzyme
c. Regulate pH
b. Permit the colorimetric reaction to
d. Inhibit competing enzymes
occur

A1 PASSERS REVIEW CENTER 320-2728 //

104. Which of the following is true of an isoenzyme


a. The substrate is different from each isoenzyme
b. The rate of reaction is the same for each enzyme with its own substate
c. The electrophoretic property is different for each isoenzyme
d. Any of a group of isoenzymes will react the same to heat denaturation
105. As the temperature is increases from 25C to 37C is the aspartate aminotransferase reaction, the activity of
the reaction
a. Almost doubles
c. Is tripled
b. Is halvec
d. Is reduced to zero
106.
107.
108. The Michaelis- Menten constant in the rate of conversion of substrate to product is determined by
a. Substrate concentration
b. Substrate concentration and rate dissociation of enzyme substrate complex
c. Temperature , pH and substrate concentration
d. Temperature pH
109. In a zero order reaction, there will be no further increase velocity because
a. There is no excess substrate
c. All enzyme is bound to substrate
b. The temperature is 35C
d. There is excess enzyme
110. Lipase catalyzes the hydrolysis of
a. Starch
c. Glycerides
b. Gelatine
d. Sugar
111. Which of the following is true of the isoenzyme LD-1 it is
a. The slowest moving of the lactate dehydrogenese LD isoenzymes electrophoresis
b. The most positively charged fraction
c. A heat labile fraction
d. Present in the greatest amount in normal heart issue
112. Lactate + NAD +pyruvate+NADH. The catalyst in the reaction is
a. Aspirate aminotransferase AST
c. Lactate dehydrogenase (LDH)
b. Aldolase
d. Alanine aminotransferase ALT
113. Serum creatine kinase is clinically significant in diseases of the
a. Pancrease
c. Muscle
b. Liver
d. Gonads
114. Creatine kinase is frequently elevated in disorders such as acute myocardial infarction MI and
a. Duchenna muscular dystrophy
c. Cirrhosis
b. Acute hepatitis
d. Infections
115. The kinetic or rate enzymatic methodologies optical density decreases as
a. Triphosphopyridine mucleatide TPN is changed to triphosphopyridine nucleotide reduced form TPNH
b. Nicotinamide adenine dinucleotide NAD is changed to nicotinamide adenine dinucleotide reduced
form NADP
c. Nicotinamide adenine dinucleotide NADP is changed to nicotinamide adenine dinuclectide NAD
d. Ceric ion is change to cerous
116. In myocardial infarction, the clinically significant enzymes are
a. Creatine kinase, aspirate amino transferaase, alnine aminotransferase
b. Creatine kinase, aspartate aminotransferase, lactate dehydrogenase
c. Aldolase, aspartate aminotransferase, alanine aminotransferase
d. Lactate dehydrogenase, alanine aminotransferase, aspartate aminotransferase, aldolase
117. During a suspected myocardial infarction you should perform creatine phosphokinase isoenzyme and
lactate dehydrogenase isoenzymes to demonstrate the:
a. MB fraction of creatine kinase and LD-5
b. MB fraction of creatine kinase and the LD 1-2 flip
c. LD-5 and LD-3
d. BB fraction of creatine kinase and LD-1
118. In muscular dystrophy , the clinically significant enzymes are
a. Creatine kinase, aldolase , aspartate aminotransferase
b. Alanine aminotransferase, aldolase, lactate dehydrogenase
c. Creatine kinase, aspartate aminotransferase , alamine aminotransferase
d. Aldolase aspartate aminotransferase , alanine aminotransferase
119. Ceruloplasmin is an alpha, globulin that binds
a. Copper
c. Iron
b. Cobalt
d. Manganese
120. Possibly the most sensitive enzyme indicator of liver function, particularly in obstructive jaundice is;
a. Alkaline phosphatase
b. Alanine aminotransferase

A1 PASSERS REVIEW CENTER 320-2728 //

c. Acid phosphatase
d. Lactate dehydrogenase
121. In acute viral hepatitis which of the following would not be suspected?
a. Alkaline phosphatase increase greater than aspartate aminotransferase
b. Lactate dehydrogenase -5 increase
c. Gamma glutamyl transferase increase mild
d. Aspartate aminotransferase and alamine minotransferase increae 10 to 200 fold
122. What is the optimum pH for acid phosphatase?
a. 3.0
b. 5.0
c. 7.0
d. 9.0
123. The clinical significance of an elevated acid phosphatase is
a. Renal disease
c. Bone disease
b. Pancreatic disease
d. Prostatic disease
124. If an acid phosphatase test cannot be performed immediately after collection , how should the serum be
treated?
a. Acidity to pH of 2
c. Cap and freeze
b. Freeze
d. Either acidity or freeze
125. Tests for acid phosphatase detect levels of the enzyme for tissues such as
a. Bone, liver, spleen, kidney, erythrocytes and platelets
b. Bone, liver and kidney
c. Muscle, spleen and platelets
d. Liver, spleen, prostate, and platelets
126. The substrate for the cherry-crandall lipase method is
a. Glyceride
c. Olive oil
b. Nitrophenyl acetate
d. Starch
127. Because ionized calcium can be changed without affecting the total calcium level, what other parameters
must be known to property evaluate calcium?
a. pH
c. Magnesium
b. Phosphorous
d. pH and protein
128. The classic clark-collip method for calcium is based on the assumption that calcium will be
a. Precipitated as calcium carbonate
b. Precipitated as an oxalate and converted to an oxide
c. Chelated with EDTA
d. Precipitated as an oxalate and converted to oxalic acid
129. In the titration of calcium oxalate with potassium permanganate, the temperature must be maintained at
a. 1-4 C
c. 60-80C
b. 37C
d. 100C
130. A low parathyroid hormone level will cause a
a. Low serum phosphorous
c. Low serum calcium
b. High urine calcium level
d. High serum calcium level
131. In the compleximetric titration (EDTA) method for calcium the pH must be adjusted to prevents
a. Incomplete precipitation of calcium
c. A reversible color change
b. Interference by magnesium
d. Co binding of phosphorous
132. What is the purpose of lanthanium in the atomic absorption determination of calcium? Lanthanium:
a. In involved in starting up the instrument
c. Will bind calcium
b. Is part of the hollow cathode discharge
d. Will bind phosphate
tube
133. All of the following can cause low chloride levels , EXCEPT
a. Diabetic acidosis
c. Prolonged vomiting
b. Renal failure
d. Dehydration
134. Inorganic phosphate can be determined from a combination of trichloroscetic acid filtrate and
a. Stannous chloride
c. Tungstate
b. Glacial acetic-sulfuric acid
d. Molybdate
135. Al o pH of 7.4 most of the inorganic phosphate in the plasma is in which form?
a. HPO4
c. H2PO4
b. PO4
d. P2O5
136. A phosphorous value of 2.5 mEq/L would be how many mg/dL (At. W P=31)?
a. 4.3
c. 8.5
b. 5.2
d. 2.5
137. The colometric method for magnesium employs which reagent?
a. Molybdate
c. Titan yellow
b. Calcium oxalate
d. 8-hydroxy-5-quinoline
138. The first step in the serum iron method involves
a. Breakup of the iron protein complex
b. Reaction of iron with a chromogen

A1 PASSERS REVIEW CENTER 320-2728 //

c. Addition of excess iron for protein to absorb


d. Measurement of amount of free serum iron
139. The major extracellular cation is
a. Chloride
c. Potassium
b. Manganese
d. Sodium
140. The major intracellular cation is
a. Chloride
c. Potassium
b. Manganese
d. Sodium
141. Which of these potassium values would be termed hypokalemia
a. 3.0mmol/L
c. 4.7 mmol/L
b. 4.0 mmol/L
d. 1.5 mmol/L
142. What is the internal standard when sodium and potassium are measured by flame photometry?
a. Calcium
c. Magnesium
b. Lithium
d. Copper
143. In the classic schales and schales (mercurimetric utration) method for chloride, what substrate reacts with
the indicator to form a violet color?
a. HgCI2
c. Excess Hg
b. HNO3
d. Br
144. In the coulometric amperometric method for chloride the amount of chloride is measured by AgCI produced
a. Time needed to reach the titration endpoint
b. Amount of current needed to generate Ag
c. Rate of generation of current
d. Time needed to reach the titration endpoint
145. In gasometric analysis of CO2 the liberated gas is absorbed with
a. Lactic acid
c. Sulphuric acid
b. Hydrochloric acid
d. Sodium hydroxide
146. What is the purpose of caprylic alcohol in the gasometric method for CO 2? To :
a. Prevent foaming
b. Release CO2
c. Absorb CO2
d. Maintain the atmospheric pressure at a constant value
147. The normal pH of blood is
a. 6.50-7.50
c. 7.35-7.45
b. 7.05-7.35
d. 7.45-7.65
148. Which of the following is the Henderson Hasseld which equation?
a. pK3=pH+log
b.

dissoclated salt
undissociated salt

dissoclated salt

c. pH=pK4+log undissociated salt


d.

e. pH= pKa +

f.

dissociated
salt
g. pH=pKa+log
log

149. The ratio of bicarbonate : carbonic acid in normal plasma is


a. 1:20
c. 10:1
b. 5:1
d. 20:1
150. Most of the CO2 in the blood present in which form?
a. Dissolved CO2
c. Calcium carbonate
b. Bicarbonate ion
d. Ammonium carbonate
151. What will happen if blood is exposed to air during collection for pH and blood gas studies?
a. CO2 content increases
c. PO2 decreases
b. pH decreases
d. pCO2 decreases
152. if the glass electrode of the pH instrument is a coated with protein , the pH value will be
a. too high
c. unchanged
b. too low
d. variable
153. Which of the following is true of the pco2 electrode/
a. Only charged particles can cross the membrane
b. Both charged and uncharged particles can cross the membrane
c. The actual pH of the sample has no effect
d. There is no calibration available for the electrode because gases are involved
A1 PASSERS REVIEW CENTER 320-2728 //

154. The wavelength at which oxyhemoglobin and reduce haemoglobin have the same absorbance is referred to
as the:
a. Differential point
c. Equivalence point
b. Isoelectric point
d. Equilibration point
155. Which of these statements is appropriate for the plotting of a concentration curve? On
a. Semi log paper, plot % units on y axis and standard concentration on x axis
b. Semi log paper, plot standard concentration on y axis and absorbance on x axis
c. Linear graph paper, plot absorbance on x axis and standard concentration on y axis
d. Linear graph paper plot, plot absorbance on y axis and % on T on x axis
156. When performing spectrophotometer quality control checks, the holmium oxide glass filter is used to
assess:
a. Stray light
c. Absorbance accuracy
b. Linearity
d. Wavelength accuracy
157. For the ultraviolet range, which of these must be employed in the spectrophotombler?
a. Diffraction grating
c. Quartz cuvette
b. Glass prism
d. Tungsten lamo
158.
159. Measurement by flame photometry involves
a. Absorption of energy when an element is ionized
b. Electrometric titration
c. Colorimetricanalysis
d. Emission of a color when an element is burned
160. Atomic absorption spectroscopy is based on the measurement of
a. Light given off by excited atoms
b. Light absorbed at wavelength of resonance line by unexcited atoms
c. Energy emitted by ultraviolet treated atoms
d. Energy emitted by infrared treated atoms
161. What is the light source in atomic absorption instrumentation?
a. Cathode lamp
c. Grating
b. Flame
d. Prism
162. Quenching is a disadvantage encountered in
a. Fluorometry
c. Flame photometry
b. Atomic adsorption
d. Nephelometry
163. To achieve the best levels of sensitivity and specificity , a type of detector system that a gas
chromotograph may be coupled to is a/an
a. Fluorescence detector
c. Bichromatic spectrophotometer
b. Ultraviolet spectrophotometer
d. Mass spectrophotometer
164. Automated chemistry systems. Whether discrete or continuous flow, must include
a. Unknown, reagent, mixing of the previous two mentioned ingredients, reaction analysis, calculation
and reporting of results
b. Unknown, mixing, incubation, calculation, and reporting of results
c. Standards, mixing, incubation, and reporting of results
d. Unknown , mixing , incubation and reporting of results
165. Is an isotope has a haft life of 12 hours and an activity of 24 millicuries (mCi), what will be its activity in 36
hours?
a. 3 mCi
c. 9mCi
b. 6mCi
d. 12mCi
166. Concentrated sulphuric acid has normality of
a. 12
c. 18
b. 16
d. 36
167. Match the following types of instruments with the description
1. Mixtures can be separated into individual compounds to be measured on the basis of differences in
their physical characteristics
2. Analyses are measured in a sample by means of tubing diameter and flow rate
3. Samples and reagents are pipette into curvettes read in photometer and results are calculated
4. Light absorbed by an element as it falls from a high energy state to a low energy state is converted into
the concentration of the element
5. The light emitted by an element as it falls from a high energy state to alow energy state in converted
into the concentration of the element
a. Atomic absorption
d. Continues flow analysis
b. Discrete processing system
e. Chromatography
c. Flame photometry
168. The major binding protein T-4 is:

A1 PASSERS REVIEW CENTER 320-2728 //

10

a. Albumin
c. Thyroid binding prealbumin (TBPA)
b. Thyroid binding globulin (TBG)
d. None, T-4 is free in plasma
169. What purpose does iodine serve in the protein bound iodine (PBI) methodology?
a. Holds organic iodine in the protein bound state
b. Acts as a catalyst in the ceric arsenite reaction
c. Compensates for loss of inorganic iodine in the washing stage
d. Keeps mercury from suppressing the color reaction
170. A T-4 measured as iodine is 1.8 mcg/dL. To what thyroxine value is if equivalent?
a. 1.10
c. 3.60
b. 2.75
d. 5.40
171. In T-4 by radioimmunoassay (RIA), the more hyroxine present in the patient serum
a. The more the amount of radioactive labelled thyroxine bound to antibody
b. The less the amountof radioactive labelled thyroxine bound to antibody
c. The less free radioactive labelled thyroxine to be absorbed
d. A and C
172.
173.
174. T-3 uptake is actually a measurement of
a. T-3
c. TBG
b. T-4
d. Free thyroxine
175. The following tests are good indications of hypothyroidism except
a. T-3 uptake
c. Total T-4 by radioimmunoassay (RlA)
b. Free t-4 by radioimmunoassay (RlA)
d. T-3 by radioimmunoassay (RlA)
176. Which of the steroids is identified as an 18 carbon compound that possesses a phenolic A ring?
a. Adrenocortical steroids
c. Estrogens
b. Androgens
d. Progesterone
177. The estrogen with the greatest concentration in urine is
a. Estriol
c. Estrone
b. Estradiol
d. Progesterone
178. The standard for 17 ketosteroid determinations is
a. Testosterone
c. Dehydroplandrosterone
b. Progesterone
d. Aldosterone
179. Which of the following is true regarding pregnanediol? It is
a. A C-19 compound
c. Manufactured in the adrenal medulla
b. The excretion product of progesterone
d. Altered in the adrenal medulla
180. Which of the following is true of steroids
a. They are excreted as hormones rather than metabolise
b. There is no different value for male and female
c. They are lipids
d. They contain the 21 carbon atoms of the cyclopentanoperthydrophenanthrene ring
181. What is the hormone that controls the reabsorption of sodium in the kidney?
a. Aldosterone
c. Estrogen
b. Alcohol dehydrogenase
d. Growth hormone
182. The kober reaction refers to the pink color resulting from the combination of hot concentrated sulphuric acid
and extracted
a. Estrogens
c. Cortiocosteroids
b. Androgens
d. 17-ketosteroids
183. The litrich extraction for estrogens involves the reagent
a. Phenythydrazone
c. M-dinitrobenzene
b. P-nitrophenol
d. Sodium bismuthate
184. The porler silber reactions employs phenylhdrazine to detect
a. Estrogens
c. 17-ketosteroids
b. Corticosteroids
d. Catecholamines
185. The Zimmerman determination of 17 ketosteroids based on reaction with
a. Acetic anhydride
c. Meta dinitrobenzene
b. Ehrich reagent
d. Potassium ferricyanide
186. For the measurement of 17 ketogenic steroids (17 hydroxycorticosteroids), prior treatment with what
reagent is required?
a. M dinitrobenzene
c. Sodium bismuthate
b. Hot sulphuric acid
d. Chloroform
187. Catecholamines are secreted by the
a. Kidney
c. Adrenal cortex
b. Pituitary
d. Adrenal medulla

A1 PASSERS REVIEW CENTER 320-2728 //

11

188. Which
a. 17
of ketosteroids
the following is a metabolite of epinephrine
c. Vanillymandelic acid
b. Follicle stimulating hormone
d. Thyroid stimulating hormone
189. A two hour pregnancy test can measure
a. Androgens
c. Estrogens
b. Chorionic gonadotropins, beta subunit
d. Progesterone
190. Which of these statements is true of an immunological tests for pregnancy?
a. Only first morning urine specimen is suitable
b. With the agglutination inhibition method, the presence of agglutination means a negative test
c. Drugs do not present a problem with interference
d. The hormone level in the urine detectable by this method steadily increases the first two weeks of
gestation
191. Which sub unit of human chorionic gonadotropin (hCG) will given a very specific radioimmunoassay test for
the measurement of hCG?
a. Alpha
c. Gamma
b. Beta
d. Della
192.
193. Which of the following specimens is/are used for pregnancy tests?
a. Cerebrospinal fluid
c. Urine
b. Blood
d. B and C
194. An ectopic pregnancy is one that occurs in the
a. Stomach
c. Fallopian tubes
b. Urethra
d. Ovaries
195. Human chorionic gonadotropin (hCG) is secreted by the
a. Thyrus gland
c. Ovary
b. Placenta
d. Ureter
196. The highest level of human chorionic gonadotropin (hCG) occurs in which trimester of pregnancy?
a. First trimester
c. Third trimester
b. Seconds trimester
d. NOTA
197. A glycoprotein consisting of two nonidentical noncovalently bound subunits is
a. Estrogen
c. Androgen
b. Progesterone
d. Human chorionic gonadotropin
198. Some commercial kits now available for pregnancy tests incorporate which of the following?
a. Nonoclonal antibodies
c. Heterophile antibodies
b. Forssman antibodies
d. Apsonins
199. To produce reliable results at which time should blood specimens for lipid studies be drawn?
a. 2-4 hours lasting
c. 8-10 hour fasting
b. 6-8 hour fasting
d. 12-15 hour fasting
200. The reagent for color development in the Liebemann-burchard reaction for cholesterol is
a. Sulphuric acid
c. Glacial acetic acid/sulphuric acid
b. Acetic anhydride/sulphuric acid
d. Ferric chloridetethanol
201. Which of the statements applies to Bloors reagent ? it:
a. Provides for color development in the cholesterol methods
b. Is composed of ethanol and either in a ratio of 31
c. Compensates for the otherwise unequal colon measurement resulting from free and esterified
cholesterol
d. Compensates for possible water contamination of the reagents
202. The purpose of the saponification step in some cholesterol methods is to
a. Break the protein lipid bond
b. Precipitate the protein
c. Convert esters to free cholesterol so that all measured cholesterol so that all measured cholesterol is
the same type
d. Remove the error of nonspecific chromogen interference
203. The purpose of the digitonin in cholesterol methodology is the
a. Precipitation of free cholesterol
c. Color developer
b. Precipitation of esterified cholesterol
d. Reagent for extraction
204. In which of these clinical conditions may blood cholesterol be increased?
a. Hypothyroidism
c. Lupus erythematosus
b. Malnutrition
d. Pancreatic cancer
205. Which of the following is the Friedewald formula by which low density lipoprotein (LDL) cholesterol can be
estimated? (TC=total cholesterol, TG=triglycerides, PL=phospholipids )
a. LDL cholesterol =TC (TG=triglycerids, PL=phospholipids)
b. LDL cholesterol = TC-(TG +PL)
c. LDL cholesterol = TC- HDL cholesterol

A1 PASSERS REVIEW CENTER 320-2728 //

12

d. LDL cholesterol =HDL-TC


206. When evaluating a coronary risk index for a patient you should know the total cholesterol and
a. Total lipids
c. Low density lipoprotein cholesterol
b. Triglycerides
d. Low density lipoprotein cholesterol
207. Most triglyceride procedures involve measurement of
a. Fatty acids
c. Lipoprotein
b. Glycerol
d. Phospholipids
208. Which of these lipids has the lowest density?
a. Alpha lipoprotein
c. Chylomicrons
b. Beta lipoprotein
d. Pre-beta lipoprotein
209. The more current kinetic methods for the quantilation of serum triglycerides employ enzymatic hydrolysis.
The enzymatic hydrolysis of triglycerides may be accomplished by what enzyme?
a. Lactate dehydrogenase
c. Aldolase
b. Lipase
d. Amylase
210.
211.
212. The migration sequence , from the origin of lipoproteins is
a. Chylomicrons, beta, pre-bella , alpha
c. Alpha, beta, pre-bella, chylomicrons
b. Chylomicrons, pre-bella , beta, alpha
d. Chylomicrons, alpha, pre-beta, beta
213. Diseases of which system are associated with elevated beta lipoprotein values?
a. Pulmonary
c. Gastrointestinal
b. Cardiovascular
d. Connective tissue
214. Which statement is true for the oral route of drug administration?
a. Drug action occurs most rapidly when taken orally
b. The oral route of administration is most reliable
c. The oral route compensates for drug solubility problems
d. Patient compliance is not always reliable
215. Which of the following therapeutic drugs is not an anticonvulsant?
a. Digoxin
c. Carbomazepine
b. Dilantin
d. Phenobarbital
216. The point of equilibrium between drug dosage intake and drug elimination is referred to as:
a. Peak concentration
c. Steady state
b. Trough concentration
d. Drug half life
217. A major disadvantage of drug detection by immunoassay is
a. Speed of performance
b. Stability of reagents
c. Technical difficulty of procedure
d. Simultaneous assay of multiple drugs in one specimen is not feasible
218. Lithium is measured by:
a. Gas chromatography
c. Visible spectrophotometry
b. Flame photometry
d. High pressure liquid chromatography
219. Tricyclic antidepressants include
a. Diazepam
c. Lithium
b. Dilantin
d. Doxepin
220. When a kidney stone is tested with sodium cyanide and sodium nitroprusside , the appearance of a
magenta color identifies what substance?
a. Carbonate
c. Calcium
b. Cystine
d. Uric acid
221. The sweat chloride test is useful in the diagnosis of
a. Dehydration
c. Hyperkalemia
b. Cystic fibrosis
d. Paroxysmal nocturnal hemoglobinuria
222. The use of radioisotope 51 Cr provides a direct measurement of which of the following?
a. Plasma volume
c. Hematocrit
b. Red cell mass
d. Total blood volume
223. The average blood volume for human beings, expressed in ml/kg body weight is
a. 40
c. 60
b. 50
d. 80
e.

f. CHEMISTRY /URINALYSIS
224. Cloudiness in a freshly voided urine could indicate the presence of
a. Protein
c. White blood cells
b. Sugar
d. Any of the above
225. In acute febrile disease, urine may acquire a reddish color due to
A1 PASSERS REVIEW CENTER 320-2728 //

13

a. Haemoglobin pigment
c. Urobilinogen
b. Porphyrin
d. Urcerythrin
226. A smoky red brown urine should be suspected of containing
a. Bilirubin
c. Porphyrin
b. Haemoglobin
d. Urobilinogen
227. A urine that turns brown-black upon standing may contain
a. Homogentisic acid
c. Porphyrin
b. Myoglobin
d. Red blood cells
228. When urine specimen is allowed to stand at room temperature , which one of the following Not occur?
a. Disappearance of formed elements
c. False positive glucose
b. Elevation in pH
d. False positive glucose
229. The first morning specimen of urine is often tested because this specimen
a. Contained most firmed elements
d. Will display the phenomenon of alkaline
b. Is fresh
tide
c. Is most likely contain protein
230.
231. What is the highest specific gravity to which the kidney is able to concentrate urina?
a. 1.022
c. 1.040
b. 1.030
d. 1.050
232. All of the following systems may be used to determine the specific gravity of urine EXCEPT
a. Spectrophotometer
c. T-S meter
b. Refractometer
d. Reagent test strip
233. At a temperature at 26C, a urine specific gravity reads 1.014. if temperature correction is employed, what
would the corrected specific gravity be?
a. 1.22
c. 1.040
b. 1.030
d. 1.050
234. A refractometer would be used in a urinalysis department to measure
a. Glucose
c. Specific gravity
b. Ketones
d. Bilirubin
235. If the plasma osmolality is normal (185mCsm/ kg water), the urine osmolality should be at least
a. The same osmolality that of the plasma
c. Three times that of the plasma
b. 500 mOsm/Kg
236. Which of these plasma substances is not normally filtered through the glomerulus in significant amount?
a. Creatinine
c. Protein
b. Glucose
d. Urea
237. Precipitation tests for urine protein DO NOT include which of the following?
a. Acid and heat
c. Sulfosalicytic acid
b. Dipstick
d. Trichloracetic acid
238. The normal excretion of small amounts of protein in urine is considered to be less than what daily amount?
a. 5 mg
c. 150 mg
b. 50 mg
d. 300 mg
239. All of the following apply to the dipstick tests for urine protein , based on the protein error of indicators ,
except
a. Most of the indicator is in nonionized form
b. The protein will combine with the nonionized form of the indicator
c. Protein combination with indicator changes the ratio ionized and nonionized forms
d. The color change on the stick is due to the pH change
240. Which of the following does not apply to urine protein estimation by the dipstick method?
a. Bence Jones protein interferes with measurement of albumin
b. Highly alkaline buffered urine can cause a false positive test
c. The principle of the test is the protein error of indicators
d. The test is more sensitive to albumin than the globulin
241. Which of the following is true of the detection of urinary glucose?
a. Any reducing substance can give a false positive reaction with the copper reduction method for
glucose
b. The copper reduction method is specific for glucose
c. Glucose cannot appear in the urine of the absence of an elevated plasma glucose
d. Ketonuria may produce a false negative dipstick test for glucose
242. The enzyme dipstick test for glucose in urine has a sensitivity of
a. 10 mg/dl
c. 100mg/dl
b. 50 mg/dl
d. 200 mg/dl
243. Which of the following may give false positive reaction on the glucose dipstick?
a. Galactose
b. Lactose

A1 PASSERS REVIEW CENTER 320-2728 //

14

c. Maltose
d. Oxidizing agent
244. For fructose detection , which of these reagents is employed?
a. Ehrlichs reagent
c. Fesorcinol and hydrochloric acid
b. Ferric chloride
d. Sodium nitroprusside
245. Which of the following is the most effective test for phenylketonuria (PKU)?
a. Direct measurement of blood level of phenylalanine
b. Guthrie tests on blood of a newborn
c. Measurement of liver enzymes
d. PHENISTIX test on a freshly voided urine
246. The urine dipstick tests for occult blood depend on the fact that
a. Haemoglobin can act as a peroxidise
b. Haemoglobin can release an enzyme from the strip
c. Haemoglobin in urine combines with oxygen on the strip to produce a blue color
d. Red blood cells in urine are lysed by peroxide from the strip
247.
248.
249. Which of the following statements is true regarding the test for heme pigment in urine?
a. Color in organic phase identifies urobilinogen
b. Color in the organic phase identifies porphobilinogen
c. Ehrlich reagent contains resorcinol and hydrochloride acid
d. Prior removal of billirubin is necessary
250. The addition of Fouchets reagent to the barium chloride precipitate of urine results in a green color in the
presence of
a. Billirubin
c. Protein
b. Ketones
d. Urobilinogen
251. The addis count is the semi quantitative estimate of the 12 hour excretion of which urinary constituent?
a. Bacteria and protein
c. Number and type of casts
b. Casts, red blood cells, and white blood
d. White blood cells and tubular epithelial
cells
cells
252. The most common use of polarized microscopy is to visualize
a. Crystals
c. White blood cells
b. Lipids (fats)
d. Casts
253. Which statements does not apply to white blood cellsin the urine sediments?
a. A few white blood cells may present in normal urine
b. An increase in white blood cells is associated with an inflammatory process
c. White blood cells in the urine must be accompanied by proteinuria
d. Unstained white blood cells in urine appear spherical and granular
254. With infections of the urinary system, white blood cells are frequently seen in the urine sediment. What type
of white blood cell is seen most frequently in the urine sediment?
a. Eosinophil
c. Monocyte
b. Lymphocyte
d. Neutrophil
255. Vaginal contamination may be responsible for the appearance of which element in the urine from a female
patient?
a. Glucose
c. Crystals
b. Ovaline cast
d. Squamous epithelial cells
256. Red blood cells in the urine may be confused with any of the following except
a. Calcium oxalate crystals
c. Oil droplets
b. Cholesterol crystals
d. Yeast cells
257. Which urinary crystals can assume the greatest variety of forms?
a. Crystine
c. Phosphate
b. Oxalate
d. Uric acid
258. Which urinary crystal is unlikely to appear in an acid urine?
a. Ammonium biurate
c. Cystine
b. Calcium oxalate
d. Cholesterol
259. Which crystal is unlikely to appear the alkaline urine?
a. Ammonium biurate
c. Cystine
b. Calcium oxalate
d. Uric acid
260. The Coffin lid crystals is that of
a. Ammonium biurate
c. Sodium urate
b. Calcium oxalate
d. Triple phosphate
261. The finding of six aced puffy crystals in the acid urine of an infant should arouse a suspicion of
a. Cystinosis
c. Oxaluria
b. Cystinuria
d. Tyrosinuria

A1 PASSERS REVIEW CENTER 320-2728 //

15

262. Yeast, oval fat bodies, and calcium oxalate crystals are common in persons who suffer from
a. Obesity
c. Diabetes mellitus
b. Chronic nephritis
d. Chronic crystitis
263. The matrix of urinary casts has been identified as which protein
a. Albumin
c. Glycoprotein
b. Globulin
d. Tam horsfall
264. Which of the following is true of casts in urine/
a. A high power objective is employed in a search for these elements
b. Some kind of stain is needed to make them visible
c. Their reactive index makes them easily visible
d. They tend to accumulate near the edges of the coverslip
265. A granular cast is thought to represent the decomposition of which of these casts?
a. Cellular
b. Fatty
c. Hyaline
d. Waxy
266. Which of these casts is most easily visualized in unstained urinary sediment?
a. Fatty
c. Red blood cell
b. Hyaline
d. White blood cell
267. Which may appear in urine in the absence of renal disease?
a. Fatty
c. Red blood cell
b. Hyaline
d. Waxy
268. The best single indicator of renal disease is
a. Glycosuria
c. Proteinuria
b. Polyuria
d. Pyuria
269. Glitter cell is term used to describe a specific type of
a. Fatty droplet
c. Oval fat body
b. Ketone body
d. Neutrophil
270. Which of the following not true about oval fat bodies?
a. Cholesterol esters break down into fatty globules in desquamated tubular epithelial cells
b. Their fat content is related to the level of lipids in the plasma
c. They are associated with degenerative tubular disease
d. They are associate with the appearance of the Maliese cross formation under polarization
271. Casts are formed primarily in which portion of the kidney?
a. Distal convulted tubule
c. Loop of henle
b. Glomerulus
d. Proximal convoluted tubules
272. Which of the following statements is not true ?
a. A clean catch urine is preferable in most instances to specimen collected by catherization
b. In a routine culture of urine 102 microorganisms /ml of urine in considered significant
c. The presence of bacteria in uncentrifuged specimen of urine collected under controlled condition is
associated with a significant colony count on culture
d. The presence of bacteriuria can be recognized by a positive result on the nitrate portion of a dipstick
casts
273. In glomerulonephritis , which of the following should be expected in the urine specimen?
a. Clumps of white blood cells and white blood cells casts
b. Fatty and granular casts
c. Red blood cells and red blood cell and fatty casts
d. Waxy and fatty casts
274. Which of the following is true statement?
a. A high specific gravity of urine may result in lysis of red blood cells
b. Hemoglobinuria is a more common finding than hematuria
c. A low pH of urine may result in the conversation of red blood cells to haemoglobin
d. True hemoglobinuria occurs after intravascular hemolysis

275.
276. CHEMISTRY
277. The mean for the series of laboratory results is 10.0 , the 95% confidence limits are 9.8 to 10.2. what is the
standard deviation?
a. 0.2 SD
b. 0.1 SD
c. 0.4SD
d. 0.5SD
278. If 0.5 ml packed red blood cells are diluted to a volume of 10ml with normal saline, what dilution is
achieved?
a. 1.5
c. 1.20
b. 1.10
d. 1.40

A1 PASSERS REVIEW CENTER 320-2728 //

16

279. 5 ml filtrate are required for a procedure. The final colored solution proves to be too high to read accurately
on the spectrophotometer . 1 ml filtrate and 4ml distilled water are substituted for the original filtrate and the
procedure is run as before. The graph reading is 46ml/dl. What is the actual amount of substance in the
patient serum?
a. 9.5mg/dl
c. 184.0 mg.dl
b. 11.5mg/dl
d. 230.0mg/dl
280. If 1.0 ml serum is diluted with 4 ml 25% benzoin acid, how much serum is represented in 2.0 ml of the
solution?
a. 0.20ml
c. 0.40ml
b. 0.25ml
d. 1.00ml
281. 90ml distilled water is added to 10ml of a 3% solution. What is the strength of resulting solution?
a. 0.10%
c. 0.30%
b. 0.27ml
d. 0.90%
282.
283. 0.1ml serum is added to 0.4ml saline in the first tube of serial dilution. Half (0.25) of the contents of the first
tube are transferred to the second tube, which already contains 0.25 ml saline. What is the serum dilution in
the second tube?
a. 1:20
c. 1:80
b. 1:50
d. 1:10
284. In a thrombocytopenia, twice the usual amount of blood is drawn into the red cell diluting pipet (blood to the
1.0 mark), which is then filled with Rees-Ecker diluents to the line at the top of the bulb. All nine squares of
the hemacytometer are employed to increase the accuracy of the count. In this nine squares , 27 platelets
are counted in cells/cu mm. What is platelet count for the patient?
a. 300
c. 6,000
b. 3,000
d. 27,000
285. Dilution of a spinal fluid is accomplished in a white cell diluting pipet, using diluents to the 1.0 mark and
spinal fluid to the 11 mark. All nine squares of a hemacytometer are counted , with tally of 45 white cells. In
cells /cu mm, what is the spinal fluid cell count.
a. 50
c. 500
b. 56
d. 1000
286. A 10% (W/V) solution of sodium chloride is needed. Of what does it consists?
a. 10mg NaCl/d
c. 10g NaCi/L
b. 10g NaCi/dl
d. 1 g NaCi/100ml
287. A solution of calcium chloride contains 3 grams per 100ml. What percent is this solution?
a. 1.5
c. 15.0
b. 3.0
d. 30.0
288. How much sodium citrate is required to make 500ml of a 3.2% solution for coagulation studies? (M.W of
sodium citrate =258)
a. 1.6g
c. 8.0g
b. 3.2g
d. 16.0g
289. A concentrated acid has a specific gravity of 1.5. what is the weight of 30 ml of this acid?
a. 4.5g
c. 45.0g
b. 20.0g
d. 200.0g
290. A concentrated sulphuric acid has a specific gravity of 1.84 and a percent purity of 98%. How many grams
of sulfuric acid are there in 200ml of concentration acid?
a. 111
c. 368
b. 361
d. 720
291. 150ml urine weights 154 grams . what is the specific gravity of the urine?
a. 1.013
c. 1.030
b. 1.027
d. 1.060
292. How much NaOH is needed to make 4 liters of a 1N solution? (M. W. Na=23; S=32; O=16; H=1)
a. 5.5
c. 55.0
b. 11.1
d. 111.0
293. How many milligrams of CaCl2 are needed to prepare 100 ml of a 1 x10 -3 M solution?
294. (M. W. Ca=40; Ci=35.5)
a. 5.5
c. 55.0
b. 11.1
d. 111.0
295. How much Na2SO4 is there in 400ml of a 4M solution? (M. W Na=23; S=32; O=16)
a. 114g
c. 190g
b. 150g
d. 227g
296. What is the normality of isotonic saline (0.85%)? (M. W. Na=23; CI=35.5)
a. 0.014N
c. 0.140N
b. 0.028N
d. 0.280N

A1 PASSERS REVIEW CENTER 320-2728 //

17

297. What is the normality of 4M NaOH? (M. W Na=23; O=16; H-1)


a. 0.2N
c. 2.N
b. 0.4N
d. 4.0N
298. A solution of HCI contains 84 grams per liter. What is the normality? (M. W. H=1; S=32;O=16)
a. 1.8N
c. 4.9N
b. 3.6N
d. 7.2N
299. What is the normality of a 3.6M sulfuric acid solution? (M. W. H=1: CI=35.5)
a. 1.8N
c. 4.9N
b. 3.6N
d. 7.2N
300. How many ml of 95 alcohol do you need to make 1 liter of a 70% solution?
a. 74ml
c. 737ml
b. 136ml
d. 1360ml
301.
302. Hno3 has a percent purity (assay) of 70 and specific gravity of 1.42. how many ml of concentrated acid
are needed to make 500ml of 0.22N acid?
a. 3.5ml
c. 6.9ml
b. 3.7ml
d. 7.0ml
303. What percent solution is 0.4N HCI? (At. W. H. =1; CI=35.5)
a. 1.42%
c. 14.20%
b. 1.46%
d. 14.60%
304. What amount of 10N NaOH must be used to prepare 1 liter of 1N NaOH?
a. 10ml
c. 100ml
b. 40ml
d. 200ml
305. A serum calcium level of 10mg/dl would be expressed as how many mEq/L? (At. W. Ca=40)
a. 4
c. 8
b. 5
d. 10
306. The microgram can be expressed as which of the following?
a. 1/10 gram
c. 1/1000 gram
b. 1/100 gram
d. 1/1000 mg
e.

f. MICROBIOLOGY AND PARASITOLOGY


1.
2.
3.
a.
b.
4.
a.
b.
5.
a.
b.
6.
7.
8.
a.
b.
9.
a.
b.
10.
a.
b.
11.
a.
b.

g.
Capsules can be used for a:
Complements fixation
c. Serotyping by swelling
Hemagglutination test
d. Precipitation test
To demonstrate capsules, grow the culture on media containing:
Antibiotics
c. Serum
Milk
d. B and C
The quelling test depends on the antigenic speficity of the:
Nucleus
c. Flagella
Cell wall
d. Capsule
Dark staining granules are called:
a. Spores
c. Cysts
b. Capsules
d. Metachromatic
Organisms that vary in size and shape are reffered to as:
a.Psychrophillic
c.Palisades
b.Thermophillic
d.Pleomorphic
In the bacterial growth cycle ,growth ceases because nutrients are exhausted or toxic metabolic products
have accumulated in the:
Stationary phase
c. Lag phase
Logarithmic phase
d. A and B
Which genera of bacteria form spores?
Corynebacterium
c. Clostridium
Bacillus
d. B and C
Bacteria that grow in the absence of atmospheric (free) oxygen and obtain oxygen from oxygen-containing
compounds are called:
Anaerobes
c. Capnophiles
Aerobes
d. Aerotolerants
The temperatures at which bactewria groe best in known as:
Thermophilic
c. Optimum
Psychrophilic
d. Mesophilic
A1 PASSERS REVIEW CENTER 320-2728 //
18

a. A
12.
Blood
Slimy
agar
colony
is too
onold
blood agar medium indicates which of the
c. following
Plate was
characteristics
incubated toomay
longbe present?
b. Organism has a capsule
d. Plate was incubated too high temperature
13. On blood agar paltes a small zone of alpha-hemoklysis surrounded by zone of beta-hemolysis after
refrigeration is known as:
a. Gamma-hemolysis
c. Alpha-hemolysis
b. Beta-hemolysis
d. Alpha-prime
14. When using fractional steritization the sequence is:
a. Heating (kills vegetative bacteria)
b. Incubation (spores germinate)
c. Heating (kills remaining vegetative bacteria)
d. AOTA
15. What is the most effective method of sterilization?
a. Autoclave
c. Boiling
b. Dry heat oven
d. Disinfection
16. Which of the following is used for quality control for the dry heat oven?
a. Bacillus Subtilis
c. Staphylococuss aureus
b. Bacillus stearothermophilus
d. Salmonelle lyuhi
17. Which of the following filters can give 100% sterility?
a. Seits
c. Millipore (0.5 um)
b. Millipore (0.22 um)
d. A and B
18. Media that cannot be heated can be steriLized by:
a. Filtration
c. Antiseptics
b. Chemical disinfectants
d. Cannot be steruilizede
19. Which of the the following is not a disinfectant?
a.Phenol
c.QUATS
b.70% alcohol
d.2% aquecus glutaraldehyde
20. Iodophors are composed of iodine and:
a. Phenolic compound
c. 70% alcohol
b. Detergent
d. 90-95% alcohol
21. Quaternary ammonium disinfectants are easily inactivated by:
a. Heat
c. Organic material
b. phearuginosa
d. Tap water
22. What is the term that describes a process or treatment that renders a medical device, instrument, or
environmental surface safe to handle?
a. Decontamination
c. Disinfectant
b. Artyispesis
d. Sterile
23. What do gram positive bacteria stain?
a. Purple
c. Red
b. Green
d. Maroon
24. The mordant used in the Zieht-Neelsen acid-fast-stain is:
a. Grams iodine
c. Heat
b. Auramine
d. Acid-alcohol
25. How is smear for an acid-fast stain fixed?
a. Methyl Alcohol
c. Tergitol no.7
b. Acid alcohol
d. Slide warmer at 65%C for 2 hours
26. Acid-fast bacteria appear w3hat color microscopically?
a. Red
c. Green
b. Blue
d. Purple
27. Acid-fast organisms in tissue are best stained by whic of the following methods?
a. Hiss
c. Ziehi-Neelsen
b. Kinyoun
d. Negative stain
28. In the Ziohi-Neelsen staining method, the decolorizer is:
a. Heated saline
c. Acetone + alcohol
b. Acetone
d. Alcohol + HCI
29. An example of an negative stain is:
a. Auramine-rhodamine stain is
c. India Ink stain
b. Gyram stain
d. Methylene blue stain
30. Which of the following is NOT gram-negative?
a. Peptococcus
c. Branhamella
b. Salmonella
d. Aeromonas
31. Which of the following is NOT an anaerobe?
a. Veillonella
c. Antinomyces
b. Fusobacterium
d. Campylobacter

A1 PASSERS REVIEW CENTER 320-2728 //

19

32.
a.
b.
c.
d.
33.

If a patient is taking antimicrobials, which of the following can neutralize the antimicrobials?
Tryptic soyn broth
Antihistamine
Thiol broth
Brain-heart infusion broth
From a bronchial washing, organisms are seen on a Gram stain, but no growth occurs aerobically and
anaerobically. This is most likely due to :
a. Anaerobes
b. Inhibition by antibiotics therapy
c. Uncultivable bacteria e. G., rickettsiae, mycoplasma, M leprae
d. The specimen being incubated instead of refrigerated
34. Which of the following should be used to remove antimicrobials before culturing?
a. Antimicrobial Removal device
c. Septi-Check
b. Isolator
d. NOTA
35. Sodium polyanethol sulfonate may be used as an anticoagulant in blood cultures because it
a. Removes some antimicrobials
b. Prevents phagocytosis
c. Neuthalizes the bactericidal effect of human serum
d. B and C
36. What is the most abundant normal flora in troat cultures?
a. Micrococcus
c. Escherichia coli
b. Alpha-hemolytic Streptococcus
d. Legionella pneumophila
37. What is the most common pathogen is throat cultures?
a. Group A streptococcus
c. Staphylococcus edidermidis
b. Alpha-hemolytic
d. Coybobacterium diphtheriae
38. Why most blood agar plates for troat cultures be incubated aerobically and anaerobically when betahemolytic streptococci are suspected?
a. Some may producebeta-hemolysis under aerobic conditions
b. Streptococci only grow under aerobic conditions
c. Streptococci only grow under anaerobic conditions
d. Neither A or B
39. Nasopharyngeal swabs are recommended for the detection of carriers such as:
a. Hemophilus influenzae pertussis
b. Neisseria, vibrio, bordetella pertussis
c. Hemophilus influenzae, Erysipelothrix, Vibrio
d. Erysipelothrix, Neisseria,LIsteria
40. Todd-Hewitt broth is recommended for:
a. Determination of mycobacterial growth rate
b. Primary culture for anaerobes
c. Stool enrichment for salmonella but not for shigella
d. Cultuure of beta-hemolytic streptococci for flouresecnce microcopy
41. Phenylethyl alcohol is used in media to:
a. Inhibit gram-positive bacteria
c. Stimulate gram-positive bacteria
b. Inhibit gram-negative bacteria
d. Stimulate gram-negative bacteria
42. Why is a first morning urine specimen preffered for urine cultures?
a. Specimen is diluted
c. Specimen is more concentrated
b. Bacteria are all motile
d. A and B
43. What indicates vaginal or uu8rethral contamination of urine?
a.Many squqmous epithelial cel;ls
c.1-2 RBCs/hpf
b.Few white blood cells
d.Morphous urates
44. In the pour-plate method for colony counts, how does the amount of agar added affect the dilution.
a. Dilution is increased
c. Dilution is increased 20%
b. Dilution is decreased
d. Dilution is not affected
45. Smears of cerebrospinal flyuid are prefared from:
a. Cerebrospinal fluid sediment
c. Heated cerebrospinal;
b. Uncentrifuged cerebrospinal
d. Frozen cerebrospinal fluid
46. Smears of cerebrospinal fluid are u8suall stained with;
a. ACID-FAST=STAIN
c. India ink stain
b. Gram stain
d. B and C
47. Which of the following tests for bacterial antigen on cultures isolated from cerebrospinal fluid is more
sensitive and faster?
a. Coagglutination tests
c. Counterimmunoelectrophoresis test
b. Latex agglutination tests
d. Coagulase test

A1 PASSERS REVIEW CENTER 320-2728 //

20

a.
48.
b.
49.
a.
b.
50.
51.
a.
b.
52.
a.
b.
53.
a.
b.
54.
a.
b.
55.
a.
b.
56.
a.
b.
57.
a.
b.
58.
a.
b.
59.
a.
b.
60.
a.
b.
61.
a.
b.
62.
a.
b.
c.
d.
63.
a.
b.
64.

72.
a.
b.
c.
d.
73.
a.

Coagglutination protein
Staphylococcal
test A coasted with antiserum is used inc.which
Counterimmunoelectrophoresis
of the following serological tests on
cerebrospinal
Latex
agglutination
fluid?test
d. ELISA
Which of the following is not cause of a venereal disease?
Nyelsseria gonorrthoeae
c. Staphylococcuhs aureus
Troponema pallitum
d. Trichomonas vaginalis
Using sheep blood foer blood agar pltes eliminates beta-hemolytics:
Staphylococcus
c. Streptococcus
Neisseria
d. Hemophllus
A positive tube coagullase test is observed for:
Bubbling
c. Liquefaction
Agglutination
d. Clotting of plasma
What type of plasma is used for the tube coagulase test?
Rabbit
c. Sheep
Human
d. Horse
Some citrate-positive organisms cause a falkse-positive tube coagulase test because the organism uses
the citrate
And release calcium
c. As a bets-lactamase
As a source of calcium
d. And release oxygen
A nonhemolytic, catalaswe-positive, coagulase-nehgative, gram-positive coccus is most likely to be:
Staphylococcus aureus
c. Alpha-hemolytic streptococci
Staphylococcus ediermidis
d. Steptococcus faecalis
The preffered method of differentiating S. Aureus from S. Epirmidis is:
Catalase test
c. Dnase/mannitol fermentation
Coagulase test
d. Phage typing
Which of the following staphylococci is the cause of urinary tract infection in young females?
S. Epidermidis
c. S. Aureus
S. Saprophyticus
d. S. Nominis
Staphylococcus aureous can be isolated from stool cultures by the use of:
Potassium tellurite medium
c. Medium with 7.5% salt concentration
Macconkey nagar
d. Lowenstein-Jensen medium
The most common cause of bacterial food poisoning in the united state is:
Clostridium botullinum
c. Clostridium perfringens
Staphylococcus aureus
d. Shigella dysenteriae type 1
Which of the following gram-positive cocci ferment glucose?
Staphylococci
c. A and B
Micrococci
d. Neither A nor B
Which hemolysin,produced by beta-hemolytic streptococci, is oxygen stable and nonantigenic?
Streptolysin A
c. Streptolysin G
Streptolysin S
d. Streptolysin O
Cultures for beta-hermolytic streptococci must include:
Incubation at 25 C to ensure growth of all strains
Anaerobic incubation to detect hemolysis in all strains
Incubationwiythy carbon dioxide
Media with glucose to show typical hemolysis
Which of the following is destroyed by oxygen?
Botulism toxin
c. Catalase
Streplolysin O
d. Coagulase
Match the following organisms with the biochemical test that presumptively identifies them:
65. Tests may be used more than one time:
66. 1. Group A, beta-hemolytic streptococci
a. hippurate hydrolysis positive
67. 2. Streptococcuspneumoniae
b. CAMP test positive
68. 3. alpha-hemolytic streptococci
c. bacitracin susceptible
69. 4. Group b, beta-hemolytic streptococci
d. bile-esculin positive
70. 5. Enterococcus
e. Optochin disk susceptible
71.
f. optochin disk resistant
Whenever a biochemical test is substituted for a serological test it is reported:
By the genus and species name
As presumptive with the name of the biochemical test
By its common name
NOTA
CAMP is a factor produced by the group B beta-hemolytic streptococci that:
Reduce the zone of lysis formed by the streptococci

A1 PASSERS REVIEW CENTER 320-2728 //

21

b.
c.
d.
74.
a.
b.
c.
d.

Hydrolysis the B factor


Causes a change in color
Enlarge the zone of lysis formed by staphylococcal beta-hemolysis
A positive quelling test is:
Oxidation but not fermentation
Virtual proof that the organism is a pathogen
Visible only by fluorescent light
From capsular swelling due to an antigen-antibody reaction
75.
76. Which of the following serological tests is used for a confirmatory test of streptococci?
a. Phadebac test
c. Lancefield precipitin test
b. Fluorescent antibody test
d. NOTA
77. When performing a bacitracin differentiation test for group A beta-hemolytic streptococci, use disks that
have how many units?
a. 10.00 units
c. 0.02-0.04
b. 500 units
d. 1.00-2.00 units
78. Match the disease in the first column with the organism in the second column. Letters in the second column
may be used more than once:
79.
1. Scarlet fever
a. group B streptococci
80.
2. Major pathogen of the newborn
b. group A streptococci
81.
3. Step throat
c. alpha-hemolytic streptococc
82.
4. Subacute bacteria endocarditis
83. What do optochin and bacitracin tests have in common?
a. Autolysis is a negative result
b. Growth inhibition is a positive result
c. They should not be done on blood agar
d. Either one will differentiate enterococci from viridens streptococci
84. The bilo-esculin test is used to differentiate:
a. Enterococci from other group D
c. Groupm D streptococci from other strep
streptococci
d. Group A from b streptococci
b. Streptococci faecalli from listeria
85. Hippuratae hydrolysis is used to differentiate:
a. Group A and Group b streptococci
c. Listeria from streptococci
b. Group B streptococci from enterococci
d. Pneumococci from other viridians
86. A bacitracin-resistant, hippurate-hydrolysis positive bile-esculin positive, beta-hemolytic streptococcus that
growns in 6.5 % NaCI is probably a/an:
a. Pnemococcus
c. Group D strep
b. Enterococcus
d. Diphtheroid
87. Pneumococci that arwe resistant to penicillkin should be tested for:
a. Producti0n of beta-laclamase
c. Growth in NaCl
b. Bile solubility
d. Resistant to bacitracin
88. Gram-negative, coffe bean-shaped diplococci with adjacent sides flattened is descriptive of which of the
following?
a. Neisseria
c. Listeria
b. Staphylocopccus
d. Chlamydia
89. The diagnostic of gonerrhoesae in males can be made from:
a. Positive urethral smear
c. History AOTA
b. Symptoms
90. Whioch of the following specimens may be appropriate for culturing neisseria gonorrhoeae?
a. Eyes
c. Oral cavity
b. Rectum
d. AOTA
91. What is the medium of choice for culturing gonococci and meningococci?
a. Lowentien-jenser
c. Sheeop blood agar
b. Modified Thayer-Martin
d. Potassium tellurite
92. Which of the following differentiates Thayer-Martin medium from modified Thayer-Martin medium?
a. Nystatin
c. Colstin
b. Vancomycin
d. Trimethoprim lactate
93. Thayer-Martin medium is basically a/an:
a. Blood agar
c. Enriched chocolate agar
b. Enriched Macconkey agar
d. Potassium tellunte agar
94. What color is a positive oxidase-positive?
a. Moraxella
c. Neisseria
b. Aeromonas
d. AOTA

A1 PASSERS REVIEW CENTER 320-2728 //

22

95. Whena.performing
Moraxella genera are oxidase-positive?
c. Neisseria
b. Aeromonas
d. AOTA
96. When performing the oxidase test, which of the following techniques can be used?
a. Put a drop of reagent on the colony
b. Rub colony on a filter paper strip and add a drop of reagent
c. Rub the colony on a piece of filter paper containing the reagent
d. AOTA
97.
98. What do PPNG gonococci produce?
a. Penicillin-producing goncciocci
c. A and B
b. Penicillinase-producing gonccocci
d. Neither A nor B
99. Which of the following are methods for testing for the producyion of beta-factamase?
a. Chromogenic cephalosporin method
c. Iodometric method
b. Acidometric method
d. AOTA
100. Which of the following indicates a positive reaction for the beta-lactamase chromogenic cephalosporin
method?
a. Production of acid
c. Reduction of nitrates
b. Color change
d. Decolorization of starch-iodine mixture
101. An oxidase positive,gram-negative, coccus from a troat culture might be:
a. Staphylococcus
c. Fusobacterium
b. An EC
d. Neisseria
102. A fastidious, oxidase-positive ,gram negative coccus from a rectal swab might be:
a. Neisseria gonorrhoeae
c. Listeria
b. Pseudomonas aeruginosa
d. Branhamella catarrhalis
103. Neisseria meningitidis degrades which of the following sugars?
a. Glucose
c. Lactose
b. Maltose
d. A and B
104. Which of the following tests xcan give a prasumtive identification of branhamella catarrhalis if the isolate is
a gram-negative diplccoccus that is oxidase-positive and isolated from middle-ear fluid?
a. Nitrate test
c. Inudole test
b. Urease test
d. Beta-lactamase test
105. Why should beta-lactamase tests be performed with growth from primary isolation media?
a. Plasmid coding for the enzyme may be lost on subculturing
b. May react wiyh chemicals in the medium
c. Because of the absorbance of oxygen from the medium
d. NOTA
106. What is the purpose of potassium tellurite in tellurite medium?
a. Inhibits the normal flora
c. Enriches pleomorphism
b. Enriches the medium
d. Enhances granule production
107. Potassium Tellurite medium produces what color colories of corynebacterium diphtheriae?
a. Green
c. Red
b. Blue
d. Gray-Black
108. When culturing Corynebacterium diphtheriae, Which of the following media will enhance pleomorphism and
granule production?
a. Loeffter serum agar
c. A and B
b. Palcoagulated egg medium
d. Neither A nor B
109. what is the morphology of corynecbacterium diphthenae?
a. Gram-negative, motile cocci
c. Gram-positive, nonmotile rods
b. Gram-positive, motile cocci
d. Gram-negative, nonmotile rods
110. The term palisadingpicket fence letter describe the common arrangnment of cell of:
a. Corynebacterium
b. Mycobacterium tuberculosis but not other mycobacteria
c. Actinomyces but not nocardia
d. Corynebacterium diphtheiae but not other cotrynebacteria
111. Babes-Ernst granules are characteristic of:
a. Bacillus anthracis
c. Mycobacterium tuberculosis
b. Listeria momocytogenes
d. Corynebacterium diphtheria
112. Which of the following are considered diphtheroids?
a. Corynebacterium xerosis
c. Corynebacterium JK
b. Corynebacterium diphtheriae
d. A and C
113. Which of the following dipheroids is found in the normal throat?
a. Propionibacterium xerosis
c. Corynebacterium xerosis
b. Corynebacterium diphtheriae
d. Rhodococcus equipment

A1 PASSERS REVIEW CENTER 320-2728 //

23

114. The elect test is for the detection of:


a. Corynebacterium diphtheriae toxin, in vivo
b. Corynebacterium diphtheriaetoxin, in vivo
c. Bacillus anthracis toxin, in vivo
d. Clostridium botulinum toxin, in vivo
115. The morphological cycle of rhodococcus equi from coccoid to rod form tales:
a. 5 hours
c. 24 hours
b. 12 hours
d. 48 hours
116. Which of the following is a gram-positive to gram-variables coccobacillus?
a. Stapyhylococcus epidermidis
c. Listeria monocytogenes
b. Corynebacterium diphtheriae
d. Haemophiluis influence
117. Listeria monocytoganes produces a positive in which of the following biochemical tests?
a. Coagulase test
c. Catalase
b. Lactose dfermentation test
d. A and C
118. Which of the following is a vilurence test for listeria monocytogenes?
a. Elek test
c. ELISA test
b. Anton test
d. Feeley-Gorman
119. Refrigiration of the specimen for several months may enhance isolation of:
a. Psychrophillic blood bank
c. Neisteria meningitidis
contaminmants
d. Actinomyces
b. Listeria monocytogenes
120. Which opf the following will differentiate listeria monocytogenes from the corynebacteria?
a. Nonmotile and salicine-negative
c. Nitrate-positive and sucrose-negative
b. Motile and salicin-positive
d. Mobile and urease negative
121. Spore-forming cells in bamboo pole arrangement of bacillus anthracis can be fopund in:
a. Specimen from patient Guinea pigs
c. NOTA
b. Cultures
122. Which aerobic, gram-positive,sporulating rods can cause food poisoning?
a. Bacillus subtilis
c. Mycobacterium phlei
b. Bacillus cerrrreus
d. Clostridium ramosum
123. Very large gram-negative rods with spore are in a nonturbid throglycollate broth culture of spinal fluid, but
there is no growth anaerobically or aerobically. Thios is most likely due to:
a. Too small an amount of inoculum that caused the organisms to die
b. Exchange of genetic material between bacillus sp and Escherichia coli
c. Bacillus sp. In the dehydrated medium are killed and made gram-negative by autoclaving the medium
d. A very high spinal fluid glucose allows the Escherichia coli to multify in broth but not on solid medium
124. Match the following dyes or reagents used in the ziehi-neelsen acid fast training procedure in the second
column with their purpose in the first column:
125. 1. Mordant
a. acid (3% HCI)alcohol
126. 2. Primary dye
b. dilute methylene blue
127. 3. Counterstain
c. carbolfuchsin
128. 4. Decolorizer
d. heat to speed staining
129. Non-acid-fast bacilli what color?
a. Green
c. Red
b. Blue
d. Orange
130. Acid fast stain that does not use heat as the colonies of mycobacteria can be examined microscopically?
a. Methylene blue
c. Gentian violet
b. Malachite green
d. Carbolfuchsin
131. An acid-fast stain that does not use heat as the mordant is which of the following?
a. Truant flourochrome stain
c. Kinyoun stain
b. Ponder stain
d. Gomori stain
132. Which of the following media is clear so that the colonies of mycobactweria can be examined
microscopically?
a. Mueller-hinton
c. Blood agar
b. Lowenstein-Jensen
d. Middlebrook 7H11
133. What color is a negative enzymatic hydrolysis of tween 80 test?
a. Red
c. Green
b. Amber
d. Blue
134. Mycobacterial cultures should be incubated:
a. In 5% carbon dioxide
c. At25C
b. In increased humidity
d. Under anaerobic conditions
135. A slow-growing,unbranched, acid-fast rod that is nitrate reduction-negative and nlacin-negative is most
likely to be:

A1 PASSERS REVIEW CENTER 320-2728 //

24

a. Mycobacterium tuberculosis
c. Mycobacterium kansasii
b. Mycobacterium bovis
d. Actinomtyces israelil
136. Mycobacterium tuberculosis is best differentiated from Mycobacterium bovis by:
a. Growth rate
c. Hydrolysis of tween 80
b. Niacin and nitrate reduction tests
d. Catalase test at 68 c
137.
138.
139.
140. Which of the following are slow growers?
a. Mycobacterium kansasil
c. Mycobacterium tuberculosis
b. Mycobacterium avium-intracellulare
d. AOTA
complex
141. Which differential test for identifying the species of Mycobacteriyum splits phenolphthalein from
iripotassium phenolphthalein-sulfate within 2-3 days?
a. Catalase test
c. Arysulfatase test
b. Tween 80 hydrolysis
d. Nitrate reduction test
142. Which of the following is used mfor the identification of mycobacteria?
a. Blochemical reactions
c. Growth rate
b. Pigment production
d. AOTA
143. One tube of Lowenstein-Jensen medium is incubated covered with foil and one tube is incubated
uncovered. when is the covered tube observed?
a. When growth appears on the
c. After 2 weeks of incubation
uncovered tube
d. At any time
b. After 1 week of incubation
144. Which of the following belong to group IV (rapid growers)?
a. Mycobacterium gordonae
c. Mycobacterium phlei
b. Mycobacterium smegamatis
d. B and C
145. What are lepra cells?
a. Neutrophils caontaining gram-negative cocci
b. Basophils containing gram-positive diplococci
c. Macrophages containing acid fast bacilli
d. Red blood cells containing acid-fast bacilli
146. Acid fast rods in a specimen from nasal mucosa:
a. Are diagnostic for leprosy
c. Are called lepra cells
b. Must be cultured on hansens agar for
d. Are not a diagnostic point
definitive identification
147. The fite-faraco acid-fast stain is different from other acid-fast stains because it uses:
a. Hematoxylin rather than methylene blue as a counterstain
b. Carbon gentian violet rather than safranin as a counterstain
c. Malachite green rather than hematoxylin as a counterstain
d. India ink with no counterstain
148. The specimen is usually used for the diagnostics of Hansens disease?
a. Tissue juice
c. Sputum
b. Blood
d. Urine
149. Lumpy jaw is caused by:
a. Nocadia brasilliensis
c. Antinomyces israella
b. Trichophyton rubrum
d. Microsporum canis
150. Which of the following is enaerobic?
a. Actinomyces
c. Mycobacterium
b. Nocardia
d. Bordetella
151. Which of the following is urease-positive?
a. Salmonella
c. Nocardia
b. Shigella
d. Chlamydia
152. NOcardia will grow on any media that does contain:
a. Antibiotics
c. Peptone
b. Blood serum
d. Agar
153. Match the organism in column 1 with the disease in column2:
154. 1. H. Influenzae
a. normal flora in respiratory tract
155. 2. H. Ducreyl
b. pinkeye
156. 3. H. Aegyptius
c. Ulcerative chancroid
157. 4. H. haemolyticus
d.meningitis
158. Which medium is preffered for the culture of most haemophilus?
a. Nutient agar
b. Tryptic soy agar

A1 PASSERS REVIEW CENTER 320-2728 //

25

c. Bordet-gengou plates
d. Enriched cholocate agar
159. A fastidious, small to filamentous , gram-negative rod from a nasopharyngeal swab is most likely to be:
a. Actinomyces israell
c. Branhamella catarrhalis
b. Neisseria meningtidis
d. Hemophilus influenzae
160. Cultures of staphylococcus supplies which of the following for cultures of haemophilus?
a. Factor I
c. X factor
b. Factor III
d. V wfactor
161.
162. What type of blood is used in blood agar plates for better production of beta-hemolysis of haemophilus?
a. Horse
c. Human
b. Sheep
d. Rabbit
163. Which of the following organisms cause whooping cough?
a. Haemophilus haemolyticus
c. Bordetelia pertussis
b. Legionella pneumophila
d. Streptococcus pneumoniae
164. Which of the following species of Bordetella are urease and oxidase-positive?
a. Parapertussis
c. Pertusis
b. Bronchiseptica
d. NOTA
165. The preffered medium for the isolation of bordetella pertussis:
a. Bordet-gengou medium
b. Chiocolaye agar
c. Charcoal-caphalexin agar
d. Clotted rabbit blood
140. A brucella isolate that does not produce H2S1, does not require CO2 and not inhibited by thionin of basic
fushin is probably which pecies of brucella?
a. Brucella canis
c. Brucella suis, biotype 1
b. Brucella abortus,biotypes 1
d. Brucella melitensis, biotype 1
141. A gram-negative, fastidious bacillus that has been isolated from air-conditioning tower is:
a. Proteus vulgaris
c. Edwardsiella tarda
b. Brucella abortus
d. Legionella pneumophila
142. A gram-negative, fastidious bacillus that is difficult to stain by the gram stain is:
a. Pruteus vulgais
c. Edwardsiella tarda
b. Brucells abortus
d. Legionella pneumophila
143. A relatively slow growing anad fastidious, gramnegative rod that produces a characteristics brown pigment
on Feeley-gorman agar is:
a. Hemophlus influence
c. Bordetella pertussis
b. Legionella pneumophila
d. Brucella melitensis
144. At the present time, which of the following is most sensitive for the diagnostic of infections with legionella
pneumophila?
a. Demonstration of a significant rise in antibody titer
b. Isolation of organism by culture
c. Direct visualatiopn of the organism in htissue
d. Detection of soluble microbial products nin body fluids
145. The best medium for the isolation of legionella is:
a. Iron-cystine agar
c. V agar
b. Campy-thio agar
d. Buffered charcoal yeast extract agar
146. What does the campy gas mexture consist of?
a. 10% co2
c. 85% NITROGEN
b. 5% o2
d. AOTA
147. Rice water stools often ncontain:
a. A pur culture nof vibrio cholera
c. Toxigenic staphylococcus aureus
b. Toxigenic clostridium botulinum
148. A curved, gram-negative rod that is nonfermenutive, nitrate-positive and microaerophilic is:
a. Flavobacterium
c. Campylobacter jojuni
b. Spinilum mi9nus
d. Leptospira
149. Campylobacter is a an:
a. Obligate microaerophuile
c. Facultative anaerobe
b. Obligate aerobe
d. Obligate anerobe
150. A gram-negative, slender, curved rod with s single polr fiagellum is the cause of gastroenteritis and best
isolated jon:
a. Sheep blood agar
c. Campy blood agar
b. Human blood tween agar
d. XLD agar
151. Clue cells may be found in infection with:
a. Campylobacter coli
b. Gardnerella vaginalis

A1 PASSERS REVIEW CENTER 320-2728 //

26

c. Legionella pneumophila
d. Brucella canis
152. Mycoplasmas are not true bacteriam because they:
a. Have 4 nuclei
c. Are anaerobic
b. Grow rapidly
d. Have no cell wall
153. Which of the following microorganisms form colonies that are embedded in the agar giving a fried egg
appearance?
a. Mycoplasmapneumoniae
c. Clostridium difficile
b. Mycobactyerium kansasii
d. Ureaplasma urealyticum
154. What is the most method for the identification of mycoplasma species?
a. Nitrate reduction test
c. Radial immune diffusion test
b. Inhibition of growth by specific antisera
d. Production of acation
155. What is the purpose of enrichment fluid media when culturing enterobacteriate?
a. Extends lagf phase of normal flora
c. Extends lag phase of pathogens
b. Decrease lag phase of pathogens
d. A and B
156. Match each item in the first column with one or more of the media in the second column?
157. 1.____ eosin Y and methylene blue
a. MacConkey medium
158. 2.____ lactose and sucrose
b. Holt-Harris, EMB medium
159. 3.____ lactose only
c. Levines EMB agar
160. 4.____ Crystal violet
161. 5.____ Bile salts
162. 6.____ Neutral red
163. 7. ____Lactose
164. 8. ____Black or purple colonies
165. 9.____ brick red colonies
166. 10.___ calories colonies
167. Nonfermenters produce what reaction in triple sugar iron agar (TSI)?
a. Alkaline slant, Alkaline or neutral butt
c. Alkaline saint, Alkaline butt
b. Acid slant, acid butt
d. Alkaline slant, acid butt
168. What is the hydrogen sulfide indivcator in triple sugar iron agar?
a. Tryptophan
c. Ferric ammonium citrate
b. Sodium Thiosulfate
d. Alpha-naphthol
169. IMViC is a series of which of the following teats?
a. Indole, methyl red, Voges-Proskauer, citrate
b. Indole,motility, voges-proskauer, citrate
c. Inositol,mannitol, voges-proskaue, citrate
d. Isositol, methyl red, Voges-Proskauer, carbohydrate
170. What is the indicator in the methyl red test?
a. Bromthymol blue
c. Bromcresol purple
b. Phenol red
d. Methyl red
171. Most enter bacteria cease give what type methyl red and voges-Proskauer reactions?
a. Opposite
c. Identical
b. Similar
d. NOTA
172. What is the indicator in simmons citrate agar?
a. Phenol red
c. Bromoreasol purple
b. Bromthmol blue
d. Neutral red
173. Match each item in the first column with one or more of the media in the second column:
174. a. salmoneilla-shigella (SS)
175. b. Bismuth-Sulfite (BS)
176. c. Xylose-lysine-deoxycholate
177. d. hoktoen Entric agar ebrilliant green agar
1. ___ glucose (dextrose)
2. ___ Lactose only
3. ___ Lactose, sucrose, salicin
4. ___ ferrous sulfate
5. ___ phenol red
6. ___ brilliant green onlyn; inhibitor for gram-positive organisms and coliforms:
7. ___ bile salts and brilliant green
8. ___ ferric(ammonium )citrate+sodium thiosulfate
9. ___ xylose, lactose, sucrose
10. ___ Bromthymol blue and acid tuchsin
11. ___ Neutral red
12. ___ Red colonies
13. ___ Colorless colonies

A1 PASSERS REVIEW CENTER 320-2728 //

27

14.
15.
16.
17.
18.
19.
20.

___ Red colonies with black centers


___ Black colonies surrounded by black zone with metallic sheen
___ Green colonies
___ Blue green colonies with black centers
___ Orange-salmon pink colonies
___ Citrate to inhibit coliforms and proteins
___ Whitish snowflake colony surrounded by brilliant red agar
178.
179. Which of the following amino acid are used in the test to decarboxylase?
a. Cadaverine, putrescine, ornithine
c. Lysine, arginine, ornithine
b. Ornithine, arginine, cadaverine
d. Arginine, putrescine, cadaverine
180. Testing 3 amino acid would require 4 tubes, WHY?
a. 2btubes are needed for testing lysine
c. 1 tube is neede for a control
b. 2 tubes are neede for testing arginine
d. Only 3 tubes are needed
181. Which of the following is an enzyme that removes the amino group (NH2) from an amino acid?
a. Decardoxylase
c. Gelatine
b. Urease
d. Deaminase
182. Organisms that produce urease do what to the medium?
a. Produce phenylphruvic acid
c. Hyydrolyze area to NH$ OH
b. Liquefy the medium
d. Produce hydrogen sulfide
183. Hydrogen sulfide production, other than in triple sugar iron agar require:
a. An organic source of sulfure
c. A butter
b. A source of metal
d. A and B
184. What color indicates a negative result in the molonate utilization test?
a. Green
c. Blue
b. Yellow
d. A and B
185. Gelatinase breaks down gelatine to:
a. Sodium malonate
c. Hydrogen sulfide
b. Amino acids
d. Aceton
186. What indicates a positive dnase result after 0.1N HCI is added to the plate?
a. Gels on reprigeration
c. Agar becomes cloudy
b. Groen slant
d. Agar clears around the colony
187. The DNase test is based on:
a. Color
b. Terbidity indicating growth in a medium containing DNA
c. Destruction of sugars in the culture medium
d. Product detection of DNA degradation by DNase
188. If the negative nitrate reduction test doest not change color after the powdered zinc is added, how is it
reported?
a. Positive
c. Negative
b. Doubtful
d. Do not report
189. Saimonella, shigella,Escherichia coll, and serratia are all:
a. Dextrose-negative
b. Lactose-negative
c. Common hospital-aquired uninary tract infectious agents
d. Members of Enterobactrianceae
190. What is the purpose of the ONPG test?
a. Determines that amount of glucose
c. Determines the amount of sucrose
b. Determines the Fermentation of
d. Detects slow lactose fermenters
sucrose
191. An organisms isolated from a urine culture has the following reactions: gram-negative rod: TSI A/AG +G;
IMVIC + motile: urease-negative: Which of the following organisms is it?
a. Enter bacteria cloacae
c. Klebsiella pneumoniae
b. Serratia marcescens
d. Escherichia coli
192. An organisms isolatedfrom a blood culture has the following reactions: Gram-negative rod; TSIA A/A+G;
IMVIC..++: motile;lysine decarboxylase- negative; no capsule;;Which of the following organisms is it?
a. Enter bacteria cloacae
c. Klebslella pneumoniae
b. Escherichia coli
d. Serratia marcescens
193. An organisms isolated from a blood culture has the following: reactions:Gram-negative rod.,urease and
phenylalanine doeminase positive;;; TSI K/A +G,-H2S: lysine decarboxylase-negative and opmithine
decarboxylase-positive;:.Which of the following organisms is it?
a. Proteus mirabilis
c. Morganella morgani
b. Providencia alcalifaciens
d. Proteus vulgaris

A1 PASSERS REVIEW CENTER 320-2728 //

28

194. An organisms isolated from a stool culture has the following reactions: gram negative rod; uresaes and
phenylalanine demaninase negative:TSI K/A-G_H2S; lysine decarboxylase negative ;nonmotile;;Which of
the organisms is it?
a. Salmonella subgroup 3 (Arizona)
c. Shigella spp.
b. Proteus mirabilis
d. Citrobacter freundli
195.
196.
197. Which of the following allows viability of acid production by oxidizers in oxidation-fermentation (o-F)
medium?
a. Carbohydrate contents is high
c. Peptone content is low
b. Sodium chloride content is low
d. Peptone content is high
198. An organism isolated from a stool culture has the followinh reactions; gram-negative rod; urease and
phenyialanine deaminase negative TSIA k/A +G,+h2S;motile; lysine decarboxylase-positive;indolepositive:Which of the following is oxidase-positive?
a. Yersinia enterrocolitica
c. Vibrio cholera
b. Aeromonas hydrophila
d. B and C
199. Which of the following is oxidase-positive?
a. Yersinia anterocolitica
c. Vibrio cholera
b. Aeromonas hydrophila
d. B and C
200. Which of the following is the reagent used for the string test?
a. Methyl red
c. Carbonate
b. Bromthymol blue
d. ).%% sodium desoxycholate
201. Any organism that is indole-positive and nitrate reduction-positive is also:
a. ONPG positive
c. Ornithine decarboxylase-negative
b. Cholera red-positive
d. Phenylalanine deaminase-positive
202. How many tubes of oxidation-fermentation media is are inoculated and what precautions should be taken?
a. 1 tube, under aerobic conditions
b. 2 tubes, one covered with oil so air excluded
c. # tubes, under anaerobic conditios
d. 2 tubes, under aerobic conditions
203. Which of the following is oxidase-positive?
a. Moraxella osloensis
c. Acinetobacter calcoaceticus biotype
b. Xanthomonas maltophilla
anitratus
d. Acinetobacter calcoaceticus biotype lwoff
204. Which of the following is part of the normal m flora of the skin?
a. Alcaligenes faecalis
c. Acinetobacter calcoaceticus biotype lwoff
b. Xanthomonas maltophilia
d. Moraxella osloensis
205. What does PRAS mean?
a. Prereduced, aerobically sterilized
c. Phosphate, rifampin, aerobically sterilized
b. Preerduced, anaerobically sterilized
d. Penicillin, rfampin, anaerobically sterilized
206. Why should thiolycollate broth be boiled for 10 minutes before being used?
a. To drive off oxygen
c. To activate the thioglycollate
b. To drive off hydrogen
d. To deactive the thioglycollate
207. Nagler agar is a selective medium for?
a. Bacteroides
c. Clostridium
b. Proplonibacterium
d. Arachnia
208. Peptococcus and peptostreptococcus are both:
a. Anaerobic, gram-positive cocci
c. Common agents of meningitis
b. Microaerophilic
d. Resistant to penicillin
209. Proplonibacteria are:
a. Anaerobic, gram-positive rods
c. Aerobic, gram negative rods
b. Anaerobic, Gram-negative rods
d. Aerobic, gram-positive rods
210. A gram-negative, anaerobicoccus that produces a red fluorescence under ultraviolet light is:
a. Peptococcus
c. Neisseria
b. Veillonella
d. fusobacterium
211. Which anaerobics, gram-positive rods produce terminal lollipop spores?
a. Ciostridium tetani
c. Clostridium butyricum
b. Eubacterium lentum
d. Bacteroides ureolyticus
212. Which of the following anaerobes causes an antimicrobial-associated diarrhea?
a. Clostridium ramosum
c. Clostridium butyricum
b. Clostridium difficile
d. Clostridium alactolyticum
213. Which of the following clostridla produce a double zone a hemolysis around colonies on blood agar?
a. Clostridium perfringens
b. Clostridium tetani

A1 PASSERS REVIEW CENTER 320-2728 //

29

c. Cllostridium sphenoides
d. Clostridium difficele
214. Mycoplasma pneumoniae is incubated:
a. Aerobicvally, in a sealed container, in
c. Aerobically
carbon dioxide
d. NOTA
b. Anaerobically in a GASPAk
215.
216.
217. A positive urease test for ureaplasmas is indicated by a:
a. `clear zone surrounding the colonies
c. Brown halo surrounding the colonies
b. Green zone surrounding tyhe colonies
d. Red halo surrounding the colonies
218. Which of the following organisms produce no hazr in a broth culture?
a. Mycopllassma hominis
c. Staphylococcus aureus
b. Mycoplasma pneumoniae
d. Mycloplasma pneumoniae
219. What is the cause of primary a typical pneumonia?
a. Klebsiella pneumoniae
c. Staphylococcus aureus
b. Streptococcus pneumoniae
d. Mycoplasma pneumoniae
220. Which of the following has both ribonucleic acid (RNA) and deoxyribonucleic acid (DNA)?
a. Rickettsia
c. Viruses
b. Chlamydia
d. A and B
221. Rickettsias are transmitted by:
a. Mosquitoes
c. Contamination
b. Aerosol
d. Arthropod vectors
222. Which of the following is not caused by Rickettsia?
a. Tyuphus
c. Typhoid fever
b. Q fever
d. Rocky mountain spotted fever
223. Which of the following have a bacterial type cell wail?
a. Viruses and chlamydias
c. Rickettsias and chlamydias
b. Myccplasma and chlamydias
d. Rickettsias and viruses
224. Which of trhe following is a small nonmotile, coccobacillus that is cultured in yolk sac of a chick embryo?
a. Chlamydia
c. Rickettsia
b. Mycoplasma
d. Bifidobacterium
225. Chlamydia are cultured in:
a. Yolk sac of chick embryo
c. Blood agar
b. Mccoy cells
d. NOTA
226. Genital cultures for sexually transmitted diseases include all of the following, except:
a. Chlamydia trachomatis
c. T. Pallidum
b. Herpes simplex virus
d. Campylobacter coli
227. TRICconjunctivitis includes which of the following?
a. Inclussion conjunctivitis
c. Trench fever
b. Trachonia
d. A and B
228. Which serological test is done for Chlamydia trachomatis on genital smears?
a. Enzyme-linked immunosorbent assay
c. Direct FA usisng monocional abs
(ELISA)
d. Indirect FA using Chlamydia ags
b. Radioimmunoassay (RIA)
229. What is the common tick-borne disease in the U>S?
a. Botulism
c. Lyme disease
b. Cholera
d. Legionnaires
230. What is erythema enronicum migrants (ECM)?
a. Cardinal sign of lyme disease at site of tick bite
b. Detects the entry of hookworm larvae
c. Initial stage in psittacosis
d. Rash in ricketnsial infectios
231. What are broad spectrum antibiotics?
a. Act against gram-negative bacteria
c. Act against bacterial and nonbacterial
b. Act against gram-positive bacteria
organisms
d. AOTA
232. Subtances, produced by microorganisms, that is very small amounts inhibit other microorganisms are
called:
a. Antibiotixcs
c. Antiseptic
b. Antibody
d. A and B
233. Bacteriostatic agents do which of the following?
a. Cause death of the organism
c. A and B
b. Inhibit the growth of the organism
d. Neither A nor b

A1 PASSERS REVIEW CENTER 320-2728 //

30

234. Thea.time
Full-life
takes half a dose of antibiotic to disappear from the blood
c. isHalf-life
called the:
b. Absorption time
d. Real-life
235. Resistance to an antibiotic can be transferred from resistant to a susceptible organism by
a. Selectivity pressure
c. Innate passage
b. Mutation
d. Extrachromosomal plasmid
236.
237. In the Kirby Bauer susceptibility test, the 0.5 Mcfarland standard is used to
a. Measure the thickness of the media in the Petri plate
b. Determine how close the antibiotic discs should be placed
c. Adjust the turbidity of the inoculums
d. AOTA
238. After inoculating the Mueller-Hinton plates for the Kirby Bauer susceptibility test, how long should the plates
dry before adding the disk?
a. 3-5 minutes
c. 30 minutes
b. Not more tahn 15 minutes
d. A and B
239. How does one measure the zone of growth inhibitor correlate with if the correct procedure has been used?
a. On the underside of the plate
c. Using a ruler, calliper or template
b. With the unaided eye
d. AOTA
240. What does the size of the zone of growth inhibition correlate with if the correct procedure has been used?
a. Maximum inhibitory concentration
c. Minimum lethal concentration
b. Minimum inhibitory concentration
d. Minimum bactericidal concentration
241. If there is too much moisture on the surface of a Mueller-Hinton plate when perfo ming a Kirby Bauer
susceptibility test, what will happen to the growth?
a. Larger zone
c. Confluent growth
b. Smaller zone
d. Band C
242. Which of the following are the best indicators of poor storage?
a. Chloramhenicol and cephalosporins
c. Penicillin and methicillin
b. Lincomycin and penicillin
d. Methicillin and linomycin
243. In Mueller Hinton agar, what can cause increased resistance of Psedomonas aeruglnosa to
aminoglycosides
a. Decrease peptone
c. Increased calcium and magnesium
b. Decreased sodium chloride
d. Decreased calcium and magnesium
244. When doing a susceptibility test with sulfonamides, there are two concentric zones around the disk. How
should the zone be measured?
a. Measure the diameter of the inner zone
c. Measure the both zones and take the
b. Measure the diameter of the outer zone
average size
d. Zone cannot be measured
245. The lowest concentration of drug that will kill all but the minimum defined proportion of viable organisms
after incubation for a fixed time under a given set of conditions is definition of the
a. Minimum lethal concentration
c. Maximum lethal concentration
b. Minimum bactericidal concentration
d. A and B
246. For quality control when monitoring reagents, how often should reagent disks be checked?
a. When container is first opened
c. Once each month
b. Once each week of use
d. A and B
247. How often should catalase , oxidase , and coagulase reagents be tested?
a. Once each day of use
c. When vial is first opened
b. Once each week of use
d. A and C
248. Record temperature of incubators, water baths, heating blocks, refrigerators, freezers and thermometers
a. At each time of use
c. At the end of each day
b. At the beginning of each day
d. AOTA
249. Check the face velocity of safety cabinets each
a. Day
c. Month
b. Week
d. Year
250. Settings of rpms marked on the face of therheostat control on the centrifuge should bechecked once
a. Weekly
c. Monthly
b. Daily
d. Every other week
e.

f. MICROBIOLOGY /MYCOLOGY
g.
1.
2. Characteristically , fungl:
a. Are members of the plant kingdom
c. Lack of chlorophyll
b. Lack roots and stems
d. AOTA
A1 PASSERS REVIEW CENTER 320-2728 //

31

3. Coenocytic hyphae is a synonym for


a. Sepatate hyphase
c. Paseudohyphea
b. Arthroconidia
d. Aseptate hyphae
4. Which type of mycoses produce no cellular response by the host?
a. Deep mycoses
c. Superficial mycoses
b. Cutaneous
d. Systemic mycoses
5. Arthrononidia on the outside of the hair as well as hyphae within the hair shaft is called
a. Ectothrix
c. Dimorphic
b. Endothrix\scutula
6. True yeast produce which of the following in the asexual stage?
a. Chlamydospores
c. Balstoconidia
b. Favic chandeliers
d. Arthroconidia
7. Septate hyphae have
a. No divisions
c. Pseudohyphae
b. Divisions
d. Vegetative parts
8. Molds with aseptate hyphae produce a specialized hyphal structure called a
a. Conidiophores
c. Thallospore
b. Sporangiophore
d. Zygospora
9. The basic , branching, intertwining structures of molds are referred to as
a. Mycelium
c. Cohidia
b. Phizoids
d. Spores
10. Arthroconidia are formed
a. Directly from the hyphae by fragmentation
b. By daughter cells piching off from portions of the mother cell
c. By knots of twisted hyphae
d. Within the hyphae
11. Mycelia within the colony that grow into the substrate are called
a. Aerial hyphae
c. Macroconidia
b. Conidiophores
d. Vegetative hyphae
12. Chlamydospores that grow within the hyphae are referred to as
a. Spirals
c. Intercalary
b. Terminal
d. Sessile
13. Budding forms in which the daughter cell forms as a bud from a single mother cell or forms laterally from a
mycelium or pseudomycelium are called
a. Blastopsores
c. Ascospores
b. Chlamydospores
d. Sporanglopores
14. Which of the following are the result of asexual reproduction?
a. Arthrospores
c. Zygospores
b. Ascospores
d. B and C
15. Conidia are
a. Asexual spores
c. Vegetative spores
b. Sexual spores
d. Asexual or sexual spores
16. Macroconidia are usually
a. Multicellular
c. Square
b. Unicellular
d. Rectangular
17. The swollen portion of the conidiophores is called the
a. Spherule
c. Sterigmata
b. Sporangium
d. Vesicle
18. Which of the following statements does not apply to true yeasts?
a. Ascospores are produced in the sexual phase
b. Blastospores are the sexual reproductive form
c. Saccharomyces is a representative of this group
d. They exhibit thermal dimorphism
19. Conidia with rough or spiny surfaces are described as
a. Catenate
c. Pedunculate
b. Echinulate
d. Seasile
20. Conidia in chains are said to be
a. Catenate
c. Pedunculate
b. Echinulate
d. Sessile
21. When preparing temporary mounts of fungal elements , why is the slide gently heated?
a. Preserves the specimen
c. Mounts the specimen
b. Increase the rate of clearing
d. AOTA

A1 PASSERS REVIEW CENTER 320-2728 //

32

22. Whicha.ofLactophenol
the followingcotton
acts as
blue
a clearing agent to eliminate debris and
c. make
70% alcohol
fungal elements more prominent
in skin,
b. 10%
hair,KOH
and nalls?
d. Ether
23. What is the stain in Aman medium?
a. Cotton blue
c. Prussian blue
b. Methylene blue
d. Bromthymol blue
24. Which of the following is negative stain?
a. Periodic acid Schiff stain
c. Acridine orange stain
b. Giemsa stain
d. India ink stain
25. In the periodic acid Schiff stain (PAS), what colors denote the presence of glycogen?
a. Blue to green
c. Pink to red
b. Red to violet
d. Yellow to brown
26. Which of the following stains is used for sharp delineation of fungal elements by fluorescent microscopy?
a. Gomoli methenamine silver
c. Hematoxylin
b. Calcoflour white
d. Eosin
27. When the fungus wall is invisible with the hematoxylin and eosin stain, which of the following may be used to
restain the slide?
a. Acid fast stain
c. Gram stain (Hucker modification)
b. Periodic acid Schiff stain
d. Gomori methenamine silver stain
28. Fungal elements in the acridine orange staining procedure
a. Stain red
c. Stain purple
b. Do not stain
d. Fluoresces a green color
29. How does MYCOSEL medium differ from Sabouraud dextrose agar?
a. Contains chloramphenicol and cycloheximide
b. Does not contain dextrose
c. Contains an indicator
d. Stimulates the production of chlamydospores
30. Which fungus medium is used for the primary isolation and maintenance of fungal cultures?
a. Sabouraud dextrose agar
c. Staibs medium
b. Cornmeal agar
d. Rice agar
31. What is the indicator in Dermatophyte Test medium?
a. Methyl red
c. Bromcresol purple
b. Phenci red
d. Methyl orange
32. If 1% glucose is added to cornmeal agar, trichophyton rubrum can be differentiated from Tricophyton
mentagrophytes by the production of
a. Hyphae
c. Pigment
b. Ascospores
d. Sporangia
33. Fungi should be incubated at
a. 25-30 C
c. 35-37C
b. 10-15C
d. 50-56C
34. How long should fungus cultures be held before reporting as negative
a. 5 days
c. 48 hours
b. 14 days
d. 30 days
35. Which of the following antibiotics can be added to media for the isolation of pathogenic fungi from
contaminated material?
a. Penicillin and streptomycin
c. Cyclohemixide and chloramphenicol
b. Vancomycin and nystatin
d. Streptomycin and nystatin
36. Which of the following media stimulates the productionof chlamydospores?
a. Brain heart infusion blood agar
c. Cornmeal agar
b. Czaapeks agar
d. Urease test medium
37. Which of the following media identifies species of aspergillus?
a. Urease medium
c. Czapeks agar
b. Rice agar
d. Ascospore agar
38. A positive result for the hair baiting test is
a. V-shaped penetration of the hair shaft
c. The production of red pigment
b. The production of germ tubes
d. The production of brown pigment
39. A tinea infection is commonly referred to as
a. Taeniasis
c. Pityriasis
b. Ringworm
d. Piedra
40. Which of these organisms is the usual cause of white piedra?
a. Trichosporon beigelli
c. Piedraia horate
b. Microsporum canis
d. Malassoizia furfur
41. Tinea ungulum is a dermatophyte infection of the
a. Nall plates
b. Beard

A1 PASSERS REVIEW CENTER 320-2728 //

33

c. Palm of hand
d. Feet
42. The favus type tinea capitis is caused by which of the following?
a. Trichophyton rubrum
c. Microsporum canis
b. Trichophyton tonsurans
d. All of these organisms
43.
44. Which of the following are/is anthrophillic?
a. Microsporum audouin
c. Microsporum gypseum
b. Microsporum canis
d. All of these organisms
45. Hairs infected with which of these dermafophytes fluoresce under Woods lamp?
a. Trichophyton rubrum
c. Microsporum canis
b. Trichophyton tonsurans
d. Epidermophyton floccosum
46. Colonization by dematophytic fungi of the hair, nails and skin is called
a. Mycetoma
c. Phaeohyphomycosis
b. Chromoblastomycosis
d. Dermatophytoses
47. Which of the following fungi is/are the cause of dermatophytoses?
a. Epidermophyton
c. Microsporum
b. Trichophyton
d. AOTA
48. 49. Which of the following produces macroconidia that are large, multicellular and club shaped with smooth walls?
a. Fonsecaea pedrosol
c. Epidermoph NOCCOSUM
b. Microsporum audouinii
d. Trichophyton mentagrophytes
50. Which statement is NOT true?
a. Trichophyton mentagrophytes is urease positive
b. Terichophyton mentagrophytes penetrates the hair shaft
c. Terichophyton rubrum is noted for its balloon forms
d. Terichophyton rubrum produces a rod pigment on some media
51. Which of the following produces small, very slow growing colonies and has favic chandeliers?
a. Terichophyton tonsurans
c. Trichophyton schonie
b. Microsporum canis
d. Epidemophyton floccosum
52. Which of the following is diagnostic for chromoblastomycosia
a. Flowerette conidia
c. Sclerotic body
b. Asteroid body
d. Genn tube
53. Which of the following are found in the yeast phase of sporothrix schenckii on culture medium at 37C?
a. Sclerotic bodies
c. Cigar shaped yeast cells
b. Asteroid bodies
d. Germ tube
54. Which of the following is the result of an antigen antibody reaction in cases of sporotrichosis?
a. Sclerotic body
c. Sleave conidia
b. Flowerette conidia
d. Asteroid body
55. In older mold cultures of Sporothrix schencikii, single conidia bome along the sides of the hyphae are referred as
a. Cigar shaped
c. Asteroid body
b. Flowerette
d. Sleeve
56. Which of the following infections is caused by dematiaceous saprobes that invade organs of
immunosuppressed hosts?
a. Phaephyphomycosis
c. Hyalohyphomycosis
b. Chromoblastomycosis
d. Dermatophytoses
57. The most common cause of mycetoma (maduramycosis) in the U. S is
a. Pseudoallescheria boydii
c. Rhinosporidium seeberi
b. Nocardia asteroids
d. Actinomadura madurae
58. Dematiaceous fungi have colonies with:
a. Surface and reverse side light
c. Surface light and reverse side dark
b. Surface and reverse side dark
d. Surface dark and reverse side light
59. Which of the following are a dematiaceous fungi?
a. Sporotrichum and aspergillus
c. Penicillin and acremonium
b. Phialophora and cladosporium
d. Fusarlum and paecilomyces
60.
61. Sclerotic bodies are found in the tissue in an infection with
a. Candida
c. Chrysosporum
b. Cladosporium
d. Coccidioides
62. Infection of chromoblastomycosis is NOT caused by which of the following fungi?
a. Exophiala
c. Phiatophora
b. Cladosporum
d. Fonsecaea
63. A fungus that grows as a mold at 35-37 C and a mold at 25C is described as
a. Monomorphic
b. Mycelia

A1 PASSERS REVIEW CENTER 320-2728 //

34

c. Dimorphic
d. Eumycetic
64. True fungi that produce hyphae and spore forms are known as
a. Eumycotic
c. Dimorphic
b. Monomorphic
d. Yeasts
65. Which of the following causes and infection by hyaline saprobes that invade most tissues or body fluids of
immunosuppressed hosts?
a. Aspergillus
c. Fusarium moniliforme
b. Penecillum marnefici
d. AOTA

A1 PASSERS REVIEW CENTER 320-2728 //

35

66. In you observe cleistothecia containing ascospores , report the organism as


a. Scedosporidium apiospermum
c. Nocardia asteroids
b. Actinomyces israelii
d. Pseudoalloscheria boydii
67. The actinomycetes are best classified as
a. Acid-fast bacteria
c. Funguslike bacteria
b. Eumycetes
d. Opportunistic fungi
68. Which of the following is anaerobic, gram positive but not acid fast and does not stain with fugal stains?
a. Nocardia asteroids
c. Actinomyces israelii
b. Nocardia braciliensis
d. Actinomadura madurae
69. Which of the following has a capsule?
a. Candida albicans
c. Crytococcus neoformans
b. Geotrichum candidum
d. Blastomyces dermatitis
70. When culturing crytococcus on Sabcuraud dextrose agar, which of the following antibodies should be in the
medium?
a. Trimethropin
c. Aminoglycoside
b. Vancomycin
d. Cycloheximide
71. In the direct microscopic examination of cerebrospinal fluid, which of the following can be mistaken for a
lymphocyte?
a. Candida
c. Geotrichum
b. Crytoccus
d. Blastomyces
72. Why does crytoccus neoformans produce a brown color on Birdseed or staib agar?
a. Assimilates creatinine
c. Ferments glucose
b. Assimilates niacin
d. Ferments trehalose
73. Cryptococcus neoformans latex agglutination test on spinal fluid detects crytococcal:
a. Creatinine
c. Carbohydrate
b. Antibody
d. Antigen
74. Cryptococcus neoformans latex agglutination test on spinal fluid detects cryptococcal:
a. Creatinine
c. Carbohydrate
b. Antibody
d. Antigen
75. Candida albicans produces
a. Blastospores
c. Pseudohyphae
b. Chlamydospores
d. AOTA
76. Which of the following is another name for oral candidiasis?
a. Ringworm
c. Barbers itch
b. Thrush
d. Favus
77. A presumptive identification of Candida albicans can be made by checking the ability of he organism to
produce what structure with serum
a. Blastospores
c. Germ tube
b. Chlamycospores
d. Pseudohyphae
78. Cornmeal agar plus tween 80 is used to identify candida albicans through the organism production of
a. Capsules
c. Germ tubes
b. Chlamydospores
d. Sporangia
79. Geotrichum candidum colonies appear as
a. Slow growing dematiaceous growth
c. Slow growing, violet, waxy growth
b. Rapid,yeastlike growth
d. Rapid growing suede like growth
80. Laboratory workers are in no danger when handling which form of dimorphic fungi?
a. Yeast form (tissue)
c. A and B
b. Hyphal form (culture)
d. Neither A nor B
81. Sepedonium produces spiny , tuberculate macroconidla but is a
a. Dimorphic fungus
c. Monomorphic mold
b. Yeastlike fungus
d. A and B
82. Which of the following are produced by Geotrichum candidum?
a. Rectangular, evenly staining
c. Cigar-shaped yeast cells
arthroconidia
d. Uberculate macrocchidia
b. Small, budding yeast cells
83. Which of the following is/are dimorphic fungi?
a. Hitoplasma capsulatum
c. Paracoccidioides brasiliensis
b. Blastomyces dermatitidis
d. AOTA
84.
85. The tissue phase of Histoplasma capsulatum, characteristically shows:
a. Pseudohyphae with blastospores
c. Yeast cells within macrophages
b. Yeast forms with multiple buds
d. Tuberculate chlamydospores
86. The characteristics structures of the mycelia phase of Histoplasma capsulanum are

A1 PASSERS REVIEW CENTER 320-2728 //

36

87.
88.
89.
90.
91.

92.
93.
94.

a. Blastopores
c. Tuberculate chlamydospores
b. Arthrospores with a germ tube in one
d. Yeast cells usually in monocytes
corner
Leishmania can be differentiated from Histoplasma because Leishmania has:
a. A different size
c. Central nuclear material
b. A different shape
d. NOTA
Which of the following tests may be sued instead of conversion when identifying dimorphic fungi?
a. Exoantigen test
c. Immunofluorescence test
b. String test
d. ELISA
In the mycelia form, Blastomyces dermatitidis produces
a. Conidiophores that resemble lollipops
c. Broad, nonseptate , hyphae
b. Wide, septate hyphae
d. Delicate, branching , acid fast hyphae
A yeast from with a single broad based bud would identify an oraganism as
a. Blastomyces
c. Histoplasma
b. Coccidloides
d. Sporothrix
The characteristic structures of the mycellial phase of blastomycosis are
a. Clusters of blastopores
c. Numerous intercalary or terminal
b. Small pear-shaped conidia called
chlamydospores
lollipos
d. Spherures filled with endospores
Yeastlike forms with multiple buds are characteristic forms associated with
a. Blastomyces dermatitis
c. Paracoccidioides brasiliensis
b. Candida albicans
d. Sporothrix schenci
Immature spherules of coccidioides are differentiated from yeast forms of Blastomyces because
a. They produce buds
c. Their buds have a narrow base
b. Their buds have a wide base
d. They never bud
Which of the following fungi is a major biohazard to laboratory personnel?
a. Coccidiodes immits
b. Cryptococcus neoformans
c. Fonsecaea pedrosoi
d. Histoplasma capsulation

A1 PASSERS REVIEW CENTER 320-2728 //

37

95. The exoantigen test is a/an


a. Indirect passive agglutination test
c. Neutralization test
b. Microscopic immunodiffusion test
d. Rocket electrophoresis technique
96. In the exoantigen test for Histoplasma capsulatum, which bands are present in appositive result?
a. H and/or M
c. A and/or D
b. J and/or L
d. O and/or P
97. When differentiating histoplasma capsulatum from Leishmania using special fungus stains, Leishmania
stains?
a. Red
c. Blue
b. Green
d. Will not stain
98. Which of bands in the exoantigen test for Blastomyces dermatitidis is present in a positive result?
a. A
c. H
b. G
d. M
99. Which of the following fungi produces a spherule that is filled with endospores ?

e.

f. MICROBIOLOGY AND PARASITOLOGY


g.
1. The motile, reproducing stage, feeding stage of the Protozoa is which of the following?
a. Cyst
c. Ova
b. Trophozoite
d. Adult worm
2. Which of the following amebae has chromatoid bodies in the cyst stage?
a. Endolimax
c. Dientamoeba
b. Iodamoeba
d. Entamoeba
3. Amebiasis is caused by
a. I. bustchli
c. E. Histolytica
b. E. Polecki
d. E. Coli
4. Which of the following organs of the body is most often involved in extraintestinal amebiasis?
a. Lungs
c. Pancreas
b. Kidneys
d. Liver
5. Which of the following is characteristic of true amebae?
a. Peripheral chromatin on the nuclear
c. Include pathogens and nonpathogens
body
d. AOTA
b. Chromatin bodies in the cyst
6. Which of the following is a true ameba?
a. E. Ana
c. D. fragilis
b. E. Hartmanni
d. I. butschlii
7. Amebae that inhibits the gastrointestinal tract of man are nonmotile nonfeeding and infective during which
stage?
a. Pseudopod
c. Trophozolite
b. Cyst
d. Cryptozoite
8. Mature cysts of E. Polecki have how many nuclei?
a. 2
c. 1
b. 8
d. 0
9. Which of the following has cysts with chromatoid bodies that have two pointed ends or that can be round,
triangular or oval/
a. E. Coli
c. E. Histolytica
b. E. Hartmann
d. E. Nana
10. Trophozoites of Entamoeba histolytica have the following characteristics
a. Small, delicate nuclear chromatin
b. Fine , even , peripheral chromatin
c. Progressive motility with hyaline, finger like pseudopods
d. AOTA
11. Which of these trophozoites , when acting as a pathogen , is likely to ingastred blood cells of the host?
a. E. Coli
c. E. Histolytica
b. E. Hartmann
d. E. Nana
A1 PASSERS REVIEW CENTER 320-2728 //
38

12. The point of differentiation between Entamoeba histolytica and E. Hartmanni is:
a. Presence of peripheral chromatin
b. Presence of chromatoid bodies
c. Only one possesses a cyst form
d. Size
13. The mature cyst of Entamoeba histolytica has how many nuclei?
a. One
c. Four
b. Two
d. More than four
14. Failure to find bacteria in purulent spinal fluid should alert one to find the possibility of an infection with:
a. Viruses
c. flagellates
b. Amebae
d. Worms
15.
16. Which trophozoite is distinguished by the possession of achromatic granules surrounding its karyosomai
chromatin?
a. E. Nana
c. E. Coli
b. E. Histolyca
d. I. butschlii
17. Which of the following wouyld have a double walled wrinkled cyst form?
a. N. Fowleri
c. A. Castellanii
b. E. Hartmanni
d. D. fragilis
18. A cyst that possesses a single nucleus and a large glycogen vacuole that stains deeply with iodine belong to
a. E. Nana
c. D. fragilis
b. I . butschlii
d. E. Coli
19. Some amebae have a spiny, hyaline extensions called/
a. Pseudopods
c. Cribriform plates
b. Limax forms
d. Acanthopods
20. Which are the best staining procedures for species of naegleria and Acanthamoeba?
a. Gram stain
c. Wright stain
b. Hermotoxylin and eosin
d. B and C
21. Amebae inhabiting the central nervous systems enter the body through the
a. Mouth
c. Ears
b. Nasal mucosa
d. Feet
22. In which specimen are Acathamoeba and naegleria usually found?
a. Blood
c. Urine
b. Cerebrospinal fluid
d. Joint fluid
23. Naegleria has which of the following characteristics?
a. Found in the brain
c. Trophozoite can become an ameba
b. Trophozoite can assume a limax form
flagellate
d. AOTA
24. The tropozoite whose karyosomal chromatin appears as a rosette of 4-6 granules is identified as
a. N. Fowleri
c. T. Gondii
b. D. fragilis
d. B. Coli
25. Which of the following are often mistaken for cysts of amebae?
a. Blastocystis hominis
c. Ammonium magnesium phosphate
b. Ammonium carbonate crystals
crystals
d. Epithelial cells
26. Intestinal flagellates are usually which shape in the trophozoite stage?
a. Round
c. Pear shaped
b. Oval
d. Triangular
27. Which of the following is pathognomonic for G. Lamblla and the stage it is found in?
a. Spiral groove trophozoite
c. Cytostome trophozoite
b. Undulating membrane cyst
d. Ventral sucking disk-trophozoite
28. Which of the following protozoa have an undulating membrane?
a. Trichomonas
c. Chilomastix
b. Trypanosoma
d. A and B
29. Infections with D. fragills can show which of the following symptoms?
a. Diarrhea
c. Asymptomatic
b. Abdominal discomfort without diarrhea
d. AOTA
30. What does D. fragills can show which of the following symptoms?
a. Both are ciliate
c. Neither has a cyst form
b. Both can appear as urine contaminants
d. AOTA
31. Which flagellate can be a pathogen of the small intestine?
a. Giardia
c. Chilomastix
b. Trichomonas
d. Strongyloides

A1 PASSERS REVIEW CENTER 320-2728 //

39

32.
33. Which intestinal flagellate trophozoite has a sucking disc , 2 nuclei, 8 flagella and an exostyle?
a. C. Meshili
c. G. Lamblia
b. D. fregilis
34. Red and white blood cells in stool specimens are characteristic of
a. Meningoencephalitis
c. Giardiasis
b. Bacillary dysentery
d. NOTA
35. A pear shape flagellate with jerky motility that is found in a urine specimen is identified as
a. T. Hominis
c. E. Coli
b. T. Vaginalis
d. Leptospira
36. Eighty percent of the trophozoites of D. fragilis have:
a. 2 nuclei
c. 1 nuclei
b. 3 nuclei
d. No nuclei
37. The onlybilaterally symmetrical protozoan is
a. Trichomonas
c. Giardia
b. Dientamoeba
d. Balantidium
38. Which of the following is the intracellular form of blood and tissue flagellates?
a. Crithidial
c. Trypanosomal
b. Leishmanial
d. Leptomonad
39. Which of the following is the cause of African sleeping sickness?
a. Leishmania
c. Plasmodium
b. Trypanosoma
d. Babesia
40. Which of the following is the vector of African sleeping sickness?
a. Reduviid bug (Triatoma)
c. Tsetse fly (Glossina)
b. Sandfly (Phlebotomus)
d. Ticks
41. Which species of Trypanosoma is the cause of Chagas disease?
a. Rhodesiense
c. Gambiense
b. Cruzi
d. A and C
42. Which of the following is characteristic of the trypanosomes of sleeping sickness/
a. Prominent kinetoplast
c. Thick organism
b. C-shaped
d. AOTA
43. Which specimen would be suitable for a demonstration of the trypanosomes of sleeping sickness?
a. Blood
c. CSF
b. Fluid from lymph node
d. AOTA
44. The extracellular form of Trypanosoma is a slender organism characterized by an undulating membrane and a
free flagellum that arises:
a. Anteriorly from the nucleus
c. Centrally from the anterior
b. Posteriorly from the kintoplast
d. Posteriorly from the nucleus
45. Which of the following is found within the reticulcendothelial cells?
a. L. Donovani
c. T. Gandil
b. H. capsulatum
d. AOTA
46. How is Trypanosoma cruzi transmitted?
a. Inhalation
c. Bite of reduviid bug
b. Bite of tsetse fly
d. Sexual contact
47. A chagoma is a lesion seen in infections with
a. Trypanosoma gambiense
c. Trichuris trichiura
b. Trichomans vaginalis
d. Trypanosoma
48. Which oraganism is the cause of kala-azar?
a. L. Donovani
c. T. Cruzi
b. L. Braziliensis
d. T. Gambiense
49. In the laboratory diagnosis of L. Donovani , which is the preferred specimen in which find Leihman Donovan
bodies?
a. Bone marrow
c. Vaginal secretions
b. Blood
d. CSF
50. Which of the following is the only elliate that is pathogenic in humans?
a. Babesia
c. B. Coli
b. Isospora
d. E. Coli
51. A very large cyst whose doubie wall encloses a ciliated organism with one visible nuclei would be
a. Transmitted by a mosquito to man
b. Acquired by a accident by man , since it customarily infects swine
c. A member of the sporozoa
d. Demonstrated only by the trichrome stain
52. Which of the following structures are used for motility of Balantidium coli?

A1 PASSERS REVIEW CENTER 320-2728 //

40

53.
54.
55.
56.

57.
58.

59.
60.
61.
62.
63.

64.
65.
66.
67.
68.
69.

70.
71.

a. Flagella
c. Pseudopodia
b. Cilia
d. Endulating membrane
Which nucleus in the trophozoite of Balantidium coli is the reproductive one?
a. Micronucleus
c. A and B
b. Macronucleus
d. Neither A nor B
Conjugation of trophozoites of Balantidium coli never occurs between
a. Large organisms
c. Large and small organisms
b. Same size organisms
d. Very large organisms
The definitive host (vector) to Plasmodium is the:
a. Tsetse fly (Glossina)
c. Female anopheles mosquito
b. Sandfly (Plebotomus)
d. Male reduviid bug (Tratoma)
Which of the malarial organisms presents as pale, very ameboid ring trophozoites , infecting a large pale red
blood cell with dots of haemoglobin?
a. P. Ovale
c. P. Falciparum
b. P. Malariae
d. P. Vivax
Which of the malarial organisms preferentially invades reticulocytes?
a. P. Falciparum
c. P. Vivax
b. P. Malariae
d. AOTA
The malarial organism whose schizont resembles a fruit ple in which the merozoite form a rosette around
the malarial pigments is
a. P. Malariae
c. P. Ovale
b. P. Vivax
d. P. Falciparum
Which malarial organism characteriscally has a band form trophozoite stretching across the red blood cell?
a. P. Malariae
c. P. Vivax
b. P. Ovale
d. P. Falciparum
The gametocyte of plasmodium falciparum can be differentiated from that of other malarial species by
a. Number of nuclei
c. Size
b. Nuclear chromatin
d. Shape
In which type of malaria can Ziemanns stippling be found?
a. Vivax
c. Ovale
b. Malariae
d. Falciparum
In which type of malaria can Maurers dots be found?
a. Vivax malaria
c. Ovale malaria
b. Malariae malaria
d. Falciparum
Which malarial organism features ring trophozoites that have double chromatin dots, appliqu forms and often
have multiple parasites in the infected red blood cells?
a. P. Vivax
c. P. Malariae
b. P. Falciparum
d. P. Ovale
Which malarial organism has large, coarse, are dots within a large, pale red blood cell with fimbriated edges?
a. P. Vivax
c. P. Ovale
b. P. Falciparum
d. P. Malariae
The sexual reproduction cycle in Plasmodium and coccidian is referred to as:
a. Sporogony
c. Sporocyst
b. Schizogony
d. NOTA
What is the infective stage of malarial parasite to the vector?
a. Cryptozoites
c. Gametocyte
b. Trophozoite
d. Ookinete
What is the infective stage of malarial parasite to humans
a. Gametocyte
c. Schizonts
b. Cryptozoites
d. Sporozoites
In which of malaria is there synchronized rupture of the red blood cells every 72 hours?
a. Vivax malaria
c. Ovale malaria
b. Falciparum malaria
d. Malariae
In falciparum malaria there may be a sudden massive intravascular hemolysis producing hemoglobinuria. This
is called;
a. Blackwater fever
c. Macrophilia
b. Hemoglobinopathies
d. A and C
Which haemoglobin is incomplete with malaria parasite survival?
a. Hb-CC
c. Hb-SS
b. Hb-F
d. Hb-A
What is the name of the laboratory test that allows laboratory bred reduviid bugs to feed on patients suspected
of having Chagas disease?
a. Complement fixation
b. Serodiagnosis

A1 PASSERS REVIEW CENTER 320-2728 //

41

c. Xenodiagnosis
d. Western blot
72. Humans are infected with Babesia by
a. Bite of ticks
c. Bite of sandfly
b. Blood transfusion
d. A and B
73. If immature cocysts of isospora belli are found in stool specimens from infected human, what would be done
with the specimen for identification?
a. Leave at room temperature
c. Leave in the incubator
b. Leave in the refrigerator
d. Leave in the water bath
74. What stage of isospora is infective to humans?
a. Oocysts
c. Larval
b. Pseudocysts
d. Tachyzoites
75. Isospora belli immature oocysts contains
a. Sporozoites
c. Pseudocysts
b. Tachyzoites
d. Sporoblast
76. Infective oocysts of isospora belli contain
a. Sporozoites
c. Pseudocysts
b. Tachyzoites
d. Sporoblast
77. The demonstration of retinochoroiditis and cerebral calcifications in a newborn would result in which of these
laboratory requests?
a. Casoni skin test
c. Ramon flocculation test
b. MHA-ABS test for congenital syphilis
d. Titer of toxoplasma antibodies
78. What is both the definitive and intermediate host of toxoplasma gondii?
a. Dog
b. Cow
c. Cat
d. Chicken
79. What is appearance of toxoplasma gondii in tissue fluids of man?
a. Oval
c. Limax form
b. Crescent
d. Ring form
80. What constitutes a positive result in the Sabin-feldman dye test?
a. Toxoplasma becomes nonmotile
b. Toxoplasma can no longer be demonstrated as an intracellular organism
c. Toxoplasma loses its affinity for methylene blue dye
d. The mouse into which the immune serum is injected does not die from toxoplasmosis
81. Which of the following tests is used for the detection of Cryptosporidium?
a. Sabin Feldman dye test
c. Xenodiagnosis
b. G-6-PD test
d. Sheathers sugar flotation
82. Which of the following parasites poses a particular hazard for immunodeficient or immunosuppresses
individuals?
a. Cryptosporidium
c. Strongyroides
b. Glardla
d. AOTA
83. Which of the following parasites is associated with AIDS?
a. Isospora helli
c. Sabeala
b. Sarcocystis
d. Cryptosporidium
84. Which operculated ovum contains a miracidium and can appear in sputum, often accompanied by blood and
charcot-leyden crystals?
a. Fasclola hepatica
c. Schistosoma mansoni
b. Paragonimus westermani
d. Dipylidium caninum
85. Which of the schistosome eggs may be recovered in rectal biopsy?
a. Schistosoma haematobium
c. Schistosoma japonicum
b. Schistosoma mansoni
d. B and C
86.
87. The scientific name of the head of a tapeworm is
a. Scolex
c. Strobila
b. Proglottid
d. Rostellum
88. Which of the following worms have an oral and ventral shaped sucker in the adult stage?
a. Cestodes
c. Nematodes
b. Trematodes
d. Filarae
89. Which of the following are hermaphroditic?
a. Flukes
c. Tapeworms
b. Roundworms
d. A and C
90. Which of the following is the first intermediate host of the flukes?
a. Snail
c. Fish
b. Water plant
d. Crab
91.
92. The common names for the schistosomes is

A1 PASSERS REVIEW CENTER 320-2728 //

42

a. Liver flukes
b. Intestinal flukes

c. Blood flukes
d. Lung flukes

93.
94. What is the common name for clonorchis sinensis?
a. Lung flukes
c. Intestinal fluke
b. Chinese liver flukes
d. Kidney fluke
95. What is a schistosomule?
a. Cercariae
c. Free-swimming cercariae
b. Cercariae minus tail
d. Meteceriae
96. The beef tapeworm is common name for
a. Taenia solium
c. E. Granulosus
b. Hymenolepis nana
d. T. Sagineta
97. Which tapeworm proglottid makes its way across the fecal specimen by doubling movements , and is seen
under the microscope to have numerous regular uterine branches (more than 15) resembling those of a
tree?
a. T. Saginata
c. O. Latum
b. T. Solium
d. D. caninum
98. Which of the following has an unarmed scolex?
a. Dwarf tapeworm
c. Pork tapeworm
b. Beef tapeworm
d. Dog tapeworm
99. Which species of Taenia has 7 to 12 uterine branches?
a. T. Solium
c. E. Granulosus
b. T. Saginata
d. NOTA
100. A hexacanth embryo enclosed in a radially striated shell belongs to the genus
a. Trichuris
c. Trichostrogyius
b. Taenia
d. Trichinella
101. Which tapeworm infection has to be treated with great care so that man does not acquire the larval infection?
a. Pragonimiasis
c. Dipylidiasis
b. Taeniasis solium
d. Diphyllobothriasis
102.
103. A bile stained egg that is 75 microns at its greatest diameter and contains a hexacanth embryo that lacks polar
knobs or filaments is that of
a. H. nana
c. D. latum
b. H. diminula
d. T. Saginata
104. The dwarf tapeworm infection is another name for
a. H. nana
c. D. latum
b. H. diminula
d. T. sollium
105. The eggs of Echinococcus granulosus are found in the feces of
a. Cats
c. Dogs
b. Fish
d. Cattie
106. Which of the following is found in the intermediate host of Echinococcus granulosus?
a. Adult worm
c. Hydatid cyst
b. Egg
d. Pseudocyst
107. Hydatid cyst infection of man is due to a larva of the tapeworm
a. D. latum
c. E. Granulosus
b. F. Hepatica
d. H. nana
108. The first intermediate host of Diphyllobothrium latum are:
a. Copepod
c. Fish
b. Snail
d. Crab
109. The second intermediate host of Diphyllobothrium latum are
a. Cattle
c. Crayfish
b. Water plants
d. Freshwater
110. What is the infective stage of the broad fish tapeworm to humans?
a. Plerocercoid
c. Hydatid cyst
b. Coracidium
d. Cysticercus
111. The eggs of this tapeworm are developed and have an opeculum at one end and a small abopercular knob at
the other end. The tapeworm is
a. D. latum
c. P. Westermani
b. D. caninum
d. H. diminuta
112. In some individuals , particularly those of Scandinavian extraction , this worm can cause megaloblastic
anemia.
a. D. latum
c. T. Solium
b. D. caninum
d. N. Americanus

A1 PASSERS REVIEW CENTER 320-2728 //

43

113. Whicha.ofIntestinal
the following
flukes
worms have separate sexes?
c. Tapeworms
b. Liver flukes
d. Blood flukes
114. The male roundworm is differentiated from the female roundworm by its
a. Dorsally curved posterior
c. Square posterior
b. Pointed posterior
d. Rounded posterior
115. The best way to demonstrate a pinworm infection is by which of the following techniques?
a. Rectal biopsy
c. Cellophane tape preparation
b. Third portion of urine voiding
d. Flotation method
116. The gravid female of Enterobius vermicualris deposits her embryonated eggs
a. In the small intestines
c. In the large intestines
b. On the perianal skin
d. In sputum
117. Eggs that average 60 microns in length, are oval with one flattened side, and contain a motile larva, are those
of:
a. E. Vermicularis
c. A. Lumbricoides
b. T. Solium
d. N. Americanus
118. Eggs that are bile stained and have clear polar plugs belong to the
a. Pinworm
c. Whipworm
b. Large roundworm
d. Hookworm
119. Which of the following nematodes does not have a free living stage?
a. E. Vermicularis
c. A. Lumbricoides
b. T. Solium
d. S. Stercoralis
120. Which stage of Trichiura is infective to humans?
a. Rhabditiform larva
c. Cyst
b. Filariform larva
d. Embryonated egg
121. Where does the larva of ascaris lumbricoides go after it hatches in the small intestine?
a. Migrates through the blood, liver , lungs, pharynx, and then back to small intestines
b. Migrates through the blood, brain and pharyx
c. Migrates through the lymph nodes and back to the intestines
d. It does not migrate
122. If an Ascaris egg lacks it bile stained mammilated coat, we refer to the egg as
a. Fertilized
c. Decorticated
b. Unfertilized
d. Mature
123. Trichuris trichiura, hookworm and_____ form the unholy three of roundworms
a. Enterobius vermicularis
c. T. Saginata
b. A. Lumbricoides
d. H. nana
124. Which of the following is the first stage larva of intestinal nematodes?
a. Filariform larva
c. Rhabditiform larva
b. Microfilaria
d. Cercaria
125. Which of the following organisms cause infections where the larvae migrate into the lymphatics and blood,
lung, alveoli, bronchioles, pharynx, and then the small intestines?
a. N. Americanus
c. S. Stercoralis
b. A. Duodenale
d. AOTA
126. Which of the following is characteristic of the eggs of hookworms?
a. An embryo in the two to eight cells stage of cleavage
b. Oval, thin shell
c. Clear space between shell and embryo
d. AOTA
127. The helminth that induces a hyperchromic microcyclic anemia in its host in the
a. Hookworm
c. Pinworm
b. Whipworm
d. Threadworm
128. The rhaditiform lava of the hookworm has a
a. Long buccal cavity equal to the width of the body
b. Short buccal cavity equal the width of the body
c. Medium length buccal cavity
d. A no buccal cavity
129. Which of the following larval worms enters the host by penetration of the ski?
a. S. Stercoralis
c. N. Americanus
b. A. Duodenale
d. AOTA
130. A roundworm that inhibits the small intestine and usually is demonstrated as rhabditiform larvae in the fecal
specimen is the:
a. Hookworm
c. Whipworm
b. Large roundworm
d. Threadworm
131. Muscle biopsy is a diagnostic technique employed to detect:

A1 PASSERS REVIEW CENTER 320-2728 //

44

132.
133.

134.
135.
136.
137.
138.
139.

a. Trichinosis
c. Trichuriasis
b. Taeniasis
d. Ascariasis
Which of the following roundworms give birth to her young?
a. Wuchereria
c. Brugia
b. Mansonella
d. AOTA
A viviparous female nematode:
a. Produces living larvae
b. Produces eggs
c. Does not require a male to produce eggs
d. Does not require a second intennediate host
On of the differential characteristics of the microfilariae is the presence or absence of
a. Nucleus
c. A dorsal curvature of the posterior end
b. A shealth
d. Flagella
Adults of filarial worms live in
a. Blood and feces
c. Subcultaneous tissues and blood
b. Urine and lymphatics
d. Lymphatics and subtaneous tissue
One of the pathological findings in infestations of Wuchereria is:
a. Blindness
c. Eye infections
b. Elephantiasis
d. Fibrotic nodules
The intermediate host of the guinea worm is the
a. Mosquito
c. Fly
b. Black fly
d. Copepod
Dirofilaria immitis is the
a. Dog heartworm
c. Rat lungworm
b. Dog hookworm
d. Cercanal dermatitis
Dracunculus infections cause ulcers that
a. On contact with water, release larvae
c. Appear on the eye
b. Contain eggs
d. NOTA

140.
141. Which of the following microfilariae does NOT have a shealth?
a. Wuchereria
c. Mansonella
b. Loa
d. Brugia
142. Which of the following is the eye worm?
a. Wuchereria
c. Onchocerca
b. Brugia
d. Loa
143. Cutaneous larva migrans or creeping eruption is caused by
a. Larvae of pinworms
c. Filariform larvae of dog hookworm
b. Microfilariae
d. Metacercariae
144. Which of the following causes a chronic cough , pulmonary and artery obstruction but no invasion of the heart
in humans/
a. Dirofilaria immitis
c. Anisakis
b. Philippinensis
d. A. Brasillensis
145. Which of the following is caused by the ingestion of snails by humans?
a. Dog hookworm
c. Dog heartworm
b. Rat lungworm
d. Cat tapeworm
146. Cercarial dermatitis, commonly known as swimmers itch, is caused by
a. Larvae of roundworms
c. Schistosomule
b. Eggs of tapeworms
d. Cercariae of schistosomes
147. What is the scientific name of the body of a tapeworm?
a. Strobila
b. Scolex
c. Rostellum
d. Proglottid

A1 PASSERS REVIEW CENTER 320-2728 //

45

148. The gravid segments of a tapeworm are filled with:


a. Male reproductive organs
c. Eggs
b. Female reproductive organs
d. Suckers
149. A six hooked embryo is known as
a. Proglottid
c. Strobila
b. Scolex
d. Hexacanth
150. Ground itchoccurs in infections of hookworms at the
a. Site of the larval penetration of the skin
c. Allergic reaction
b. During larval migration
d. A and C
151. Direct examination of stool specimens stained with iodine make the trophozoite of the Potozoa stain what
color?
a. Yellow
c. Trophozoites are destroyed by iodine
b. Brown
d. Do not stain
152. What color does the cytoplasm of protozoan cysts stain in an iodine wet stool preparation?
a. Red
c. Colorless
b. Green
d. Yellow-brown
153. What color do chromatoid bodies of protozoan cysts stain in an iodine wet stool preparation?

161. What will happen if blood from finger stick mixes with the alcohol use to clean the rea?
a. Fixes red blood cells
c. A and B
b. Thick blood firms become unsuitable
d. Neither A nor B
for staining
162. Venipuncture blood is not recommended for
a. Malaria, babesia, hemoflagellates
b. Amebae, malaria, babesia
c. Babesia, cryptosporidium, hemoflagellates
d. Isospora, amebae, hemoflagellates
163. To detect stippling, prepare blood films venipuncture within
a. 1-2 minutes
c. 2 hours after being drawn
b. 30 minutes to 1 hour after being drawn
d. 5-10 minutes
e.
f. MICROBIOLOGY AND PARASITOLOGY
g.
1. A mature virus particle containing a nucleic acid core surrounded by a protein coat, with or without envelope ,
is called a
a. Nucleocapsid
c. Virion
b. Genome
d. Capsomer
2. Viruses are characterized by the presence of
a. Mitochondrial DNA
c. Ribosomal RNA
b. DNA and RNA
d. RNA
A1 PASSERS REVIEW CENTER 320-2728 //
46

3. What is the largest DNA virus?


a. Herpesvirus
c. Adenovirus
b. Poxvirus
d. Parvovirus
4. Which of the following is the smallest RNA virus?
a. Paramyxovirus
c. Adenovirus
b. Togavirus
d. Enterovirus
5. What is the first step in the replication cycle of a virus?
a. Attachment and penetration
c. Release
b. Assembly of vision
d. Uncoating
6. The virus capsid morphology is
a. Square or round
c. Ribbon like or square
b. Long or short
d. Helical or icosanedral
7. From what part of the virus is the envelop acquired?
a. Nuclear or cytoplasmic membrane
c. Capsomeres or genomes
b. Nucleus or cytoplasm
d. mRNA or DNA
8. Where is the site of vision assembly?
a. capsomere
c. Genomes
b. Nucleus or cytoplasm
d. Membranes
9. Which of the following is a DNA virus?
a. Retroviridae
c. Herpesviridae
b. Orthomyxoviridae
d. Poxviridae
10. Which of the following is a RNA virus?
a. Pseudomyxoviridae
c. Harpesviridae
b. Paoviridae
d. Poxviridae
11. Specimen collected for virus isolation should be kept at:
a. 37C
c. 4C
b. 20C
d. 10C
12. At what temperature should clinical specimen suspected of containing viruses kept for transport that takes
days?
a. -70C
c. 4C
b. 37C
d. -10C
13. In waht family of viruses,after primary infection, does an individual become latently infected and then the
infection can be reactivated?
a. Adenoviridae
c. Pocviridae
b. Herpesviridae
d. Papovaviridae
14. Cytomegalovirus isolation is best accomplished using;
a. Monkey kidney cell
c. Human embryonic fibroblasts
b. A549 cells
d. Embryonated hents eegs
15. Which of the following viruses causes acute central nervous system disease in humans and animals?
a. Rabies
c. Mumps
b. Influenza
d. Measles
16. What specimens should be collected from a patient with suspected enteroviral meningitis?
a. Stool
c. Cerebrospinal fluid
b. Troat swab
d. AOTA
17. Influenza A viruses can be detected in cell cultures by
a. Complement fixation test
c. Immunoperoxidase test
b. Hemadsoption assay
d. Hemagglutination inhibition test
18. The best host systems for influenza virus isolation are
a. Monkey kidney cells and embryonated hens eggs
b. Monkey kidney cells and human embryonated hens eggs
c. Human embryotic fibroblasts and embryonic hens eggs
d. Monkey kidney cells and A549 cells
19. What common antigen in cross reactive in all human adenoviruses?
a. Fiber
c. Hemagglutinin
b. Haxon
d. Capsomere
20. The following characteristics are similar in both influenza and parainfluenza viruses, except:
a. Helical symmetry
c. Budding from cytoplasmic membrane
b. Lipid envelope
d. Segmented RNA genome
21. Myxoviridae virus envelope contains both
a. Neuraminidase and hemagglutinin
b. Lipoprotein and nonsegmented nucleocapsid
c. Lipid and double stranded DNA
d. Hemagglutinin and nonsegmented nucleocapsid

A1 PASSERS REVIEW CENTER 320-2728 //

47

22. Eosinophilic cytoplasmic inclusions are elementary bodies of the


a. Herpesvirus
c. Poliovirus
b. Poxvirus
d. Papilloma virus
23. German measles is caused by:
a. Rubeola virus
c. Varicella zoster virus
b. Rubella virus
d. Vaccinia virus
24. What is an alastrim infection?
a. Smallpox
c. Mild form of smallpox
b. Chicken pox
d. Measles
25. Which of the following procedures is routinely used for the detection of hepatitis B virus (HBV) in blood donor/
a. Solid phase RIA
c. Culture
b. ELISA
d. A and B
26. Which of the following laboratory procedures is the most rapid method for diagnosis of a virus infection?
a. Culture
c. Direct electron microscopy
b. Serological tests
d. Cytopathic effect (CPE) in cell cultures
27. What specimens should be collected from a patient with suspected influenza?
a. Stool
c. Urine
b. Throat swab
d. AOTA
28. What virus group contains a double stranded, segmented RNA genome?
a. Herpesvirus
c. Parainfluenza virus
b. Influenza virus
d. Reovirus
29. Enteroviruses can be differentiated from rhinoviruses by
a. Acid resistance
c. Ether stability
b. Size determination
d. Ribonnuclease treatment
30. What virus is the most frequent cause of gastroenteritis in children during the winter months?
a. Influenza viruses
c. Rotavirus
b. Adenoviruses
d. Hepatitis A virus
31. All of the following groups of viruses are resistant to ether treatment , except:
a. Herpesviruses
c. Enteroviruses
b. Adenoviruses
d. Reoviruses
32. The most common method for rapid electron microscopy examination is
a. Negative staining
c. Gram stain
b. Acid fast staining
d. Periodic acid-Schiff stain
33. Various virus groups can be differentiated in negativity stained specimens by their
a. Envelope
c. Morphology
b. Cytoplasmic membrane
d. Nucleous
e.

f. IMMUNOLOGY SEROLOGY
1. Where does differentiation of T cells occur?
a. Heart
c. Thymus
b. Brain
d. Bone marrow
2. Which organs or systems are capable of responding to circulating antigens with the production of specific
lymphocytes and plasma cells?
a. Lymph nodes and spleen
c. Reticuloendothelial system and biliary
b. Pancreas and liver
tract
d. Bacteria and granulocytic cells
3. How can cell mediated immunity be transferred to a normal individual?
a. With lymphoid cells from a hypersensitive individual
b. With transfer of specific liver cells from an infected individual
c. With transfer of monocytes from a normal person
d. With polymorphonuclear cells from a normal person
4. The first defectable antibodies in primary infections are usually
a. IgA
c. IgG
b. IgD
d. IgM
5. Which of the immunoglobulins is present in the greatest amount and rises later in an infection?
a. IgA
c. IgG
b. IgD
d. IgM
6. An IgG antibody is a good
a. Indicator of early infection
c. Indicator of allergy
b. Precipitating antibody
d. Agglutinating antibody
7. Antibodies are primarily
a. Alpha globulins
c. Albumin
b. Beta globulins
d. Gamma globulins
A1 PASSERS REVIEW CENTER 320-2728 //
48

8. Reactivea.sites
Determinants
on an antigen are known as
c. Haptens
b. Valence
d. Immunoglobulins
9. Which of the following cells recognize certain substances as foreign (nonself)?
a. T lymphocytes
c. Lymphocytes
b. Macrophages
d. Basophils
10. Which of the following cells ingest and destroy bacteria, damaged host cells, or tumors?
a. Macrophages
c. B cells
b. T cells
d. Basophils
11. T cells are involved with
a. Humoral immunity
c. Cell mediated immunity
b. Wheal and flare reaction
d. Antibody protection
12. Humoral immune response is due to
a. Macrophage activity
d. B lymphocytes that transform into
b. Suppressor T lymphocytes
plasma cells
c. Lymphokines
13. Which of the following cells are thymus dependent?
a. B cells
c. Macrophages
b. T cells
d. Plasma cells
14. The production of antibodies by B cells is turned off by
a. Suppressor T cells
c. Helper T cells
b. B lymphocytes
d. Cytotoxic T cells
15. Lymphokines, fiberated by sensitized T cells recruit
a. Neutrophils
c. Macrophages
b. Monocytes
d. All of the above
16. A heterologous antigen reacts with
a. An antibody whose production it included
b. An antibody whose production it did not induce
c. Nonspecific substances
d. Specific substances
17. Which immunoglobulin is able to cross the placenta?
a. IgA
b. IgG
c. IgD
d. IgE
18. Which immunoglobulin is predominant in body secretions?
a. IgA
c. IgM
b. IgG
d. IgE
19. Which immunoglobin plays a role in hypersensitivity ?
a. IgM
c. IgE
b. IgG
d. IgD
20. The binding strength of an antibody for an antigenis referred to as its
a. Specificity
c. Avidity
b. Affinity
d. Titer
21. The ability of clinical test to be positive in the absence of its homologous antigen is referred to as its
a. Specificity
c. Affinity
b. Sensitivity
d. Titer
22. The random movement of antigen- antibody complexes in semisolid medium occurs in
a. Agglutination
c. Complement fixation
b. Immunofluorescence
d. Gel diffusion
23. Antigen and antibody are both free to move toward each other to form precipitate in
a. Two-dimensional diffusion
c. Linear diffusion
b. Single radial diffusion
d. Three dimensional diffusion
24.
25. A combination of electrophoresis and double immunodiffusion is referred to as
a. Double radial diffusion
c. Immunofluorescence
b. Immunoelectrophoresis
d. Electroimmunodiffusion
26. In immunoelectrophoresis, which of the following forms at the equivalence point of antigen and antibody?
a. Spurs
c. Troughs
b. Precipitin bands
d. Line of intersection
27. An electrophoretically abnormal protein displace from the normal position may be recognized by
a. Precipitin band of moderate curvature
c. Precipitin band markedly curved
b. Lines of fusion
d. gull wingformation
28. Immunoelectrophoresis, which of the following forms at the equivalence point of antigen and antibody?
a. Spurs
c. Troughs
b. Precipitin bands
d. Line of intersection
29. Electroimmunodiffusion is directed movement of antigen or antigen and antibody in

A1 PASSERS REVIEW CENTER 320-2728 //

49

30.

31.
32.
33.
34.
35.
36.
37.

38.
39.
40.

43.

44.
45.
46.
47.
48.
49.

a. Solid media
c. Liquid media
b. Semi solid media
d. Any of the above
Which of the follwoign is an advantage of counterimmunoelectrophoresis?
a. Precipitation lines not sharp
b. Precipitation does not occur at the intermediate point
c. Precipitation lines visible within 30 minutes
d. None of the above
In the radioimmunoassay test the zero standard is labelled as
a. Bo
c. Antigen x
b. Antigen O
d. B1
Which of the following fluorochromes gives off a red emission at 580 nm?
a. Auramine rhodamine
c. Truant fluorochroma
b. Fluorescein isothiocyanate
d. Tetramethylrhodamine isothiocyanate
Which of the following fluor chromes give a minimal flase reading?
a. Auramine-rhodamine
c. Truant fluorochroma
b. Fluorescein isothiocyanate
d. Teramethylrhodemine isothiocyanate
Immunofluorescence tests employ a compound that is able to absorb light of a certain wavelength and to
a. Emit light of a longer wavelength
c. Emit white light
b. Emit light of a shorter wavelength
d. Phosphoresce
Which of the following is the energy source of fluorescence microscope?
a. Woods lamp
c. Mercury vapour lamp
b. Brightfield microscope
d. Black light
What system can be used in fluorescent staining technics to give a bright fluorescence?
a. Quantum efficiency
c. Enzyme inhibition
b. Biotin avidin
d. All of the above
In the enzyme linked immunosorbent assay (ELISA) . which of the following can be attached to a solid phase
support (e.g. polystyrene)?
a. Antigen and enzyme
c. Antigen and antibody
b. Antibody and albumin
d. Antigen and albumin
In the enzyme linked immunosorbent assay (ELISA), the antihuman globulin is
a. Fluorochrome conjugated
c. Hormone-conjugated
b. Auramine-conjugated
d. Enzyme-conjugated
Interferon can be produced by which of the following?
a. Macrophages
c. Virus infected cells
b. T lymphocytes
d. All of the above
One dimensional single electroimmunodiffusion is also known as
a. Immunoelectrophoresis
c. Radioimmunoassay
b. Counterimmunoelectrophoresis
d. Rocket lectrophoresis
41.
42.
Which of the following is the diagnostic laboratory test for acquire immunodeficiency syndrome (AIDS)?
a. Skin test
b. Parasites found in stool
c. <200 CD4 + T-lymphocytes /uL or a CD4+T-lymphocyte percentage of total lymphoresis<14
d. Differential blood count
Acquired immunodeficiency syndrome (AIDS) characteristically shows a deficiency in
a. Neutrophils
c. B lymphocytes
b. T lymphocytes
d. L. E cells
Acquired immunodeficiency syndrome (AIDS) shows a change in the ratio of
a. T/B cells
c. A/G
b. B1/B2 cells
d. T4/T8 cells
Acquired immunodeficiency syndrome is transmitted in all the following ways EXCEPT:
a. Blood transfusion
c. Sneezing and coughing
b. Sexual contact
d. Sharing of hyperdemic needles
Fc is which fragment of the antibody molecule?
a. Heavy chains
c. Crystallisable
b. Hinge
d. Antigen-binding
The basic immunoglobulin structure consists of
a. 1 light and 2 heavychains
c. 2 light and 1 heavy chains
b. 2 light and 2 heavy chains
d. 1 light and 1 heavy chain
In the structure of immunoglobulins, where is the antigen binding site located?
a. Carboxy terminal or constant region
c. Amino terminal or variable region
b. Disulfide bridge
d. All of the above

A1 PASSERS REVIEW CENTER 320-2728 //

50

50. Which ofa.the


Papain
following is a proteolytic enzyme that fragments an antibody?
c. Properdin
b. Anaphylatoxin
d. Zymosan
51. After an antibody has been fragmented , which term denotes the antigen binding fragment?
a. Fe
c. Fab
b. Disulfide bond
d. Carboxyl terminal
52. The antigen used in precipitation tests is
a. Soluble
c. Particulate
b. Insoluble
d. Cellular
53. Postzonal reaction is caused by
a. Too little antigen
d. Not by differing amounts of antigen and
b. Too much antigen
antibody
c. Too much antibody
54. The optimal ratio, when the most antibody is precipitated by the least amount of antigen , is referred to as
a. Opsonisation
c. Zone of inhibition
b. Zone of equivalence
d. Widal formation
55. When a precipitation reaction is converted to agglutination by increasing the size of the antigen particles the test
is then referred to as
a. Direct agglutination
c. Indirect (passive) agglutination
b. Optimal agglutination
d. Prozone reaction
56. What type of antigen antibody reaction is usually performed in the detection of the HBs antigen of hepatitis?
a. Viral hemagglutination
c. Passive agglutination
b. Capillary precipitation
d. Hemagglutination
57. Effective phagocytosis occurs in the presence of
a. Regain
c. Sensitivity
b. Anti-drug antibodies
d. Opsonins
58. Serological tests employed to identify antigens of antibodies are referred to as what type of test
a. Nonspecific
c. In vivo
b. In vitro
d. Cross reaction
59. Hybridomas are formed from
a. Antibodies
c. T lymphocytes
b. B lymphocytes
d. Phagocytes
60. How does a spleen cell become Immortal?
a. Reacts with an epitope
c. By neutralization
b. By being phagocytised
d. By fusing with a tumor cell
61. When a preparing monoclonal antibcoies
a. Inject an animal with an antigen
c. Fuse B lymphocytes with a malignant cell
b. Remove the B lymphocytes
d. All of the above
62.
63.
64. A hybridoma consists of a clone of fused cells of
a. Guinea pig kidney cells
c. Mouse lymphocytes
b. Rat myeloma cells
d. B and C
65. Which of the following are limitations of monoclonal antibodies from a hybridoma?
a. React with only 1 antigenic determinant on a multivalent antigen
b. Do not cross link to form a precipitate
c. Are not good in precipitation or hemagglutination assays
d. All of the above
66. Monoclonal antibodies cannot be used in complement fixation tests because they
a. Cannot cross link to form a precipitate
c. Can fix complement
b. Cannot fix complement
d. Are homogeneous
67. Complement fixation tests contain which two systems?
a. Antigen antibody systems
c. Test and indicator systems
b. ELISA and latex systems
d. Hemolysin and titration systems
68. What is the indicator in the complement fixation tests?
a. Sheep and red blood cells
c. Latex antigen absorbed particles
b. Human and red blood cells
d. Carbon particles
69. Complement proteins are designated by all of the following terms EXCEPT:
a. Numbers (e. g. C1, C2 )
c. Abbreviations (e. g. Fc)
b. Symbols (e. g. factor B)
d. Trivial names (e.g. properdin)
70. A hemolysin is an antibody that
a. Causes lysis of red blood cells
c. Lyses T cells
b. Causes agglutination of red blood cells
d. Sensitives white blood cells
71. What is the purpose of antisheep hemolysin in a complement fixation test/

A1 PASSERS REVIEW CENTER 320-2728 //

51

6.

8.

a. Sensitives sheep red blood cells


c. Agglutinates sheep red blood cells
b. Hemolyzes sheep red blood cells
d. Neutralizes patient antibody
72. A positive reaction in a complement fixation test is expressed as
a. Agglutination
c. Inhibition of hemolysis
b. Hemolysis
d. Neutralization
73. What is the purpose of the hemolytic system in a complement fixation test?
a. To test the patients serum for the presence of antibodies
b. To act as an indicator and provide a visible reaction
c. To test the red blood cells
d. To make certain that the serum is not anticomplementary
74. It too much hemolysin is added to a complement fixation test, what happens to the result:
a. Incomplete hemolysis
c. Complete hemolysis
b. Sensitivity is decrease
d. Hemagglutination
75. What should occur when controls for complement fixation tests are used to check for anticomplementary
factors?
a. No hemolysis
c. Hemagglutination
b. Hemolysis
d. All of the above
76. In a positive control for complement fixation tests, what should occur?
a. No hemolysis
c. Hamagglutination
b. Hemolysis
d. Fluorescence
77. What is used to sensitize red blood cells in the complement fixation tests?
a. Sheep red blood cells
c. Opsonics
b. Antisheep hemolysin
d. Horse red blood cells
78. In a negative complement fixation test sensitized red blood cells will
a. Clump
c. Be hemolyzed
b. Not clump
d. Not be hemolyzed
79. Antigen needles for the qualitative Venereal Disease Research Laboratory test (VDRL) shoud deliver
a. 1/100 (100 drops/ml)
c. 1/120 (120 drops /ml)
b. 1/60 (60 drops/ml)
d. 1/75 (75 drops/ml)
Saline needles for the quantitative venereal disease research laboratory test (VDRL) should deliver
a. 20 drops/ml
c. 100drops/ml
b. 120 drops/ml
d. 110 drops/ml
77. Needles used to deliver VDRL antigen should be checked:
a. Once a month
c. Daily or every time tests are performed
b. Bimonthly
d. Weekly
VDRL antigen emulsion is stable for:
a.
24 hours
c.
7 days
b.
48 hours
d.
30 days
79.
80. Patients with syphilis develop an antibody response to a substance known as:
a.
Reagin
c.
hemolysibn
b.
complement
d.
cardiolipin
81. the reaction between the antibody and the antigen in the venereal disease research laboratory test (VDRL)
is known as:
a.
precipitation
c.
flocculation
b.
neutralization
d.
hemolysis
82. if the interval between heating the serum of the VDRL test and testing exceeds 4 hours, the serum should
be reheated at
a.37C for 15 minutes
c.56C for 15 minutes
b.37C for 30 minutes
d.56C for 10 minutes
83.
84. In the VDRL test, a zonal reaction usually appears as which of the following results?
a.Reactive
c.Weakly reactive
b.Nonreactive
d.Any of the above
85. The serum: antigen ratio in any VDRL test maintained at
a.1;2
c.2;1
b.1;1
d.3;1
86. Inactivation of sera for serology tests is performed for what purpose?
a.Destruction of complement
b.Increasing sensitivity of test
c.Removal of particulate matter
d.Restoration of refrigerated sera to an appropriate temperature for testing
87. If a VRDL test shows small clumps of antigen with many free particles, it would be reported as:

A1 PASSERS REVIEW CENTER 320-2728 //

52

a.Reactive
c.Biological false positive
b.Nonreactive
d.Weakly reactive
88. How would be following VDRL quantitative test be reported?
89.
1:1
1:2
1:4
1:8
1:16
1:32
1:64
1:128
90.
W
W
R
R
R
R
W
N
91.
a. Reactive 4 dills
93.
c. Reactive 64 dills
92.
b. Reactive 32 dills
94.
d. cannot be reported
95. What role does cardio lipan play in the VDRL test?
a.It is the center for absorption of tissue lipids
c.It is related to reactivity of the test
b.It is related to sensitivity of the test
d.All of the above
96. Which of the following represents the reported titer in a quantitative VDRL test?
a.Lowest dilution giving a weakly reactive result
b.Lowest dilution giving a reactive result
c.Highest dilution giving a weakly reactive result
d.Highest dilution giving a reaction result
97. The quantitative VDRL test is performed on all sera in which the quantitative VDRL test is
a.Nonreactive
c.Weakly reactive
b.Reactive
d.Either reactive or weakly reactive
98. Which laboratory test for syphilis uses plastic coated cards?
a.Rapid plasma regain ( RPR)
b.Venereal disease research laboratory (VDRL)
c.Fluorescent treponemal antibody absorbed (FTA-ABS)
d.Microhemagglutination praponema pallidum (MHA-TP)
99. Rapid plasma regain (RPR) antigen contains cardio lipin with
a.10% saline
c.Charcoal particles
b.Lipoteichoic acid
d.Fluorescent isothiocyanate
100. The rapid plasma regain (RPR) test:
a.Uses zinc particles
c.Does not require a period of rotation
b.Does not require inactivate of serum
d.Employs treponemal antibodies
101. The rapid plasma regain (RPR) test is rotated at what speed for what length of time?
a.200 rpms for 10 minutes
c.180 rmps for 4 minutes
b.100 rpms for 8 minutes
d.125 rmps for 2 minutes
102. The rapid plasma regain (RPR) antigen needle with deliver how many drops?
a.60 drops per 1.00 ml
c.100 drops per 0.50 ml
b.60 drops per 0.504 ml
d.100 drops per 0.25 ml
103. The antigen that reacts with the antibody substance in the patient serum in the rapid plasma regain
(RPR)test is
a.Sheep red blood cells
c.Cardiolipin
b.Cardiolipin with added charcoal particles
d.Lecithin
104. The treponema pallidum immobilization (TPI) test requires
a.Regain
c.Treponema pallidum fixed on a slide
b.An extract of a pathogenic treponeme
d.Living treponema pallidum
105. Which of the following serves as the absorbent in the fluorescent tr6eponemal antibody test-absorbed
(FTA-ABS) test?
a.Reiters tereponeme
c.Treponema palildum, Nichols strais
b.Treponema microdentlum
d.Treponema pertenue
106. The fluorescent treponemal antibody-absorbed (FTA-ABS) test is used to identify which of the following in
the patients serum?
a.Treponemal antibody
c.Regain
b.Treponemes
d.Cardiolipin
107. The fluorescein-antihuman gamma globulin used in the FTA-ABS test
a.Makes the antigen-antibody reaction visible
b.Is added to the serum before the antigen is added
c.Is added to the antigen before the serum is added
d.Makes the antibody visible
108. The principle of the antistreptolysin O (ASO) tubetest is
a.Complement-fixation
c.Micro-hemmaglutination
b.Enzyme-inhibition
d.Precipitation
109. In the classic antistreplolysin O (ASO) test, streptolysin O should not be rehydrated until it is needed
because it:
110. The serum titer in the anti-strepplolysin O (ASO) test is reported as the
a.Highest dilution that gives a positive result
b.Lowest dilution that gives a negative result

A1 PASSERS REVIEW CENTER 320-2728 //

53

c.Asao units
d.Todd or international units
111. What does the antistreptolysin O (ASO) test measure in tyhe patients serum?
a.Oxygen labile streeptolysin
c.Sheep red blood cells
b.Antibody to atreptolysin O
d.Streptococcal antibodies
112. The antistreptolysin O (ASO) slide test is resd for the presence of:
a.Hemolysis
c.A mucin clot
b.Precipitation
d.Agglutination
113. A streptococcal extracellular product that dissolves fibrin clots is:
a.Streptokinase
c.streptodornase
b.groupA carbohydrates
d.hyaluronidase
114. the reactive on DNA-methyl green substrates, when testing for the presence of anti-DNase (antistreptornase) are
a.blue (+) yellow (-)
c.green (+) colorless (-)
b.red (+) orange (-)
d.purple (+) yellow (-)
115. Which f the following is NOT a streptococcal extracellular product?
a.Nicotinamide adenine dinucieotidase
c.Hyaluronidase
b.Isomerases
d.Deoxyribonuclease B
116. What type of red blood cells are used in the traditional heterophil antibody test?
a.Human group O
c.Rabbit
b.Sheep
d.Horse
117. In the presumptive heterophil antibody test, which of the following constitutes a passive reaction?
a.Agglutination
c.Precipitation
b.Hemolysis
d.Neutralization
118. A passive david sohn heterophil antibody test;
a.Shows hemolysis
b.Differentiate three types of heterophil antibodies
c.Indicates the presence of typhoid fever
d.Uses fresh plasma
119. The heterophil antibody test is reported as the:
a.Highest dilution that show hemolysis
b.Highestdilution that shows agglutination
c.Lowest dilution that shows hemolysis
d.Lowest agglutination that shows agglutination
120. In the heterophil antibody test, when serum is absorbed with beet cells
a.There is a decrease in the titer of infections mononuclecsis mantibodies
b.There is an increase in the titer of guinca pig kidney antibodies
c.Serum sickness antibodies are absorbed
d.Forssman antibodies are absorbed
121. Infactious mononucleosis is a self-limited disease of the reticulo endo t he lial system caused by:
a.Hepatitis C virus
c.Epstein-barr virus
b.JC virus
d.Herpes simpes simplex virus
122. Forssman antibodies are absorbed by
a.Boiled beef cells
b.Guinea pig kidney antigen
c.Neither beef cells nor guinea pig kidney antigen
d.Both beef cells and guinea kidney antigen
123. The heterophil slide test of lee and davidsohn uses what type of blood cells?
a.Sheep
c.Horse
b.Rabbit
d.Human group O
124. The weil-felix is used for the detection of which type of antibodies?
a.Salmonella
c.Rickettsia
b.Mycoplasma
d.Viral
125. The widal and weil-felix reactions are examples of techniques used to detect
a.Heterophil antibodies
c.Febrile agglutinins
b.Regain
d.Forssman antigen
126. OX 19 and OX 2 refer to
a.Strains of proteus vulgaris
c.Serotypes of brucella abortus
b.Antigens of rickettsia prowazeki
d.Asntibodies to salmonella typhi
127. In the widal test, an elevated O titer indicates
a.Currenty infection
c.Recer vaccination
b.Antigens in agglutinins to salmonella typhi
d.A cross reacbn
128. Serological cross reactions are usual

A1 PASSERS REVIEW CENTER 320-2728 //

54

a.Indicative of infection with an unrelated organism


b.In lower titers than are specific reactions
c.Detected only later tan are specific reactions
d.Of a different type than are specific reactionsm
129. The antigen used to determine the somatic antigenic grouppi9ng of enteric bacteria is
a.O
c.Ki
b.H
d. Vi
130. Brucella aborted will ceross-react with antibodies of
a.Salmonella typhi
c.Streptococcus pneumoniae
b.Proteus vulgaris
d.Franciscella tularensis
131. Which of the following can interfere with the cryptococcal antigen latex agglutination test?
a.Thyroglobulin
c.Rheumatoid factor
b.Moncional protein
d.C- reactive protein
132. The rheumatoid factor (RF) latex slide test uses diluted patients serum plus
a.Beta globulin
c.Alpha globulin
b.Microsomal antigen
d.Gamma globulin
133. Which of the following antibodies is directed against the Fc portion of IgG?
a.Antisheep hemolysin
c.Rheumatoid arthritis
b.Anitistreptolysin
d.Regain
134. The presence of C- reactive protein in a patients serum indicates
a.Inflammation
c.Group A beta hemolytic streptococcal
b.Pneumococcal pneumonia
infection
d.Typoid or paratyphoid fever
135. In the rheumatoid factor latex test
a.The antibody identified is IgG
b.Synovial fluid may give a positive reaction when serum is negative
c.The test is more specific than the Rose- Waaler sheep cell agglutinin test
d.A titer of 1:20 identifies the patients discase as rheumatoid arthritis
136. What is the incubation temperature for the cold agglutinin test?
a.Body temperature
c.Refrigerator temperature
b.Room temperature
d.Freezer temperature
137. Cold agglutinins may develop after infection with
a.Klabsiella pneumonia
c.Streptococcus pneumoniae
b.Mycoplasma pneumonia
d.Haemophilus influenzae
138. Which of the following cold reacting antibodies is present in infections of mycoplasma pneumoniae?
a.Anti-I
c.Anti Lewis
b.Anti-i
d.Anti-K
139.
140. Which pattern of fluorescence in the anti- nuclear antibody (ANA) test is most closely associated with
active systemic lupus erythematosus (SLE)
a.Cytoplasmic
c.Nucleolar
b.Homogenous
d.Speckled
141. A positive anti-ribonucleoprotein (anti-RNP) in the antinuclearvantibosy group is suggestive of which
autoimmune disease?
a.Mixed connective tissue disease (MCTD)
c.Scleredema
b.Rheumatoid arthritis (RA)
d.Systemic lupus erythematosus(SLE)
142. What does ds in anti-ds-DNA and anti ds- RNA mean?
a.Dormant stage
c.Disease sign
b.Double stranded
d.Detectable symptom
143. Which of the following is the major autoantigen of the thyroid gland?
a.Thyroglobulin
c.Monocional protein
b.Rheumatoid factor
d.Thyroxine
144. Which herpes simplex virus (HSV) can cause urogenital infections?
a.Type 1
b.Type 2
c.Type 3
d.A and B
145. A method used to serotype the herpes simplex virus (HSV) is the
a.Indirect fluorescent antibody method
c.Cell typing
b.Cell culture
d.Direct fluorescent antibody method
146. Which of the following methods has a high degree of specificity when typing the herpes simplex virus
(ELISA)
a.Direct fluorescent antibody

A1 PASSERS REVIEW CENTER 320-2728 //

55

b.Cell culture
c.enzyme linked immunosorbent assay (ELISA)
d.cell typing
147. which of the following is the easiest serological test for varicella zoster virus (VZV)
a.enzyme-linked immunosorbent assay(ELISA) test
b.complement fixation test
c.fluorescent antibody-membrane antigen (FAMA) test
d.direct fluorescent antibody test
148. which of the following is the cause of chicken pox?
a.Rubella virus
b.Primary infection of varicella zoster virus
c.Primary infection of human immunodeficiency virus
d.Herpes simplex virus
149. Blood donors should be routinelt tested for acquired immunodeficiency syndrome (AIDS) and
a.Varicella zoster virus
c.Cytomegalovirus
b.Herpes simplex virus
d.Rubella virus
150. Cytomegalovirus (CMV) infection can be diagnosed by which of the following serological test?
a.Igm anf IgG antibody
c.Direct fluorescent antibody test
b.Cell culture
d.Indirect human globulin test
151. Which test for cytomegalovirus has problems with false positive?
a.Copmlpement fixation test
c.Indirect immunofluorescent antibody test
b.Radioimmunoassay
d.Direct fluorescent aantibody test
152. The best serological method used to test for a rubella virus infection is the
a.Complement fixation test
c.Indirect fluorescent antibody test
b.Direct fluorescent antibody test
d.Enzyme-linked immunosorbent assay test
153. The most popular serological test for the diagnosis of Toxoplasma gondii is the
a.Complement fixation test
c.Indirect fluorescent antibody test
b.Direct fluorescent antribody test
d.Enyzyme-linked immunosorbent assay test
154. In the enzyme-linked immunosorbent assay (ELISA) method for typing herpes simplex virus (HSWV),
which type of red blood cells are used?
a.Sheep
c.Horse
b.Goat
d.Human
155. Which of the following is a good screening test for determining immunity to the rubella virus?
a.Direct agglutination
c.IgG antibody test
b.Complement fixation
d.IgM antibody test z
e.
f.
g.

h.

HEMATOLOGY

1. The larges quantity of normal body iron is found in the form of:
a. Transferrin
c. Free ferric iron
b. Hemosiderin
d. Hemoglobin
2. What is the ratio of iron to porphyrin in heme?
a. 1:1
c. 4:1
b. 1:4
d. 1:2
3. What is the function of the enzyme methemoglobin-cytochrome C reductase in a red blood cell?
a. To keep iron in the ferrous state
b. To keep iron in the ferric state
c. To maintain alpha and the beta chains of hemoglobin
d. To maintain an anaerobic pathway of glycolysis
4. What product is formed when ferrous iron of normal hemoglobin is oxidized to the ferric state?
a. Methemoglobin
c. Cyanmethemoglobin
b. Carboxyhemoglobin
d. Sulfhemoglobin
5. Which of the following hemoglobins could be present in a normal adult
a. Hb-A
c. Hb-F
b. H-b-A2
d. AOTA
6. Most of the hemoglobin in the newborn is found to be:
a. Hb-A
c. Hb-D
b. Hb-A2
d. Hb-F
7. Hemoglobin C disease is characterized by:
a. Target cells
c. Rod-shaped crystals
b. Basophilic stippling
d. Schihistocytes

A1 PASSERS REVIEW CENTER 320-2728 //

56

8. In the
a. Potassium
tube solubility
phosphate
test for hemoglobin S the reagent responsible
c. for
Saponin
the reduction of the hemoglobin
molecule
b. Sodium
is: dithionate
d. Drabkins solution
9. In hemoglobin C, glutamic acid is replaced is:
a. Valine
c. Cystine
b. Lysine
d. Arginine
10. Which of the following hemoglobins is insecute under lowered oxygen tension?
a. Hb-A,
c. Hb-S
b. Hb-A
d. Hb-F
11. The Kleihauer technic is used to differentiate is:
a. Hb-A2 from Hb-F
c. Hb-A2 from Hb-A
b. Hb-S from Hb-A
d. Hb-S from Hb-F
12. The cyanmethemoglobin method measure:
a. All hemoglobin pigments
c. All hemoglobin pigments except
b. Only oxyhemoglobin
sulfhemoglobin
d. Only methemoglobin
13. The diluents used in the cyanmethemoglobin method for hemoglobin determination is:
a. Pilods fluid
c. Sodium carbonate
b. 0.1NHCI
d. Drabkinsoln
14. The plasma protein that binds free hemoglobin is:
a. Myoglobin
c. Hemosiderin
b. Haptoglobin
d. Transferrin
15. The normal plasma hemoglobin per dL is:
a. 2-3mg
c. 200-300mg
b. 20-30mg
d. 2-3grams
16. A hematocrit is a measure of:
a. Red blood cell sedimentation rate
c. Total red blood cells
b. Tolal red blood cell mass
d. Packed red blood cell volume
17. An increase in the erythrocyte sedimentation rate(ESR) can be the result of an increase in the quantity of:
a. Fibrinogen
c. Platelets
b. Promrombia
d. Red blood cells
18. The WBC/ul of an undiluted synovial fluid that has 175 white blood cells counted in the large center square
of ahemocytometer is:
a. 17,500
b. 3,500
c. 7,000
d. 1,750
19. The RBC,ul of a body fluid that has 285 red blood cells counted in the 5 red blood cells squares with a 1,100
dilution is:
a. 1,425,000
c. 5,700,000
b. 142,000
d. 285,000
20. The addition of 0.02 ml of blood to 5.0 ml diluents results in a partial dilution of:
a. 1:25
c. 1:250
b. 1.50
d. 1:500
21. Lyse- resistant red blood cells in specimens aspirated in automated hematology analyzers occur with:
a. Multiple myeloma
c. Acute leukemia
b. Sickle cell disease
d. Anemia
22. On automated hematology analyzers, marked red blood cell fragments will cause an abnormal histogram
and/or scatterplots for:
a. Whit blood cells and red blood cells
b. Red blood cells and platelets
c. White blood cells, and platelets
d. White, blood cells, red blood cells, and platelets
23. The white blood cell count, when the differential count has 5 nucleated red blood cells and the automated
analyzer white blood cell count is 105, is.
a. 10.5
c. 11.0
b. 10.0
d. 9.5
24. What is the most likely cause of these result? WBC 6.5; RBC 1.69;Hb 9.1; Hct 22.8;MCv 135; MCH 53.8;
MCHC 39.9
a. Improperly mixed specimen
c. Cold agglutinin
b. Hereditary spherocytosis
d. Instrument reagent depleted
25. The factor for the conversion of counted white blood cells to WBC/ cu.mm in the manual count is:
a. 50
c. 2,000
b. 1,000
d. 10,000
26. To avoid or false crystals, synovial fluid should be collected in:
a. Oxalate
b. Powdered ethylene diaminetetraacetic acid (EDTA)

A1 PASSERS REVIEW CENTER 320-2728 //

57

27.
28.
29.

30.
31.
32.

33.
34.
35.
36.

38.
39.
40.
41.
42.
43.

44.
45.

c. Lithium heparin
d. Liquid ethylene diaminetetraacetic (EDTA)
Pilots fluid is the reagent used in the determination of:
a. Eosinophils
c. Red blood cells
b. Hemoglobin
d. Platelets
The following results indicate a noninflammatory syncvial fluid:
a. WBC <2,000; PMNS 30%
c. WBC< 2,000;PMNs>50%
b. WBC 2,000-5000; PMNs.50%
d. WBC 2,000-5000;PMNs <30%
The color of wright-stained red blood cells may be and usted by:
a. Dipping the slide in methanol before staining
b. Adjusting the buffer pH
c. Decreasing the concentration of glycerol in the stain
d. Rapid drying of slides
Parasites in red blood cells are best demonstrated with which stain?
a. Giemsa
c. New methylene blue
b. Wright
d. Crystal violet
To calculate the mean corpuscular volume , one must know the erythrccyte count and the
a. Hemoglobin
c. Hematocrit
b. Red blood cell count
d. Mean cell diameter
The blood film in iron deficiency anemia snows:
a. Occasional burr cells with decreased central pallor of macrocytes
b. Nypochromic microcytes with poikilocytes
c. Normochromic normocytic cells with basophilic stippling
d. Numerous oval macrocytes with decreased platelets
Which determination is considered a reliable assessment of effective erythropoiesis in the bone marrow?
a. Reticulocyte count
c. Red blood cell count
b. Hemoglobin
d. M.E ration of the marrow
After a due to acute blood loss is usually:
a. Microcytic
c. Normocytic
b. Macrocytic
d. Hypochromic
Abnormal variation in the size of erythrocyte is known as:
a. Anisocytosis
c. Erythrocytosis
b. Poikilocytosis
d. Leptocytosis
Erythrocytes with a diameter of 9-12 microns are referred to as:
a. Macrocytes
c. Laptocytes
b. Megalocytes
d. Drepanocytes
37.
Abnormal variation in shape of the erythrocyte is known as:
a. Anisocytosis
c. Megalocytosis
b. Poikilocytosis
d. Microcytosis
The red blood that shows a peripheral rime of the hemoglobinwith a darl staining central area is a:
a. Spherocyte
c. Schistocyte
b. Target cell
d. Sickle cell
The red blood cell with the bulleyeappearance is present in:
a. Thalassemia
c. Postsplenectomy
b. Liver disease
d. ACTA
Increased iron in the mitochrondia of normoblast are characteristics of:
a. Aplastic anemia
c. Chronic anemia
b. Sidercblastic anemia
d. Iron deficiency anemia
Chronic blood loss can lead to:
a. Iron deficiency
c. Vitamin B12 deficiency
b. Tissue iron deposits
d. Polycythemia
The characteristics finding in iron deficiency anemia is:
a. Low serum iron and low total iron
c. Low serum iron and high TIBC
pinding capacity (TISC)
d. High serum and low TIBC
b. Low serum iron and normal TIBC
Polycythemia vera is associated with:
a. Panycytosis
c. Philadelphia chromosome
b. Pancytopenia
d. Teardrop red blood cells
A macrocytic anemia characteristically accompanies all of the following , except:
a. Vitamin B12 deficiency
d. Fish tapeworn infection in some
b. Folate deficiency
individual
c. Iron deficiency

A1 PASSERS REVIEW CENTER 320-2728 //

58

46. Which
a. Decreased
of the following
platelets
is not typically seen in the peripheral smearc.inSpherocytes
megaloblastic anemia?
b. Oval macrocytes
d. Hypersegmented neutrophils
47. Punctate basophilla refers to which of the following?
a. The deposits of lead in the red blood cells
b. The aggregation of residual RNA in the red blood cells
c. The presence of DNA remnants in the red blood cells
d. Iron deposits in the red blood cells
48. An increased osmotic fragility is observed in the red blood cells of
a. Sickle cell anemia
c. Hereditary spherocytosis
b. Hereditary elliptocytosis
d. Thalassemia
49. Which of the following is true of hereditary elliptocytosis?
a. Osmotic fragility and autohemolysis usually normal
b. Unless 100% of the red blood cells are oval the diagnosis is doubtful
c. It is the only disorder associated with elliptocytosis
d. An abnormality of hemoglobin is responsible for this condition
50. Red blood cells that contain iron stained granules aret= termed:
a. Reticulocytes
c. Siderocytes
b. Megaloblast
d. Drepanocytes
51. Which of the following inclusion represents a remnant of nuclear DNA in a red blood cell?
a. Heinz body
c. Pappenheimer body
b. Dohle body
d. Howell-jolly body
52. RNA remnants in red blood cells are visualized when stained with?
a. Prussian blue
c. Peroxidise
b. New methylene blue
d. Heinz body stain
53. Speherocytes differ most from normal red blood cells in their:
a. Decreased ratio of surface area volume
c. Increased volume
b. Increased central pallor
d. Increased resistance to hypotonic saline
54. Cabot rings are inclusion found in:
a. Eosinophils
c. Monocytes
b. Erythrocytes
d. Lymphocytes
55. Match each of the following red blood cells with the correct description
56. 1. excess membrane for cell volume (bulls eye)
57. 2. decreased cell membrane with same cell volume
58. 3. pitting of the red blood cell inclusion by spleen
59. 4. crystallization of abnormal hemoglobin during oxygen deprivation
Bite cell
Sickle cell
Spherocyte
Target cell
e.
55. Match the cell content with the appropriate inclusion
f.
1. RNA
g.
2. DNA
h.
3. Iron
i.
4. Hemoglobin
a. Howell-jolly body
c. Reticulocyte
b. Heinz body
d. Pappenheimer body
56. Match the following with the correct definition
57. 1. decreased white cells , red cells and platelets
58. 2. granulocytes released prematurely to myelocyte stage
59. 3. increased polychromosia
60. 4. white blood cell count >50,000/cl
61. a. left shift
63. c. pancytopenia
62. b. reticulocytosis
64. d. leukomoid reaction
65. The most characteristics change seen in the white blood cells of the megaloblastic anemia bone marrow is:
a. Toxic granulation
c. Hyposegmentation
b. Giant bands and metamyelocytes
d. Dohle bodies
66. Burr cells represent what abnormality of red blood cells?
a. Damage to the red blood cell membrane
b. Change in hemoglobin content
c. Change in hemoglobin content
d. An abnormal hemoglobin replacing the normal varieaty
67. Of the following which is a white blood cell inclusion?
a. Schuffners granules (dots)
c. Dohle bodies
b. Heinz bodies
d. Cabot rings
68. G-6 PD deficiency is an abnormality in:
a. Glutamic acid
b. Erythropoiesis

A1 PASSERS REVIEW CENTER 320-2728 //

59

1.
2.
3.
4.

c. Emden-Meyerhoff pathway
d. HMP shunt
69. The white blood cell least often seen on a normal peripheral smear is the:
a. Neutrophil
c. Basophil
b. Eosinophil
d. Monocyte
70. In the Pelger- Huet anomaly, neutrophils shown
a. A shift to the left
c. 2 lobes and hyperclumped chromatin
b. Dohle bodies
d. Toxic granulation
71. Dohle inclusion bodies are found in the cytoplasm of:
a. Lymphocytes
c. Neutrophils
b. Monocytes
d. Erythrocytes
72. Toxic granulation of neutrophils is usually associated with:
a. Infectious mononucleosis
c. Bacterial infections
b. Leukemias
d. Pelger-Huet anomaly
73. Which of these is NOT consistent with toxic granulation of polymorphonuclear neutrophils (PMNs)?
a. Accompanies hypersogmentation
b. Appears with a shift to the left
c. May be mimicked by overstaining
d. May be accompanied by vacuolization and /or Dohle bodies
74. In examining a stained film for lupus ( LE)cells, the most probable source of confusion is:
a. Smudge cells
c. Gauchers cells
b. Tart cells
d. Downey cells
75. In lupus (LE)the cells that phagocytises the nucleoprotein is usually a:
a. Lymphocyte
c. Monocyte
b. PMN
d. Histocyte
76. A monocytwe that has phagocytised a nucleus is called a:
a. LE cell
c. Flame cell
b. Tart cell
d. Smudge cell
77. Leukoppenia can occur in which of these conditions?
a. Leukemia
c. Marrow injury by chemical agents
b. Viral infections
d. All of the above
78. Acute leukemias are usually associated WITH:
a. Immature cells of one white blood cell
c. Anemia
series
d. All of the above
b. Thrombocytopenia
79.
80.
81. At what stage will specific (secondary) granules of granulocytes appear?
a. Promyelocyte
c. Myelocyte
b. Metarnyelocyte
d. Band neutrophil
82. Chronic myelocytic leukemia ( CML) can be distinguished from polycythemia vera by:
a. Thrombocytosis
c. Lekocytosis
b. Splenomegaly
d. LAP-low to zero
83. The white blood cell anomaly that may be confused with a shift to the left is:
a. Chediak higashi
c. May hegglin
b. Chronic granulomatous disease
d. Pelger-huet
84. Reactive leukocytosis is present when the absolute count of granulocytes are:
a. >6000/ul
c. >9000/ul
b. >4000/ul
d. >8000/ul
85. What is required for the in vitro formation of LE cells?
a. Antinuclear antibodies
c. Free nuclear material
b. Neutrophils
d. AOTA
86. Match the following white blood cell anomaly with the correct cellular component
a. Alder-Relly
c. Chedlak-Higashi
b. Pelger-Huet
d. May-Hogglin
Neutrophil inclusions resembling Dohle bodies
Large dark granules in granulocytes caused by anomaly of primary granules anfd lymphocytes by lysosomal
granules
Failure to complete development of nuclear lobes
Dense granulation of granulocytes caused by abnormal enzyme
87. Which of the following findings rule out acute leukemia?
a. Anemia
c. Normal platelet count
b. Auer rods
d. Immature cells
88. Eosinophilia is found in which of the following?

A1 PASSERS REVIEW CENTER 320-2728 //

60

a. Diabetic acidosis
c. Allergic rhinitis
b. Trichinosis
d. B and C
89. What specific cytological finding can differentiate acute myelocytic leukemia (AML) irom acute lymphocytic
leukemia (ALL)?
a. Leukpcyte alkaline phosphatase (LAP) activity
b. Number of nucleoli in blast cells
c. Auer rods in blast cells
d. Variation of size of blast cells
90. The results on leukemia patient was alpha napthol- AS-D-acetase-esterace (NASDA) 3+blast NASDA+ Naf
O+ suggest:
a. Acute granulocytic leukemia (AGL)
c. Acute lymphocytic leukemia (ALL)
b. Acute monocytic leukemia (AMol)
d. Acute nonlymphocytic leukemia
91. What stain differentiates acute myelocytic leukemia from acute lymphocytic leukemia?
a. Alpha-napthol- AS D- acetate
c. Paroxidase
esterace (NASDA)
d. Methyl green pyronine
b. Leukocyte alkaline phosphatase (LAP)
92. The leukemia that does not have peroxidise activity is?
a. Acute myelomonocytic
c. Acute lymphocytic
b. Acute monocytic
d. Acute myelocytic
93. The immediate precursor of the band polymorphomuclear neutrophil is the:
a. Myelocyte
c. Stab
b. Promyelocyte
d. Metamyelocyte
94. The largest leukocyte in the normal peripheral blood smear is the:
a. PMN
c. Lymphocyte
b. Megakarocyte
d. Monocyte
95. A shift to the left means an increase in:
a. Polymorphonuclear neutrophils
c. Hypersegmented PMNS
b. Immature granulocytes
d. Metatubricytes
96. An increased number of eosinophils may be observed in all of the following except:
a. Erytroleukemia
c. Parasitic infection
b. Chronic granulocytic leukemia (CGL)
d. Allergies
97. A characteristics of immature blood cells is:
a. A nuclear cytoplasmic (N:C) ratio
c. Pale blue cytoplasm
b. Fine nuclear chromatin
d. Absence of nucleoli
98.
99.
100. The characteristics cell found to be increased in the peripheral smear of infectious mononucleosis is a:
a. Plasma cell
c. Lymphonocytes
b. Sand PMN
d. Monocytes
101. In chronic lymphocytic leukemia, there is usually a:
a. Thrombocytopenia
c. Lymphocytosis
b. Basophils
d. Lymphopenia
102. In chronic granulocytic leukemia, there is usually a:
a. Leukopenia
c. Leukocytosis
b. Lymphocytosis
d. Eosinopenia
103. A peripheral blood smear demonstrating an absolute increase in small hypermature lymphocytes and
smudge cells would be suggestive of:
a. chronic lymphocytic leukemia (CLL)
c. leukomoid reaction
b. acute lymphocytic leukemia (ALL)
d. infectious mononucleosis
104. The Philadelphia chromosome is associated with:
a. chronic granulocytic leukemia
c. myelofibrosis
b. chronic lymphocytic leukemia
d. acute granulocytic leukemia
105. Auer rods are characteristically found in the cells of which leukemia?
a. Acute granulocytic leukemia (AGL)
c. Acute lymphocytic leukemia (ALL)
b. Chronic granulocytic leukemia (CGL)
d. Chronic lymphocytic leukemia (CLL)
106. A low neutrophil alkaline phosphatase NAP/LAP is seen in:
a. Leukomoid reaction
c. Pregnant women
b. Chronic granulocytic leukemia
d. Polycythemia vera
107. A positive myeloperoxidase stain indicates the possibility of:
a. Acute lymphocytic leukemia
c. Myelomonocytic leukemia
b. Chronic lymphocytic leukemia
d. Infectious mononucleosis
108. An increased titer of antinuclear antibody (ANA) aids in the diagnosis of:
a. Systemic lupus erythematosus
b. Acute lymphocytic leukemia

A1 PASSERS REVIEW CENTER 320-2728 //

61

c. Infectious mononucleosis
d. Autoimmune hemolytic anemia
The leukemia that has a positive periodic acid Shift stain is associated with:
a. Acute promyelocytic
c. Acute monocytic
b. Acute myelocytic
d. Acute lymphocytic
110. Using the automic cell counters may increase a mean corpuscular volume (MCV) due to:
a. Cold agglutinins
c. In vitro hemolysis
b. Excess EDTA
d. Decreased WBC
111. Ninety-five percent of chronic lymphocytic leukemias have surface markers of:
a. Cells
c. Terminal deoxynucleotidyl transferase
b. T cells
d. CALLA
112. All of the following are characteristics of a bone marrow of Waldenstroms macroglobulinemia, except:
a. Marked rouleux formation
c. Increased tissue mass cells
b. Increased abnormakl lymphoid cells
d. Increased megakaryocytes
113. Reed- Stemberg cells are diagnosis for:
a. Chronic lymphocytic leukemia
c. Hodgkins disease
b. Prolymphocytic leukemia
d. Multiple myeloma
114. The leukemia that has increased basophils and/or eosinophils in the early stages is:
a. Chronic lymphocytic leukemia
c. Acute lymphocytic leukemia
b. Chronic myelocytic leukemia
d. Hairy cell leukemia
115. Lymphosarcoma cells can be described as:
a. Central nucelolus, condensed nuclear chromatin, light basophilic cytoplasm
b. Small prominent nucleoli , fine nuclear chromatin with deep clefts in thenucleus
c. Plasmacytoid
d. Small mature lymphocytes with dense nuclear chromatin
116. A leukemia that presents with leukopenia in > 50% of patients with > 90% massive splenomegaly is most
likely:
a. Hairy cell leukemia
c. Progranulocytic leukemia
b. Prolymphocytic leukemia
d. Manocytic leukemia
117. The cytochemical stain, peroxidase is negative for:
a. Monocytes
c. Neutrophils
b. Lymphocytes
d. Eosinophils
118. Most chronic lymphocytic leukemia are necpiasms of:
a. A lymphocytes
c. T4 helper cells
b. B lymphocytes
d. T8nsupprssor cells
119.
120. In the presence of marked culeaux formation on a peripheral smear, one could expect the same sample
to exhibit an increased:
a. Hemoglobin
c. Sedimentation rate
b. RBC count
d. WBC count
121. Giant bizarre platelets nucleated red blood cells(nRBCs);and teardrop red blood cells are associated with:
a. Erythroleukemia
c. Myelofibrosis
b. Multiple myeloma
d. May- hegglin
122. The last red blood cell precursor to retain the nucleus before it becomes an erythrocyte is:
a. Rubricyte
c. Rubriblast
b. Prerubricyte
d. Metarubricyte
123. The most reliable criteria for the estimation of a cells age is:
a. Cytoplasm
c. Nuclear chromatin
b. Nucleus
d. Granules
124. Which of the following laboratory results can occur with intravascular hemolysis of the red blood cell?
a. Increased plasma hemoglobin
c. Decreased plasma naptoglobin
b. Hemoglobinuria
d. All of the above
125. A peripheral blood smear from a patient with thalassemia would exhibit:
a. Teardrops cells,target cells Howell- jolly bodies
b. Target cells,nucleated red blood cells , basophilic stippling
c. Burr cells, nucleated red blood cells, sickle cells
d. Basophilic stippling re blood cells,acanthocytes
126. What laboratory results are found in an iron deficiency anemia but not in thalassemia?
a. Normal to high serum iron and normal total iron binding capacity
b. Low serum iron and increased total iron binding capacity
c. High red blood cell count and low mean corpuscular volume
d. Low red blood cell count and high mean corpuscular volume
127. The specific diagnosis of sicklecell anemia can be made from the following laboratory test is:
a. Solubility
b. Sodium metabisulfite
109.

A1 PASSERS REVIEW CENTER 320-2728 //

62

c. Hemoglobin electrophoresis
d. Presence of sickle cells
The second most common abnormal hemoglobin in the worl , which predominates in southeast Asians is:
a. Hb_S
c. Hb-C
b. Hb-E
d. Hb-M
129. The autoantibody that has maximal intravascular hemolysis at low temperature is:
a. IgG
c. Complement
b. IgM-binding complement
d. IgG binding complement
130. The FAB classification of the lymphocytic leukemia with prominent cytoplasmic vacuolation is:
a. L2
c. L1
b. L3
d. M2
131. Acute lymphocytic leukemias that have a predominance of homogenous cells with a high N/C ratio
belong to which FAB group?
a. L1
c. L2
b. L3
d. M2
132. In acute myelocytic leukemia the FAB classification of type II blasts that distinguishes them from type I
blast is:
a. Few cytoplasmic granules
c. Uncondensed chromatin
b. Cytoplasmic granules absent
d. Nucleoili prominent
133. The stain that is positive for hairy cell leukemia is;
a. Oil Red O
c. ACP
b. PAS
d. Napthol AS D acetate
134. Laboratory results of Hodgkins disease are
a. Normal erythrocytesedimentation rate, high white blood cell count, low serum iron
b. Low white blood cell count, low serum iron,high fibrinogen
c. High white blood cell count with neutrophilia, high erythrocyte sedimentation rate, low serum iron
d. High gamma globulin , low white blood cell count with neutrophilia , high erythrocyte sedimentation
rate
135. Laboratory findings of myelold metaplasia with myelofibrosis is:
a. Anemia, teardrops cells, low alkaline phosphatase, increased uric acid
b. Bizaare platelets, high hemoglobin, no blast cells
c. 5-10% blast, schistocytes, low leukocyte alkaline phosphatase
d. Teardrop cells, increased uric acid , high white blood cell count
136.
137.
138.
128.

139. COAGULATION QUESTION


1. Which of the following factors does not require vitamin K to its production by the liver?
a. II
c. VIII
b. VII
d. IX
2. The activated coagulation factor that is NOT is serine protease is:
a. X
c. IX
b. VII
d. VIII
3. The precursor of thrombin is:
a. Thromboplastin
c. Prothrombinase
b. Florinogen
d. Prothrombin
4. Fibrinogen is converted to fibrin by:
a. Thrombin
c. Plasmin
b. Thromboplastin
d. Calcium
5. The coagulation family that is defendent on vitamin K synthesis is:
a. Contact
c. Fibrinogen
b. Prothrombin
d. AOTA
6. The end result of most coagulation test is the production of:
a. Thrombin
c. Fibrin clot
b. Prothrombinase
d. Fibrinolysis
7. In the process of fibrinolysis, which of the following acts on fibrin?
a. Fibrinogen
c. Thrombin
b. Plasmin
d. Factor XIII
8. Secondary hemostasis consist of:
a. Platelet change shape
b. Secretion of adenosine diphosphate (ADP) and adenosine triphosphate (ATP)
c. Irreversible platelet aggregation
d. All of the above
9. The prothrombinase complex consist of:
A1 PASSERS REVIEW CENTER 320-2728 //

63

10.
11.
12.
13.
14.
15.
16.
17.

18.
19.
20.
21.
22.
23.
24.

25.

26.

27.

a. IXa-VIII- phospholipid- calcium


b. VIIIa-tisue factor-calcium
c. Prekallikrein, kallikrein, high molecular weight kininogen
d. Xa-Va- phospholipid- calcium
Factor XII can be activated by:
a. Exposure to subendothelium in vivo
c. A and B
b. Exposure to glass in vitro
d. Neither A nor B
The origin of platelets in the bone marrow is the:
a. Megakaryocytes
c. Hemohistioblast
b. Stem cell
d. Megaloblast
Low platelet count and normal bleeding time indicates:
a. Primary vascular abnormality
c. Qualitative platelet disorder
b. Von Willebrands syndrome
d. Autoimmune thrombocytopenia
The normal range of a direct platelet count is:
a. 50,000 to 100,000/ul
c. 140,000 to 440,000/ul
b. 75,000 to 125,000/ul
d. 300,000 to 600,000/ul
A bleeding tendency is likely to be present when the platelet count is below:
a. 50,000/ul
c. 200,000/ul
b. 150,000/ul
d. 400,000/ul
A quatitative defect of platelets is known as
a. Thrombocytopathia
c. Thromcythemia
b. Thrombocytopenia
d. Thrombocytosis
Thrombocytopenia is associated with a
a. Prolonged bleeding time
c. Abnormal prothrombin time
b. Prolonged lotting time
d. Abnormal APTT
In thrombocythemia, platelets are
a. Normal in number
c. Markedly increase
b. Normal in number but abnormal in
d. Markedly decrease
function
Classic haemophilia is the result of a deficiency of which coagulation factor?
a. VI
c. VIII
b. VII
d. IX
Factor VIII is present in highest concentration in
a. Fresh frozen plasma
c. Cryoprecipilate
b. Platelet concentrate
d. Whole blood
The Christmas is factor
a. VII
c. IX
b. VIII
d. X
A prolonged bleeding time with a normal platelet count indicates
a. Aspirin digestion
c. Von Willebrands syndrome
b. Vascular abnormality
d. AOTA
The test that uses a reagent form of tissue thromboplastin is
a. APTT
c. Prothrobin time
b. D- dimer assay
d. Euglobulin clot lysis
The phase contrast microscope is employed in which platelet count method?
a. Rees- Ecker
c. Indirect
b. Brecher-Cronkite
d. Coulter
If the prothrombin time is abnormal and the activated partial thromboplastin time (APTT) is abnormal , a
deficiency in which factor is suggested?
a. V
c. IX
b. VII
d. X
If the prothrombin time is normal and the activated partial thromboplastin time (APTT) is abnormal, a
deficiency of which factor is suggested?
a. V
c. IX
b. VII
d. X
If the bleeding time is normal, prothrombin time normal and the activated partial thromboplastin time (APTT)
is abnormal, a deficiency of which factor is suggested?
a. II
c. VII
b. V
d. VIII
The test that can monitor heparin therapy , increased levels of fibrin split products, and quantitative and
qualitive fibrinogen is
a. Fibrinogen assat
c. Thrombin time
b. Activated partial thromboplastin time
d. D-dinne

A1 PASSERS REVIEW CENTER 320-2728 //

64

28. Thea.
onlyProthrombin
test that is sensitive
time
to a deficiency of factor VII is
c. APTT
b. Thrombin time
d. Protamine sulphate
29. The activated partial thromboplastin time (APTT) serves as a screening procedure for which plasma factor
deficiencies?
a. VII and IX
c. All except VII and XIII
b. V and X
d. XI, XII and platelets
30. The activated partial thromboplastin time may be within normal limits with abnormality activity of
a. Platelets
c. Factor VIII
b. Factor I
d. Factor V
31. All of the following represent coagulation methodology; except
a. Optical
c. Immunological
b. Impedance
d. Chemical
32. If a coagulation test is prolonged due to a deficiency of a plasma coagulation factor, then the test should be
corrected by the addition of
a. Calcium
c. Vitamin K
b. Normal plasma
d. Platelets
33. 5M urea or 1% monochloracetic acid are reagents used in tests for which plasma factor deficiency?
a. VIII
c. XII
b. IX
d. XIII
34. Diagnosis of von Willebrands disease can be made from the following test results?
a. Decreased VIII:C
c. Prolonged bleeding time
b. Decreased VIIIR: RCo
d. AOTA
35. Which of the following factor of activations occur in the alternate pathway?
a. Factor Vllla, can activate IX and X
b. Factor IX and Kallikrein can activate factor VII
c. Factors Xlla, Xa, Ixa or thrombin can activate Vll
d. All of the above
36. Match the function will the following inhibitors
1.
_______ heparin
2.
_______AT-III
3.
_______protein C
4.
_______ protein S
5.
_______coumadin
6.
_______plasminogen
7.
_______streptokinase
8.
_______ lupus like anti coagulant
a.Degrades factors V and VIII
b.Inhibits vitamin K
c.Accelerates fractivation of factors V and VIII
d.Activates blasminogen for lysis of thrombi
e.Inactivates zymogen , binds fibrin at critical lysine binding sites
f. Principle inhibitor of thrombin and factor X
g.Greatly enhances activity of AT-III
h.Inactivates coagulation factors through phospholipoprotein
37. Fibrinogen has to decrease below what level to affect the various clotting tests?
a.20mg/dL
c.75mg/dL
b.50mg/dL
d.200mg/dL
38. Idiophatic thrombocytopenic purpura is usually characterized by
a.Increased platelet destruction
c.Normal platelet count
b.Increased megakaryocytes
d.Marrow infiltration (malignancy)
39. Disseminated intravascular coagulopathy (DIC) is associated with all of the following disorders, except
a.Haemophilia
d.Acute granulocytic leukemia or acute
b.Snake bites
monocytic leukemia
c.Placenta praevia
40. Laboratory results from a patient with disseminated intravascular coagulopathy (OIC)are:
a.Decreased platelets and factors I, V and VIII
b.Prolonged prothrombin time, activated partial thromboplastine time, and thrombin time
c.Positive D-dimer
d.All of the above
41. Ehlers Danlos, von Willebrand, and Bernard Soulier: are all qualitative platelet disorders that have
alnormalities of
a.Adhesion
c.Secondary aggregation
b.Primary aggregation
d.NOTA

A1 PASSERS REVIEW CENTER 320-2728 //

65

42. The qualitative platelet disorder that has abnormal primary aggregation is
a.Wiskott-Aldrich
c.Glanzmann
b.Gray platelet syndrome
d.May-Hegglin
43. Storage pool deficiency and Wiskott Aldrich are qualitative platelet disorders that have abnormal
a.Secondary aggregation
c.Primary aggregation
b.Adhesion
d.Other release mechanisms
44. Dilutions of patient and normal plasma with a prolonged prothrombin time and/or activated partial
thromboplastin time indicate a
a.Circulating anticoagulant
b.Deficiency or more than one plasma factor
c.Deficiency of a factor involved in both intrinsic and extrinsic coagulation pathways
d.Fibrinolysin

45.
46. IMMUNOHEMATOLOGY
47. Banking
1. The interval between blood donations is:
a. 6 weeks
c. 10 weeks
b. 8 weeks
d. 3 months
2. The oral temperature of a donor shall not:
a. 37.0 C
c. 98.6 F
b. 37.5 C
d. 98.0 F
3. The minimum acceptable haemoglobin for male donors is:
a. 12.5 g/dl
c. 13.5 g/dl
b. 13.0 g/dl
d. 14.0 g/dl
4. What is the lowest acceptable hematocrit for female blood donors?
a. 36%
c. 41%
b. 38%
d. $3%
5. Prospective donors who have malaria should be deffered:
a. Permanently
c. For two years cessation of treatment
b. For three years after cessation of
d. For six months
treatment
6. The name given to the infections virus of hepatitis B (HBV) is:
a. Complex virus
c. Circular DNA virus
b. Done particles
d. Infections particle
7. Symptom-free donors immunized with oral polio, measles (rubeola) or mumps vaccines are acceptance after
a periods of:
a. 24 hours
b. 2 days
c. 2 weeks
d. 24 days

A1 PASSERS REVIEW CENTER 320-2728 //

66

8. For autologous blood donations, blood should not be drawn from the donor-patient within ___ hours of the
time of the anticipated operation or transfusion
a. 12 hours
c. 48 hours
b. 24 hours
d. 72 hours
9. An autologous blood donor must:
a. Be over 21 years old
c. Have at least an 11 g/dl hemoglobin
b. Be treated for bacteria
d. Be on oral iron therapy
10. Which of the following pre transfusion test must be determined for an autologous transfusion recipient?
a. ABO and RH type
c. STS
b. HBs Ag
d. Compatibility testing
11. Which biochemical change does NOT occur in stored blood?
a. Plasma pH decreases
c. Plasma ammonia (NH2) increases
b. Plasma potassium (k) decreases
d. Inorganic phosphate increases
12. Which of the following organisms have been implicated in bacterial contamination of door blood?
a. Pseudomonas
c. Escherichia coli
b. Citobacter freundii
d. Bacillus species
13. The acceptance plasma that is stored at-18 C or lower has a shelf life of __.
a. 1-6 C
c. S-10 C
b. 1-10 C
d. 32-37 C
14. Fresh frozen plasma that is stored at 18 or lower has a shelf life of ___.
a. 6 months
c. 2 years
b. 1 year
d. 5 years
15. When a blood product is stored at 1-6C and the hermetic seal has been broken, the expiration date
becomes:
a. 24 hours
c. 48 hours
b. 6 hours
d. 8 hours
16. The optimum temperature for thawing fresh frozen plasma is:
a. -18 C
c. 30-37 C
b. -10 C
d. 58 C
17. According to DFA regulations, if blood is to be transported the blood temperature must be kept between:
a. 2-8 C
c. 1-0 C
b. 1-10 C
d. 4-6 C
18. Which blood group system was discovered first?
a. ABO
c. P
b. MNS
d. Rh
19. An amorph is gene:
a. That is expressed in the heterozygous state
b. That is expressed only in the homozygous state
c. With no directly observable product]
d. With an observable variable effect
20. Production of more antigen in the homozygous state than in the heterozygous state is reffered to as:
a. Dosage effect
c. Independent segregation
b. Antigenic determination
d. Prozone
21. Unexpected antibiotics that react 37C are:
a. Usually Igm antibiotics
c. Considered not clinically significant
b. Usually naturally occurring antibodies
d. Considered most clinically significant
22. Naturally accurring antibodies are found regularly accuring antibodies:
a. ABO
c. Kell
b. Rh
d. Kid
23. Select yhe INCORRECT statement below. Naturally antibodies:
a. React best in saline
c. Do not usually cross the placenta
b. React best at room temperature or 4C
d. Are usually IgG
24. Immune antibodies may be produced by:
a. Injection of purified blood group
c. Repeated blood transfusions
substances
d. AOTA
b. Pregnancy
25. Antyi-A1 is a seed extract from:
a. Ulex europeus
c. Arachia hypogeal
b. Vicia graminea
d. Dolichos biflocus
26. The frequency of group A individuals in the United states is:
a. 4%
b. 9%
c. 41%
d. 46%
27. Group B individuals have:
a. A antigens on their red cells
b. Anti-A in their serum

A1 PASSERS REVIEW CENTER 320-2728 //

67

c. Neither Anti-A nor Anti-B in serum


d. Anti-B in their serum
28. Red blood cells of which blood group will not be agglutinated by anti-A,B typing serum?
a. A
b. B
c. AB
29. Red blood cells of which blood group react most strongly with anti-H?
a. AB
c. O
b. A1
d. B
30. The reagent made from the seeds of dolichos bi-florus is:
a. Anti-A1
c. Anti-B
b. Anti-A,B
d. Anti-H
31. The antigen most commonly tested for i the Rh system is:
a. E
c. D
b. C
d. O
32. Which of the following is true of D?
a. D cannot be inherited
b. D is a variant of the Rho (D) antigen
c. D cells should not be considered Rho (D) negative when used donor blood
d. D is common in Caucasians
33. Which is not TRUE of D antigens?
a. D antigens are fairly common in blanks
b. D cells usually appear to be negative or weakly reactive at initial testing
c. D cells usually be incupated at 4C for one our
d. D Cells are incubated at 37C, then tested by the anti-globulin procedure
34. Anti-C will not react with:
a. Rh 1 Rh2 cells
c. Rh cells
b. Rh1 Rh1 cells
d. Rh rh cells
35. Anti-E will react with:
a. Rh cells
c. Rho rh cells
b. Rh1 rh cells
d. Rh2 Rh2 cells
36. Anti-c can be formed by persons with the genotype:
a. R1 R2
c. R2r
b. R1 R1
d. Rr
37. Anti-hr will NOT react with:
a. Rh cells
c. Rh1 Rh1 cells
b. Rh1 rh cells
d. Rh2 Rh2 cells
38. Anti-hr will react with:
a. Rhz Rh2 cells
c. Rh2 Rh2 cells
b. Rh1 Rh1 cells
d. Rhz Rhz cells
39. The f antigen refers to:
a. Ce
c. CE
b. cE
d. Ce
40. The symbol for the Bombay blood group is:
a. Oh
c. Ho
b. OH
d. HO
41. All but one of the following antibodies will be detected in the serum of Bombay genotype:
a. Anti-A
c. Anti-H
b. Anti-B
d. Anti-Oh
42. The system composed of antigens found primarily in saliva and plasma is:
a. Lutheran
c. P
b. Lewis
d. Rh
43. In the duffy blood group system, the most common antibody detected is:
a. Anti-Fya
c. AnAnti-fy4
b. Anti-Fyb
d. antiFy5
44. Which of the following usually Igg, immune and anti-globulin (coo-bs) reactive?
a. Anti-le a
c. Anti-l
b. Anti-jk b
d. Anti-n
45. The antigen I is:
a. Weakly reactive with anti-I at 4C
c. A rare antigen
b. Poorly developed on cord red blood
d. Not found in blanks
cells
46. Anti-I is most commonly detected in which cross match phase?
a. Room temperature
c. Coombs phase
b. 37 c
d. AOTA
47. Select the incorrect statement:

A1 PASSERS REVIEW CENTER 320-2728 //

d. O

68

a.
b.
c.
d.

48.

49.
50.

51.
52.
53.
54.
55.
56.
57.
58.
59.
60.
61.
62.
63.

66.
67.

Anti I is often the causative antibody in case of cold hemaglutinin disease (CHAD)
Anti-I reacts best with cord blood cells
Anti-I can be found as weak cold agglutinin in the serum of normal individuals
Anti-I agglutinin are usually cli0nically insignificant, however, they may mask more significant
antibodies, such as anti-P1, anti-M and anti-le
Which of the following is NOT TRUE of Anti-Le?
a. Occurs in Le (A-b) individuals
b. Is commonly implicated in hemolytic disease of the newborn (HDN)
c. Is neutralized by lewis blood group substance
d. S composed mostly of IgM molecules
What percentage of the population lacks the kell (K) antigen?
a. 0.02%
c. !0.00%
b. 2.00%
d. 90.00%
Select the INCORRECT stamen:
a. 90% of the population is kell (K) negative
b. Anti-K reacts best in saline at room temperature
c. Anti-K can cause hemolytic disease of the newborn (HDN)
d. Next to Rho (d0, the kell (K) antigen is the most immunogenic
Anti-k ia also known as:
a. Anti-k1
c. Anti-K
b. Anti-Mclead
d. Anti-cellano
Approximately what percentages of blacks have the Fy (a-b) phenotype?
a. 1%
c. 22%
b. 9%
d. ^68%%
Recent finding suggest there is an association between the Fy (a-b) phenotype and resistance to:
a. Chronic granulomatous disease
c. Syphilis
b. Malaria
d. Viral hepatitis
The antigen Tj is part of which system?
a. Rh
c. Kidd
b. MNS
d. P
Approximately what percentage of males are Xg positive?
a. 11.3%
c. 65.6%
b. 36.0%
d. 88.0%
Approximately what percentage of females are Xg positive>
a. 11.3%
c. 65.6%
b. 36.0%
d. 88.0%
When color-coding is used for donor blood labels, group A is;
a. Pink
c. Yellow
b. Blue
d. Black
What is the standard acceptable color for anti-A grouping serum?
a. Blue-green
c. Red
b. Colories
d. Yellow
Polyspecific antiglobulin serum is sometimes colored:
a. Blue
c. Green
b. Yellow
d. Pink
What is the color of the dye added to commercially prepared anti-B blood grouping serum?
a. Yellow
c. Green
b. Blue
d. NOTA
Anti-A, B serum:
a. Is used to confirm group O individuals
c. Is obtained from A2B individuals
b. Is obtained from A1B individuals
d. Agglutinates group O cells
Which of the following is NOT a suitable medium for suspensions of red cells for blood banking?
a. Saline
c. Albumin
b. Serum
d. Distilled water
Which of the following enzymes is NOT used in blood bank procedures?
a. Ficin
c. Bromelin
b. Amylase
d. Papain
64.
65.
Enzymes prevent detection of antibodies in the:
a. Kidd system
c. Lewis system
b. Duffy system
d. Rh system
The technique used to remove antibody bound to sensitized red cells is called:

A1 PASSERS REVIEW CENTER 320-2728 //

69

68.
69.
70.

71.
72.
73.
74.

75.
76.
77.

78.
79.
80.

81.
82.
83.
84.
85.

a. Absorption
c. Titer
b. Flution
d. Tranlocation
A cold autoagglutinin will usually have specificity for the:
a. D antigen
c. L antigen
b. E antigen
d. K antigen
Paroxysmal cold hemoglobinuria is often associated is which system?
a. MNS
c. Lewis
b. P
d. Rh
Forward grouping is using:
a. Reagen A1 and B cells to detect serum anti-A and Anti-B
b. Anti-A anti-R antisera to detect cell antigers A and/or B
c. Anti-A,B to confirm group O
d. Using anti-A1 to detect subgroups of A
Reverse grouping difficulties may be encountered in the following instances:
a. Newborn infants
c. Patients undergoing chemotherapy
b. Geriatic patients
d. AOTA
A patient that forward group an AB but reserve group as group B probably is a group A2 B with:
a. Anti-A1
c. Anti-B
b. Anti-A2
d. Anti-D
The most dangerous and unexpected antibodies in the blood bank are those that react at:
a. 4C
c. 32C
b. 25C
d. 37C
Antiglobulin reagent:
a. May be produced in laboratory animals
c. Never detects complement-dependent
b. Is produced in human beings
antibodies
d. Occurs naturally is most human beings
Which of the following drug will NOT cause a positive direct antiglobulin (Coombs) test?
a. Penicilin
c. Cephalothin
b. Alpha-methydopa
d. Aspirin
What is the optimum incubation temperature for the indirect antiglobulin (IAI) test?
a. 4C
c. 37C
b. 22C
d. 56C
The direct anti-globulin test (DAT) using poly-specific anti-hurian globulin serum is NOT of value in the
a. Diagnosis of the hemolytic disease of the newborn
b. Diagnosis of hemolytic transfusion reactions
c. Diagnosis of drug-induced hemolytic anemia
d. Differentiation of cell surface coating (complement or (IgH)
Albumim-reactive Igg antibodies are about __in length.
a. 1000 angstroms
c. 500 angstroms
b. 600 angstroms
d. 250 angstroms
By common usage, a poly specific anti-human serum at least contains antibodies to:
a. IgG and C4
c. IgG, Iga and C3d
b. IgG and Ig
d. IgG and c3d
The direct anti-globulin test is most useful for:
a. Detection of hemolytic disease of the
c. Leuloagglutinin assays
newborn
d. Detection of unexpected antibodies
b. Compatibility testing
In a direct anti-globulin tests, you are testing patient:
a. Cells
c. Cells against donor serum
b. Serum
d. Serum against donor cells
An direct anti-globulin test is incubated for:
a. 15-30 minutes at 25 C
c. 5-10 minutes at 4C
b. 15-60 minutes at 37 C
d. 15-60 minutes at 56C
The red blood cells used for screening patients serum for unexpected antibodies s hold be of what group?
a. Group O
c. Group B
b. Group A
d. Group AB
A false positive anti-globulin test ca n be caused by :
a. Inadequate w2ashing
c. Elution due to delayed washing
b. Prozone r6eaction
d. Bacterial contamination of reagents
A false negative indirect anti-globulin test can be caused by:
a. Test cells being direct coombs positive
c. Inadequate washing
b. Bacterial contamination of test cells
d. Chemical contamination

A1 PASSERS REVIEW CENTER 320-2728 //

70

86. Laboratories must check each negative anti-globulin test using red blood cells sensitized with IgG. After the
addition of these cells. Which of the following is NOT necessarily true?
a. Active anti-globulin serum was added
b. Patient serum was added
c. The washing phase as performed satisfactorily
d. Anti-globulin serum has not reacted with the original red being tested
87. Fresh serum (less than 48 hours old must be used for compatibility testing to preserve:
a. Chelating agents
c. Complement
b. Siatic acid
d. Auto-antibodies
88. Mixed field agglutination is observed under the microscope as clumps or agglutinates of cell among many
free CELLS. Which of the following is the most common reason for mixed field agglutination?
a. Twin chimetras
d. Antigen strength altered in disease
b. Transfused
states
c. Weak subgroups of A or B
89. The term type and screen refers to:
a. Typing the patients blood for rate antigens
b. Screening donor units for rare antigens
c. Testing the patients blood for ABO and Rh type and unexpected antibodies
d. Identification of unexpected antibodies in the patient blood
90. All but one of the following antibodies will usually be detected in the room temperature phase of cross
match;
a. Anti-Jk
c. Anti-M
b. Anti-Le
d. Anti-P1
91. A positive antigen-antibody reaction is indicated by:
a. Antibody formation
c. Agglutination or hemolysis
b. Antigen formation
d. No agglutination or hemolysis
92. Which of the following antibodies usually gives negative reactions in enzyme test procedures?
a. Anti-Jk
c. Anti-D
b. Anti-Fy
d. Anti Le
93. One of then eight units cross matched is incompatible in the anti-globulin phase of testing. The most likely
antibody is:
a. Anti-K
c. Anti-Jk
b. Anti-fy
d. Anti-k
94. If a recipient is incompatible wiit5h only one donor, the most probable cause is:
a. A re5ciept antibody to a high incidence antigen
b. A recipient antibody to a low incidence antigen
c. A clerical error
d. Recipient serum was not added
95. Hemolysis requires complement activation. Which of the following are required for complement activation?
a. Calcium and magnesium
c. Calcium and sodium
b. Calcium and potassium
d. Sodium and potassium
96. The initial response to a foreign antigen is known as the primary response: His primary response usually
takes how long occur?
a. It never occurs
c. 2-6 months
b. 10-12 months
d. 8-10 months
97. Secondary response to the same foreign antigen is known as:
a. Anamnestic response
c. Natural response
b. IgG response
d. Ab response
98. Plateit concentrates are most likely to be benefit in;
a. Hemolytic disease of the newborn
c. Autoimmune hemolytic anemia
b. Thrombocytopenia
d. Multiple myeloma
99. Cryoprecipitated anti-hemophillic factor (AHF) is not recommended for the treatment of:
a. Hemophilia A
c. Haemophilia B
b. Von Willebrand disease
d. Hypofibrinogenemia
100.
Red blood cells are the product of c hoice for:
a. Supplying deficient coagulation factors
c. Exchange transfusion
b. Thrombocytopenia
d. Increasing oxygen carrying capacity
101.
Deglycerolized frozen red blood cells must be transferred w2thin how many hours?
a. 4
c. 24
b. 21
d. 72
102. Recurrent nonhemolytic febrile transfusion reactions are usually caused by all of the following except:
a. Anti-HLA antibodies
c. Red cell antibodies
b. Platelet antibodies
d. White cell antibodies

A1 PASSERS REVIEW CENTER 320-2728 //

71

103. A patient is suspected of having a transfusion reaction, When should the transfusion be stopped/
a. After the unit is infused
c. After the physician has been notified
b. Immediately
d. After notification of the laboratory
104. A transfusion reaction is considered febrile when the patient experiences a:
a. 2 degree F rise in temperature within 1 hour transfusion
b. 2 degree F drop in temperature within 1 hour of transfusion
c. No temperature increase at the start of the transfusion
105. The blood phenotype O can result from which of the following genotypes?
a. AC
c. OO
b. BO
d. A2O
106. The cells and serum of a neonate were tested at room temperature.
107.
Patient cell with:
Patient serum with:
108.
Anti-A negative
A1 cells negative
109.
Anti-B- positive
b cells - negative
110.
111.
The infant is probably:
a. Group B
b. Group AB
c. Group O
d. NOTA
112.
Patient cells with
Patient serum with:
113.
114.
Anti-A positive
a1 cells positive
115.
Anti-b positive
b cells - negative
116.
Anti-AB positive
117.
Anti-A1 negative
118.
119.
Give the most likely explanation for the discrepancy in the forward and reverse grouping:
a. The patient is group A1B with autoantibody
b. The patient is group A2B with anti-A1
c. The patient is group B with an acquired A-like antigen
d. The patient is group A2b with anti-h
120. What is the optimum concentration of a cell suspension to be used in blood bank testing?
a. 1-2%
c. 8-10%
b. 2-5%
d. 10-15%
121. The weiner genotype R2 R2 is equivalent to:
a. CDE/CDe
c. cDE/cDE
b. Cde/cde
d. cde/cdE
122. What is the most common genotype that could result from the phenotype D+ C- C+ e-?
a. R2R2
c. Rr
b. R1R1
d. R1r
123. Given the reactions, D+ c+ E+-c+ e+, what is the most probable genotype?
a. R1R1
c. R1r
b. R1Ro
d. R1R2
124. Given the reactions, D+ C+ E+ e+, what is the most probable genotype?
a. R1R2
c. R2r
b. R1r
d. R2Ro
125. If cells are positive when tested with anti-D, Anti-C and anti-e and negative when tested with anti E and
anti c, what is the most likely genotype?
a. R2R2
c. R2r
b. R1r
d. R1R1
126. If red cells give positive reactions when tested with anti-Rho, anti-hr, anti-hr, and a, negative reaction with
anti-rh, which of the following genotypes is IMPOSSIBLE?
a. R1R
c. R1Ro
b. R1r2
d. Ror
127. 128. What is the most common of the Rh NEGATIVE GENOTYPES?
a. cde/cde
c. Cde/CdE
b. Cde/cde
d. cdE/cde
129. When red cells are positive when tested with anti-e and negative when tested with anti-D, Anti-c and anti-E,
what is the genotype?
a. Rr
c. Ro Ro
b. R1r
d. R1 r1

A1 PASSERS REVIEW CENTER 320-2728 //

72

130. D antigens
a. Blacksare fairly common to what race?
c. Hispanics
b. Caucasians
d. Asians
131. The d antigen is responsible for the most frequent production of unexpected antibodies. Which other blood
group antigen is responsible for a large percentage of unexpected
a O antigen
c K antigen
b A antigen
d B antigen
2 Hemolytic disease of the newborn can occur when:
a The infant lacks the antigen the mother possesses
b The mother lacks an antigen the infant possesses
c The mother possesses an antigen the father lacks
d NOTA
3 Which antibody is most likely to be involved in hemolytic disease of the newborn?
a Anti-c
c Anti-m
b Anti-le
d Anti-Js
4 Whwn severe hemolytic disease ofv the newborn is due to an unidentified antribody:
a No blood ids acceptable or transfusion
b AB negative red cells an O negative plasma should be used for transfusion
c Mothers red cells may be used for transfusion
d NOTA
5 Of the following blood groups, Which one does not usually cause homolytic disease of the newborn?
a Rh
c P
b Kell
d Duffy
6 In a case of hemolytic disease of the newborn, when the mothers is group O and the baby is group a, red
cells for exchange transfusion should be what group?
a A
c O
b B
d AB
7 In case of ABO hemolytic disease of the newborn, the mother is usually which of the following?
a Group AB
c Rho (D0 negative
b Group O
d A
8 If a mother passes an antibody to her fat us through the placenta, what type of immunity does the fat us
have?
a Naturally acquired active
c Naturally acquired passive
b Artificially acquired active
d Artificially acquired passive
9 If the mother is group A1B and the father is group A1b, which would be IMPOSSIBLWE in the offspring?
a A1
c B
b A2
d A2B
10 In the phenotype mating A2B x O, which of the following phenotypes is possible in the offspring?
a A1
c O
b A2B
d B
11 In the phenotype mating O x O, which of the following phenotypes is possible in the offspring?
a A1
c B
b A2B
d O
12 An A1B mother and an A2O father could NOT produce which of the following genotypes?
a A1A2
b A2B
c BO
d A2O

A1 PASSERS REVIEW CENTER 320-2728 //

73

13 If the mother is group O and the infant is group B, the infants blood group genotypes is:
a BB
c BO
b AO
d AB
14 Given a mother of group O and an infant of group B, which of the following blood group would eliminate
paternity?
a AB
c O
b B
d Paternity could not be ruled out
15 If a group AB mated with a group O, the following genotypes will result:
a AO and Bo
c AB and OO
b AA and BB
d AO and BB
16 In the mating of R1rxR2r genotypes, which of the following is probably NOT that of the offspring?
a CDe/cDE
c CDe/cde
b cde/cde
d CDe/cdE
17 If an R2R2 patient is transfused with R1R1 blood, it would be possible for him to form which of the following
antibodies?
a Anti-E
c Anti-ce
b Anti-Ce
d Anti-c
18 If a patient requiring a blood transfusion has an anti-e (hr) approximately what percent of donor blood tested
would be compatible?
a 2%
c 20%
b 15%
d 80%
19 Approximately what percentage of donor blood is compatible with a patient who has anti-Jk?
a 25%
c 75%
b 40%
d 90%
20 You must find two units f compatible blood for a patient who has anti-JK. How many units will you cross
match?
a 5
c 20
b 10
d 50
21 When a patient is to undergo heart by pass surgery, which antibody is considered clinically significant?
a Anti-A
c Anti-Le
b Anti-b
d Anti-P1

23
24
25
26
27
28

29

31

33

a Kidd
c Lewis
b Lutheran
d Xg
In titration of an antibody, the first 2-3 tubes are negative and later tubes are positive. This is known as:
a Rouleaux effect
c Antigen effect
b Antibody effect
d Prozone effect
Intravascular hemolysis occurs when:
a Complement is activated
c Calcium is activated
b Complement is inactivated
d Calcium is inactivated
What is the optimum pH at which antibodies bind o cells?
a 5.5-6.0
c 6.0-7.0
b 7.8-8.5
d 6.5-7.5
A blood bank has 10 nits of O Rho (D) positive blood from random donors. It should be4 easiest to find two
units of compatible blood for the patient with:
a Anti-e
b Anti-c
c Anti-k
d Anti-Le
If the patient with anti-E needs a blood transfusion, approximately what percentage of donor blood tested will
be compatible?
a 2%
b 305
c 70%
d 98%
A patient needing a blood transfusion is found to have anti-P1. According to the statistics, select the
appropriate number of compatible units:
a 1 of 4 units
c 2 of 3 units
b 1 of 10 units
d 3 of 4 units
A 56 C water bath is commonly used for:
30
1. isotonic saline 2.preparing elautos 3.inactivating sera 4. Thawing frozen cells
a 1 and 4
c 1,2 and 3
b 2 and 3
d 1,2,3 and 4
Testing media for antigen-antibody reactions include:
32
1. isotonic saline
2. LISS
3. Albumin
4.Enzymes
a 1 and 3
c 1,2 and 3
b 2 and 4
d 1,2.3 and 4
Anti-a1 agglutinin is:
34
1. found in group A1 patients
3.Found in all A2 b patients

A1 PASSERS REVIEW CENTER 320-2728 //

74

35
36

37

40

45

50

55

56

57

58

2. occasionally found in A2 patients


4. Found in group B and O patients
a 1 and 3
c 1,2 and 3
b 2 and 4
d 1,2,3 and 4
Standardize techniques for blood banking procedures by:
a Grading reactions consistently
b Timing test accurately
c Making accurate cell suspensions and controlling size of drops
d AOTA
Anti-ab serum:
38
1. Is obtained from A1B individuals
3. Is obtained from group O individual
39
2. Is obtained from A2B individuals
4. Confirms group o individuals
a 1 and 2
c 1,2 and 3
b 3 and 4
d 1,2,3 and 4
Rh antibodies:
41 1. React more strongly at 37C than at 4C
42 2. Frequency noted to be cold agglutinins
43 3. Is obtained acquired fr. Transfusion or pregnancy
44 4. Confirms group O individuals
a 1 and 3
c 1,3 and 4
b 2 and 4
d 1,2,3 and 4
Which of the following are naturally occurring?
46 1. anti P1
47 2. Anti- Le
48 3. Anti N
49 4. Anti-S
a 1 and 3
c 1,3 and 4
b 2 and 4
d 1,2,3 and 4
The group B phenotype can result from the following genotype/s?
51 1. A1B
52 2. BBB
53 3. AO
54 4. BO
a 1 and 3
c 1,3 and 4
b 2 and 4
d 1,2,3 and 4
The phenotype A can result from which of the following genotypes?
a A1A2
d A2O
b A2A2
e A1B
c A1O
In the every acute phase of hepatitis, which of the following would be indicative of the onset of a viremic
state?
a Positive HBsAG
c Positive anti-HBc
b Positive HBeAg
d Anti-HAV
What is the expirations date (In days) of red blood cells preserved with citrate phosphate- dextroseadenine (CPDA-1)?
a 2;
c 35
b 24
d 40
Low ionic strength solution (LISS):
59 1. Causes of increase in the electropositive cations surrounding the red cells
60 2. Causes decrease in the electropositive cations surrounding the red cells
61 3. Usually consists of phosphate saline and glycine
62 4. Enhances the rate and sensitivity of the antigen-antibody reactions
a 1 and 3
c 2,3 and 4
b 2 and 4
d 1,2,3,and 4

e
f SUPERVISION AND MANAGEMENT
1. Which a new employee begins orientation, the employers skills and abilities should be permanently
documents in the:
a.Personnel handbook
c.Personnel file
b.Orientation checklist
d.None of the above
2. The orientation process for a new employee includes all of the following EXCEPT:
a.A review of duties and responsibilities of the job
b.A review of personnel policies
c.The location of general use facilities
A1 PASSERS REVIEW CENTER 320-2728 //

75

d.A request for personal information on past facilities


3. During the orientation period for a new employee, all of the following actions should be performed EXCEPT:
a.Introduction to laboratory staff
b.Introduction to department management
c.Introduction to reference laboratory technical support associate
d.Introduction and identification of supervisory personnel
4. An orientation to the laboratory work area includes of the following Except:
a.A review of all tests the new employee will be expected to perform
b.The techniques and procedures employed by technical biomedical service engineers
c.Routine Preventative maintenance and troubleshooting of instruments
d.Necessary responses to grossly abnormal patient test results
5.
6.
7. All new employees should be introduced to the proper procedures for all of the following EXCEPT:
a.Use and care of personnel identification card
b.Proper use and documentation on time cards
c.Personnel reputations for time cards
d.Calculating time card hours
8. We I designed job description does NOT include which of the following?
a.The duties of the employee for the job classification
b.The minimal qualifications of the employee
c.The pay scale for the job
d.Job responsibilities and degree of supervision
9. Personnel evaluations should be conducted at least:
a.Weekly
c.Quarterly
b.Monthly
d.Annually
10. Employee personnel records should include all of the following EXCEPT:
a.Records or affidavits of civil or criminal
c.Reprimands and commendations
arrests
d.Copies of certification and credentials
b.An employees job descriptions
11. A continuing education unit (CEU) is defined as:
a.One hour
c.Ten contact hours
b.One contact hour
d.Twenty hours
12. In-service and continuing education programs for the employee should be conducted at least:
a.Weekly
c.Monthly
b.Bi-weekly
d.Annually
13. The determination of the type of services offered by the laboratory must take into consideration all of the
following EXCEPT:
a.The request of the patients
b.Usefulness of procedures for the type of patients treated
c.Needs of the medical staff
d.The reasonable cost-to-change relationship of the procedure
14. Which of the following would cause the most concern in limiting test specimen processing?
a.A temporary shortage of staffing
b.A minimum supply of reagents
c.A lack of qualified personnel to perform analysis
d.The temporary absence of a technical supervisor
15. Infection control policies and procedures should include all of the following EXCEPT:
a.Personnel protective equipment and clothing to be used
b.Engineering control procedures
c.Work practice controls
d.Cultures to be taken
16. Emergency Evacuation routes and procedures for the laboratory should be posted:
a.In the laboratory office
c.Not posted but in the safety manual
b.In the laboratory sections
d.At every entrance and exit
17. Result that are grossly abnormal should be reviewed by one or all of the following EXCEPT:
a.General laboratory supervisor
c.Physician ordering the test
b.Medical director of the laboratory
d.Section supervisor
18. In the performance evaluation, the parties involved should consider all of the following EXCEPT:
a.Goals and obligations identified
b.Review of workers actual performance during the period
c.Criteria used to evaluate performance
d.Management system to evaluate performance

A1 PASSERS REVIEW CENTER 320-2728 //

76

19. A well maintained personnel file for an employee should contain all of the following EXCEPT:
a.Job description of employee
b.Written reprimands and commendations
c.Salary and benefits records
d.Information regarding the personal life of the employee
20. Test results should be re5ported:
a.At specific time periods during the shift
b.When requested
c.Within the specified time defined by the laboratory
d.In a clinically useful time period
21. All emergency (STAT) laboratory analysis should be reported to the ordering physician, or designate, within
a.0.5 to 1 hour
c.1-2 hours
b.10-20 minutes
d.3 hours
22. All laboratory testing performed by another laboratory and reported by the originating laboratory must
include, on the report form, which one of the following?
a.The initials of the person collecting the specimen
b.The name and address of the laboratory performing the analysis
c.The type of specimen submit7ted tio the reference laboratory
d.The signature of the technical supervisor of the originating laboratory
23. A properly labelled specimen will include all of the following EXCEPT:
a.Name of patient
c.Date of collection
b.Unique identification number
d.Date of birth of patient
24. Labels that must be affixed to reagents of materials prepared by the laboratory include all of the following
EXCEPT:
a.Hazards And precautions
c.Recommended storage
b.Concentrations of reagents
d.Amount of reagent used in a procedure
25. Those who utilize laboratory services for patient testing should be provided with information pertinent to all
of the following EXCEPT:
a.Proper methods of specimen collection
b.Care necessary in collection and transport of specimen
c.Special times of collection and preparation of patient
d.Methodologies employed in the performance of testing
26. Laboratory specimens require proper labelling for positive patient identification throughout the processing of
the specimen. Identify the one unnecessary items of information from the list below:
a.Patients age
c.Date of specimen collection
b.Full name of patient
d.Unique patient identification number
27. To assure proper test performance all of the following must be available to the technical staff in the
immediate area of testing EXCEPT:
a.Specimen collecting and handling requirement
b.A methodology for the performance of the procedure
c.Proficiency report of previous inter laboratory evaluations
d.Necessary quality control materials to be utilized
28. Purchased reagents and diagnostic kits must be verified for intented reactivity:
a.Before being used for patient testing
b.When running out of old materials
c.Within 2 days of beginning use of new materials
29. The title page of a policy or procedure manual should include all of the following EXCEPT:
a.Effective date of manual
c.Signature of individual approving the
b.Date of last revision
manual
d.Index of manual
30. Each standard operating procedure must be reviewed and date by the laboratory director:
a.Weekly
c.Yearly or whenever changes are made
b.Yearly
d.None of the above
31. The standard operating procedure manual should address four primary areas. These areas are:
a.Materials, supplies, equipment, storage
b.Equipment, reporting results, technical methods, materials
c.Supplies, instrumentation, equipment, materials
d.Reporting results, reagents, clinical relevance, reference services
32. Quality assurance studies should be conducted on a regular basis
a.To identify problems and make corrections
b.To point a finger at poor performance by testing personnel
c.Determine competency of management of the laboratory
d.So that the laboratory will comply with CLIA regulations

A1 PASSERS REVIEW CENTER 320-2728 //

77

33. Management role in providing for a safe working environment includes all of the following EXCEPT:
a.Adequate orientation of new employees in safety practices
b.Annual safety training
c.Providing incentive for employees to use safety equipment
d.Maintenance of all safety equipment in good working order
34. Reagents that require storage between 2-8 c and are photo reactive should be stored in a:
a.Refrigerator in a dark bottle
c.Dark laboratory cabinet sealed from light
b.Refrigerator in a clear bottle
d.Laboratory closet in a cool location
35. Which of the following is NOT associated with improper specimen sampling?
a.Incorrect specimen collection vessels
c.Cleansing of the collection site
b.Inappropriate venipuncture site
d.Labelling of specimen with name only
36.
37.
38. Venipuncture site prepared for the purpose of obtaining ethanol levels should be cleansed with:
a.Acqueous marthiolate solution
c.Ethano9l
b.Methanol
d.Betadine
39. Daily instrument preparation and evaluation should include all of the following EXCEPT:
a.Evaluation of electronic parameters
b.Quality control of test procedures
c.Dummy runs to certify operation
d.Certification of operation within manufactures specifications
40. Food and drink may be stored in a laboratory refrigerator if:
a.Placed in a special sealed container
b.Marked as a consumable material
c.Food and drink may not be placed in a laboratory refrigerator
d.Stored for short periods of time
41. Laboratory fire extinguishers should be inspected at least:
a.Annually
c.Weekly
b.Monthly
d.Quarterly
42. Laboratory safety manuals should be available in:
a.The laboratory office only
b.The pathologists office only
c.All work areas of the laboratory
d.In the administration office and dangerous laboratory areas
43. Flammable liquids may be stored:
a.In an ordinary refrigerator with a flammable storage label affixed
b.In any refrigerator within the laboratory department
c.In an explosion-proof refrigerator
d.Only in an explosion-proof refrigerator located in a remote area
44. Laboratory supplies should NOT be stored:
a.In cabinets and drawers
c.On the floor
b.At floor level if placed upon a raised
d.On shelving
platform
45. The national fire protection association hazard, warning emblem is a three colored square label. The three
colors are red, and yellow. Within each color block is a numeral identifying the severity of the hazard
identified by the color. The color red identifies fire hazards, yellow indentifies instability of the product, and
blue identifies what type of hazard?
a.Radiation
c.Poison
b.Health
d.Acid
46. Managements role in the administration of the clinical laboratory requires daily efforts in four areas. These
four areas are planning, organizing, activating, and:
a.Reporting
c.Communication
b.Supervising
d.Evaluating
47. Directives from supervisory personnel must be:
a.Clear and concise
c.Reasonable
b.Applied with regard to the demands of
d.All of the above
the situation
48. The purchasing of laboratory supplies must be monitored through the evaluation and verification of three
important document to ascertain that the laboratory received proper products and paid the appropriate price.
These three documents are
a.Purchase order, instruction manual, packing slip
b.Invoice, packing slip, instrument manual
c.Purchase order, packing slip, invoice

A1 PASSERS REVIEW CENTER 320-2728 //

78

d.Packing slip, bill of lading, price list


49. In order to participate in the medicare program, a laboratory must meet all applicable
a.Federal laws and regulations
b.State laws and regulations
c.Federal and state laws and regulations
d.Federal of state laws and regulations
50. A laboratory must be licensed under the clinic laboratory improvement act (CLIS) if it
a.Only collects specimens
c.Performs electro cardio grams
b.Operates in mexico and Canada
d.Engages in interstate commerce
51. A hospital clinical laboratory that performs medicare work must be:
a.Accredited by JCAH
c.Approved by medicare
b.Accredited by CAP
d.A member of AHA
52.
53. Proficiency testing should consist of
a.Testing and evaluating individuals workers
b.Internal external quality control testing sample
c.Sending sample to a reference lab for analysis
d.Checking all reagents for performance
54. If a result on a single control had a result outside of standard deviations, you should
a.Repeat the run
c.Use a new standard
b.Check the reagents
d.Check the machine for a detect
55. An explosion could result if
a.Acid added to water
c.Acid is neutralized and water added
b.Water is premixed with a weak salt
d.Water is added to concentrated acid
56. An excellent general laboratory cleanser that is also a disinfectant is
a.0.85%
c.Hypochlorite
b.Tincture of iodine
d.Tincture of merthiolate
57. All accidents should have a written report made within
a.The time the accident victim is
c.24 hours
hospitalized
d.30 days
b.The week is occurred
58. One of the best preventive measured for personal safety is
a.Avoiding work areas
b.Frequent hand washing
c.Avoiding all incoming specimens that are marked CAUTION
d.Wearing protective clothing at all times
59. An laboratory procedures commonly practiced for years that must be avoided in to days laboratory is
a.Pouring chemicals from stock bottles
c.Use of nonbiodegradable chemicals
b.Use of Bunsen burners
d.Pipetting by mouth
60. The only avenue of access to a patients is laboratory record from an outside source without the patients
written permission is
a.A lawyer request
c.Some from of judicial order
b.A pathologist request
d.A hospitals request
61. Which biological samples should be regarded as potential hazards?
a.Urine specimens from AIDS patients
d.Sputum specimens from tubercular
b.Stool specimens
patients
c.All biological samples
62. Materials used in blood collecting should be disposed of by
a.Placing in acid for 24 hours
b.Boiling for 15 minutes in saline
c.Placing in a potassium dichromatic solution for 24 hours
d.Placing in a bag and autoclaving
63. Which of the following does not appear on the specimen label?
a.The name of the patient
c.The name of the requesting physician
b.A tentative diagnosis
d.The date of collection
64. If an automated test instrument should be temporarily inoperable, samples for testing should be
a.Discarded and fresh ones obtained when the instrument is operable
b.Kept 27C until the instrument is operate
c.Analyzed by alternate method or properly stored until the instrument is operable
d.Stored in the refrigerator until the instrument is operate
65. Chemical waste is best discarded by
a.Burying contaminated materials in an isolated area
b.Burning the waste in a laboratory incinerator

A1 PASSERS REVIEW CENTER 320-2728 //

79

66.
67.

68.

70.
71.

72.
73.
74.

75.

76.

77.

78.

79.

80.

c.Neutralizing , diluting and then burying or incinerating


d.Dissolving in acid and burying the results
Laboratory specimens should be regarded as
a.Routine unless otherwise noted
c.Routine with no precautions
b.Non-infectious unless noted
d.Infectious
Accumulated radiation exposure for clinical laboratory personnel should not exceed how many roentgen
equivalents in humans (rem) per quaterter?
a.15
c.10
b.3
d.7
A centrifuge should be checked with a tachometer every
a.3 months
c.9 months
b.6 months
d.Annually
69.
The degree of reproducibility among a series of laboratory determinations on the same specimen is called
a.Accuracy
c.Sensitivity
b.Precision
d.Specificity
A precision run has a mean (X) of 231 and standard deviation of 5. The patient coefficient of variation is
a.0.0216
b.21.6000
c.2.1600
d.46.2000
The disposal procedure for radioactive material (1125) used in the laboratory is as follows
a.Follow manufacturers instruction
c. Pack well and place in trach can
b.Place in a special radioactive disposal can
d.Dispose with copious amounts of water
On method to check the accuracy of a pipellor is to deliver multiple volumes into a
a.Graduated centrifuge tube
c.Class A volumentric flask
b.Graduated cylinder
d.All of the above
What is a pure, stable chemical substance of a definite composition of which measured amounts are used to
establish a point of reference against which unknown amounts of the substance can be compared and
determined?
a.Reference sample
b.Control
c.Primary standard
d.Secondary standard
Fire extinguishers would be monitored , certified , and dated every
a. Six months
b.Months
c.Years
d.Two years
How often should a spore suspension be used to check the sterility of an autoclave?
a.Once a month
b.Twice a month
c.Every three months
d.None of the above
Countertops should be disinfected at least
a.In the morning before work starts
b.In the evening after work ends
c.Once a week
d.Before work begins and when finished
Under medicare regulations the laboratory report is sent to the physician who requested the test and a
record is kept on file for what period of time?
a.90 days
b.1 year
c.2 years
d.7 years
A mathematical measure of the speed (dispreson) of statistical data above and below the average is called:
a.Reliability
b.Mean
c.Precision
d.Standard deviation
All laboratory reagents should have labels that state:
a.Name of reagent
b.Date received of prepared

A1 PASSERS REVIEW CENTER 320-2728 //

80

c.Expiration date
d.All of the above
81. A budget is a written formal proposal of expected re venues and:
a.`Standards
b.Expenses
c.Requirements
d.Income
82. A laboratory worker splashes concentrated hydrochloric acid in hid/her eye. The best safety measure is to
a.Wash the eye with dilute sodium hydroxide (NaOrt) and call a physician STAT
b.Wash the eye with distilled water and call a physician STAT
c.Wash the eye with dilute acetic acid and call a physician STAT
d.Call the physician and wait for his/her advice
83.
84.
85.
86.
87.
88.
89.
90.
91.
92.
93.
94.
95.
96.
97.
98.
99.
100.
101.
102.

A1 PASSERS REVIEW CENTER 320-2728 //

81

103.
104.
105.
106.
107.
108.
109.
110.
111.
112.
113.
114.
115.
116.
117.
118.
119.
120.
121.
122.
123.
124.
125.
126.
127.
128.
129.
130.
131.
132.
133.
134.
135.
136.
137.
138.
139.
140.
141.
142.
143.
144.
145.
146.
147.
148.
149.

You might also like